You are on page 1of 124

Nguyễn Thành Long

Khoa Toán-tin học,


Đại học Khoa học Tự nhiên Tp. Hồ Chí Minh

GIẢI TÍCH 1A
VI TÍCH PHÂN 1A

TP. Hồ Chí Minh 2021


Mục lục

Mục lục 1

1 TẬP HỢP VÀ ÁNH XẠ 3


1.1 Tập hợp . . . . . . . . . . . . . . . . . . . . . . . . . . . . . . . . . . . . . . . . . . . . . 3
1.2 Quan hệ trong một tập . . . . . . . . . . . . . . . . . . . . . . . . . . . . . . . . . . . . 5
1.3 Ánh xạ . . . . . . . . . . . . . . . . . . . . . . . . . . . . . . . . . . . . . . . . . . . . . . 8
1.4 Bài tập . . . . . . . . . . . . . . . . . . . . . . . . . . . . . . . . . . . . . . . . . . . . . . 11

2 TẬP CÁC SỐ (THỰC, TỰ NHIÊN, NGUYÊN, HỮU TỈ, VÔ TỈ) 13


2.1 Quan hệ thứ tự . . . . . . . . . . . . . . . . . . . . . . . . . . . . . . . . . . . . . . . . . 13
2.2 Chận trên nhỏ nhất (sup) và chận dưới lớn nhất (inf) . . . . . . . . . . . . . . . . 13
2.3 Định nghĩa tập số thực . . . . . . . . . . . . . . . . . . . . . . . . . . . . . . . . . . . . 14
2.4 Tập các số nguyên, tự nhiên, hữu tỉ, vô tỉ . . . . . . . . . . . . . . . . . . . . . . . . 16
2.5 Bài tập bổ sung . . . . . . . . . . . . . . . . . . . . . . . . . . . . . . . . . . . . . . . . 21

3 DÃY SỐ THỰC 23
3.1 Các định nghĩa . . . . . . . . . . . . . . . . . . . . . . . . . . . . . . . . . . . . . . . . . 23
3.2 Các tính chất và các phép tính về giới hạn của dãy số hội tụ . . . . . . . . . . . 24
3.3 Dãy con, dãy Cauchy . . . . . . . . . . . . . . . . . . . . . . . . . . . . . . . . . . . . . 28
3.4 Dãy số tiến ra vô cùng . . . . . . . . . . . . . . . . . . . . . . . . . . . . . . . . . . . . 31
3.5 Giới hạn trên (limsup) và giới hạn dưới (liminf) . . . . . . . . . . . . . . . . . . . . 32
3.6 Bài tập bổ sung . . . . . . . . . . . . . . . . . . . . . . . . . . . . . . . . . . . . . . . . 39

4 GIỚI HẠN HÀM SỐ 41


4.1 Điểm tụ . . . . . . . . . . . . . . . . . . . . . . . . . . . . . . . . . . . . . . . . . . . . . 41
4.2 Giới hạn hàm số tại điểm tụ . . . . . . . . . . . . . . . . . . . . . . . . . . . . . . . . 42
4.3 Giới hạn hàm số tại vô cùng . . . . . . . . . . . . . . . . . . . . . . . . . . . . . . . . 45
4.3.1 Giới hạn hàm số tại +1. . . . . . . . . . . . . . . . . . . . . . . . . . . . . . . . 45
4.3.2 Giới hạn hàm số tại 1. . . . . . . . . . . . . . . . . . . . . . . . . . . . . . . . 46
4.4 Giới hạn vô cùng tại điểm tụ . . . . . . . . . . . . . . . . . . . . . . . . . . . . . . . . 49
4.4.1 Giới hạn vô cùng tại điểm tụ . . . . . . . . . . . . . . . . . . . . . . . . . . . . 49
4.4.2 Giới hạn vô cùng bên phải tại điểm tụ . . . . . . . . . . . . . . . . . . . . . . 49
4.4.3 Giới hạn vô cùng bên trái tại điểm tụ . . . . . . . . . . . . . . . . . . . . . . 49
4.5 Giới hạn vô cùng tại vô cùng . . . . . . . . . . . . . . . . . . . . . . . . . . . . . . . . 50
4.5.1 Định nghĩa giới hạn vô cùng tại +1. . . . . . . . . . . . . . . . . . . . . . . . 50
4.5.2 Định nghĩa giới hạn vô cùng tại 1. . . . . . . . . . . . . . . . . . . . . . . . 50
4.6 Tổng kết các loại giới hạn (15 loại) . . . . . . . . . . . . . . . . . . . . . . . . . . . . 51
4.7 Một số hàm sơ cấp . . . . . . . . . . . . . . . . . . . . . . . . . . . . . . . . . . . . . . 51
4.8 Bài tập bổ sung . . . . . . . . . . . . . . . . . . . . . . . . . . . . . . . . . . . . . . . . 52

1
Chương 0. MỤC LỤC 2

5 HÀM SỐ THỰC LIÊN TỤC 55


5.1 Các định nghĩa . . . . . . . . . . . . . . . . . . . . . . . . . . . . . . . . . . . . . . . . . 55
5.1.1 Điểm tụ, điểm cô lập . . . . . . . . . . . . . . . . . . . . . . . . . . . . . . . . . 55
5.1.2 Hàm số liên tục tại một điểm . . . . . . . . . . . . . . . . . . . . . . . . . . . 55
5.1.3 Hàm số liên tục trong khoảng, trên đoạn, trên một tập . . . . . . . . . . . 56
5.2 Các phép toán trên các hàm số liên tục tại một điểm . . . . . . . . . . . . . . . . 56
5.3 Điểm gián đoạn. Phân loại . . . . . . . . . . . . . . . . . . . . . . . . . . . . . . . . . 56
5.4 Tính liên tục của các hàm sơ cấp . . . . . . . . . . . . . . . . . . . . . . . . . . . . . 57
5.5 Tính chất của hàm liên tục trên một đoạn . . . . . . . . . . . . . . . . . . . . . . . 58
5.6 Hàm số liên tục đều . . . . . . . . . . . . . . . . . . . . . . . . . . . . . . . . . . . . . . 61
5.7 Bài tập bổ sung . . . . . . . . . . . . . . . . . . . . . . . . . . . . . . . . . . . . . . . . 63

6 ĐẠO HÀM 69
6.1 Đạo hàm và các tính chất cơ bản của đạo hàm . . . . . . . . . . . . . . . . . . . . . 69
6.2 Đạo hàm cấp cao . . . . . . . . . . . . . . . . . . . . . . . . . . . . . . . . . . . . . . . 78
6.3 Công thức khai triển Taylor hữu hạn . . . . . . . . . . . . . . . . . . . . . . . . . . . 79
6.4 Công thức khai triển Maclaurin hữu hạn . . . . . . . . . . . . . . . . . . . . . . . . 81
6.5 Ứng dụng của công thức Taylor để tính gần đúng . . . . . . . . . . . . . . . . . . 82
6.6 Các ứng dụng của đạo hàm vào khảo sát hàm số . . . . . . . . . . . . . . . . . . . 82
6.6.1 Tính đơn điệu của hàm số . . . . . . . . . . . . . . . . . . . . . . . . . . . . . . 82
6.6.2 Điều kiện đủ để hàm số cực trị . . . . . . . . . . . . . . . . . . . . . . . . . . 82
6.6.3 Hàm lồi, điểm uốn . . . . . . . . . . . . . . . . . . . . . . . . . . . . . . . . . . 84
6.7 Bài tập bổ sung . . . . . . . . . . . . . . . . . . . . . . . . . . . . . . . . . . . . . . . . 87

7 TÍCH PHÂN RIEMANN 105


7.1 Khái niệm và tính chất căn bản tích phân Riemann . . . . . . . . . . . . . . . . . 105
7.2 Các tính chất cơ bản của phép tính tích phân . . . . . . . . . . . . . . . . . . . . . 110
7.3 Tích phân suy rộng . . . . . . . . . . . . . . . . . . . . . . . . . . . . . . . . . . . . . . 112
7.4 Bài tập bổ sung . . . . . . . . . . . . . . . . . . . . . . . . . . . . . . . . . . . . . . . . 118

BÀI TẬP CHƯƠNG 1 122

Tài liệu tham khảo 123


Chương 1

TẬP HỢP VÀ ÁNH XẠ

1.1 Tập hợp


A1. Khái niệm tập hợp và phần tử, các ký hiệu, cách xác định.
+ Tập hợp và phần tử là khái niệm ban đầu không có định nghĩa, chỉ nêu ra các hình ảnh minh họa
để diễn tả chúng.
Để chỉ x là một phần tử của tập hợp X, ta viết x 2 X, và đọc là x thuộc X hay x trong X:
Để chỉ x không là một phần tử của tập hợp X, ta viết x 2 = X, và đọc là x không thuộc X.
+ Tập hợp trống là tập hợp không chứa phần tử nào cả.
Về tên gọi, thay vì đọc là tập hợp ta có thể đọc gọn lại là tập.
Các ký hiệu thay câu nói:
8 Mọi, với mọi, với tất cả
9 Có, tồn tại,
@ Không có, không tồn tại,
9! Có duy nhất một, có một và chỉ một, tồn tại và duy nhất,
: (dấu hai chấm) đôi khi thay lời nói "sao cho".
Ví dụ 1:
8x 2 X thay vì nói "với mọi phần tử x trong X "
9x 2 X thay vì nói "có (tồn tại) một phần tử x trong X "
9!x 2 X thay vì nói "có (tồn tại) duy nhất một phần tử x trong X "
@x 2 X thay vì nói "không có (không tồn tại) phần tử x trong X "
9x 2 X : x thỏa tính chất P thay vì nói "có một phần tử x trong X sao cho x thỏa tính chất
P ".
+ Xác định một tập.
Để xác định một tập A ta có thể dùng các phương pháp sau:
(i) Liệt kê tất cả các phần tử của A;
(ii) Nêu tính chất của các phần tử của tập hợp A:
Một mệnh đề toán học chỉ lấy hai chân trị: Đúng hoặc sai, không xảy ra các trường hợp: vừa đúng
vừa sai, vừa không đúng và vừa không sai.
Cho X là một tập khác trống, P (x) là mệnh đề toán học phụ thuộc vào x 2 X.

A = fx 2 X : P (x) đúngg:

(iii) Dùng giản đồ để diễn tả tập A:


A2. Tạm thời chúng ta sử dụng các tập hợp thông dụng như dưới đây (các tập này sẽ xác định rõ
sau khi xây dựng tập số thực ở phần sau)
- Tập các số tự nhiên (nguyên dương) N = f1; 2; 3; g;
- Tập các số nguyên Z = f 3; 2; 1; 0; 1; 2; 3; g;
m
- Tập các số hữu tỉ Q = f : m 2 Z và n 2 Ng;
n

3
Chương 1. TẬP HỢP VÀ ÁNH XẠ 4

- Tập các số thực R,


- Tập các số phức C = fx + iy : x và y trong Rg.
A3. Các phép toán về tập: Phần giao, phần hội, phần hiệu, phần bù của các tập.
Định nghĩa 1. Cho hai tập A và B.
Ta nói A chứa trong B nếu mọi phần tử của A đều thuộc B; khi đó ta nói A là tập con của B và ký
hiệu A B: Trong trường hợp này ta cũng có thể nói B chứa A hay B bao hàm A, ký hiệu là B A:
Ta nói A bằng B nếu A B và B A; khi đó ta ký hiệu A = B:
Cho tập X và A; B là hai tập con của X.
Tập fx 2 X : x 2 A và x 2 Bg, gọi là phần giao của A và B và ký hiệu là A \ B:
Ta nói A và B rời nhau nếu A \ B = :
Tập fx 2 X : x 2 A hoặc x 2 Bg, gọi là phần hợp của A và B và ký hiệu là A [ B.
Tập fx 2 X : x 2 A và x 2 = Bg gọi là phần hiệu của A đối với B và ký hiệu là A r B.
Nếu B A, ta gọi A r B là phần bù của B trong A.
Đặc biệt X r B = fx 2 X : x 2 = Bg gọi là phần bù của B trong X, có ký hiệu là {X B hay B c :
Ví dụ 1: Đặt A = fx 2 R : sin x = 0g và B = fx 2 R : 2x2 + x 3 = 0g.
sin x = 0;
A \ B là tập các nghiệm của hệ phương trình
2x2 + x 3 = 0:
A [ B là tập các nghiệm của phương trình (2x2 + x 3) sin x = 0.
Cho tập A, ký hiệu là P (A) là tập tất cả các tập con của A. Có nơi dùng ký hiệu 2A để chỉ P (A):
Ví dụ 2: (Xem như Bài tập). Cho A; B và C là ba tập con khác trống của tập X sao cho A B và
B C. Chứng minh A C.
Ví dụ 3: A = f1; a; bg, lúc đó P (A) = f ; f1g; fag; fbg; f1; ag; f1; bg; fa; bg; Ag:
Ví dụ 4: A = f1; a; b; cg, lúc đó P (A) = f ; f1g; fag; fbg; fcg; f1; ag; f1; bg; f1; cg; fa; bg; fb; cg;
fa; cg; f1; a; bg; f1; a; cg; f1; b; cg; fa; b; cg; Ag:
Định nghĩa 2. Cho A và B là hai tập, tích Descartes của hai tập A và B là tập tất cả các cặp (x; y)
với x 2 A và y 2 B và ký hiệu nó là A B:
Ví dụ 5: A = f1; 2g và B = fa; b; cg, khi đó

A B = f(1; a); (1; b); (1; c); (2; a); (2; b); (2; c)g;
B A = f(a; 1); (a; 2); (b; 1); (b; 2); (c; 1); (c; 2)g:

Hình vẽ dưới đây minh họa các phần tử của hai tập A B và B A

c (1; c) (2; c)
b (1; b) (2; b) 2 (a; 2) (b; 2) (c; 2)
a (1; a) (2; a) 1 (a; 1) (b; 1) (c; 1)
1 2 a b c
A B B A

Nếu A = B, ta thường ký hiệu A A là A2 . Lúc đó A2 là tập tất cả các cặp (x; y) với x 2 A và y 2 A:
Ta cũng chú ý trong trường hợp này là (x; y) có thể khác (y; x), thí dụ như A = f1; 2; ag, khi đó

A2 = f(1; 1); (1; 2); (1; a); (2; 1); (2; 2); (2; a); (a; 1); (a; 2); (a; a)g;

mà phần tử (1; 2) khác phần tử (2; 1):


A4. Giao, hội của họ các tập.
Định nghĩa 3. Cho I là tập 6= : Nếu ứng với mỗi i 2 I, ta có tập con Ai X; khi đó ta nói là ta
có một họ các tập con của X, ký hiệu họ này là fAi gi2I hoặc fAi ; i 2 Ig hoặc (Ai )i2I hoặc (Ai ; i 2 I):
Ta còn gọi I là tập các chỉ số. T T
(i) Ta gọi phần giao của họ tập fAi gi2I là một tập con của X, ký hiệu là Ai hoặc i2I Ai và được
i2I
xác định bởi \
Ai = fx 2 X : x 2 Ai 8i 2 Ig:
i2I
Chương 1. TẬP HỢP VÀ ÁNH XẠ 5

T
Điều này có nghĩa là x 2 i2I Ai () x 2 Ai 8i 2 I: S S
(ii) Ta gọi phần hội của họ tập fAi gi2I là một tập con của X, ký hiệu là Ai hoặc i2I Ai và được
i2I
xác định bởi [
Ai = fx 2 X : 9i 2 I; x 2 Ai g:
i2I
S
Điều này có nghĩa là x 2 i2I Ai () 9i 2 I : x 2 Ai :
Trường hợp I = N, ta gọi fAi gi2N là dãy các tập con của X, (hoặc họ đếm được các tập con của X)
T
1 T T S
1 S S
ta viết Ai hoặc 1 i=1 Ai để thay cho i2N Ai , và viết Ai hoặc 1i=1 Ai để thay cho i2N Ai :
i=1 i=1
Ví dụ 6: (Xem như Bài tập).
T S
(i) {X Si2I Ai = Ti2I {X Ai ;
(ii) {X i2I Ai = i2I {X Ai :
Chú ý: Tcó thể đổi kýShiệu cnhư sau
c
(i) Si2I A i = Ti2I Aic ;
c
(ii) A
i2I i = i2I Ai :
Giải Ví dụ 5 (i).
\ \
x 2 {X Ai () x 2 X và x 2
= Ai
i2I i2I
() x 2 X và 9i 2 I : x 2
= Ai () 9i 2 I : x 2 X và x 2
= Ai
[
() 9i 2 I : x 2 {X Ai () x 2 { X Ai :
i2I

Giải Ví dụ 5 (ii). Tương tự.

1.2 Quan hệ trong một tập


Định nghĩa 4. Cho một tập hợp A khác trống. Ta nói rằng một quan hệ trong A là một tập con
R 6= của A2 . Ta viết xRy để chỉ (x; y) là phần tử của R, tức là (x; y) 2 R.
Ví dụ 7: Ví dụ về các quan hệ trong A = f1; 2; 3g, khi đó

A2 = f(1; 1); (1; 2); (1; 3); (2; 1); (2; 2); (2; 3); (3; 1); (3; 2); (3; 3)g:

Xét các quan hệ trong A như sau:


1/ R1 = f(x; y) 2 A2 : x < yg = f(1; 2); (1; 3); (2; 3)g;

xR1 y () (x; y) 2 R1 () x < y (R1 cũng ký hiệu là <)

2/ R1 = f(x; y) 2 A2 : x > yg = f(2; 1); (3; 1); (3; 2)g;

xR1 y () (x; y) 2 R1 () x > y (R1 cũng ký hiệu là >)

3/ R2 = f(x; y) 2 A2 : x = yg = f(1; 1); (2; 2); (3; 3)g;

xR2 y () (x; y) 2 R2 () x = y (R2 cũng ký hiệu là =)

4/ R3 = f(x; y) 2 A2 : x yg = f(1; 2); (1; 3); (2; 3); (1; 1); (2; 2); (3; 3)g;

xR3 y () (x; y) 2 R3 () x y (R3 cũng ký hiệu là ):

5/ R3 = f(x; y) 2 A2 : x yg = f(2; 1); (3; 1); (3; 2); (1; 1); (2; 2); (3; 3)g;

xR3 y () (x; y) 2 R3 () x y (R3 cũng ký hiệu là ):


Chương 1. TẬP HỢP VÀ ÁNH XẠ 6

Hình ảnh của các quan hệ A2 ; R1 , R1 ; R2 , R3 , R3 trong A như sau

3 (1; 3) (2; 3) (3; 3) 3 (1; 3) (2; 3) 3


2 (1; 2) (2; 2) (3; 2) 2 (1; 2) 2 (3; 2)
1 (1; 1) (2; 1) (3; 1) 1 1 (2; 1) (3; 1)
1 2 3 1 2 3 1 2 3
A 2 R1 R1

3 (3; 3) 3 (1; 3) (2; 3) (3; 3) 3 (3; 3)


2 (2; 2) 2 (1; 2) (2; 2) 2 (2; 2) (3; 2)
1 (1; 1) 1 (1; 1) 1 (1; 1) (2; 1) (3; 1)
1 2 3 1 2 3 1 2 3
R2 R3 R3
A6. Quan hệ R được gọi là đối xứng nếu " 8(x; y) 2 A2 ; xRy =) yRx"
[i.e. 8(x; y) 2 A2 ; (x; y) 2 R =) (y; x) 2 R ]
Quan hệ R được gọi là phản xạ nếu " xRx 8x 2 A "
[i.e. (x; x) 2 R 8x 2 A ]
Quan hệ R được gọi là phản đối xứng nếu " 8(x; y) 2 A2 ; xRy và yRx thì x = y"
[i.e. 8(x; y) 2 A2 ; (x; y) 2 R và (y; x) 2 R =) x = y ]
Quan hệ R được gọi là truyền (bắc cầu) nếu " 8x; y; z 2 A; xRy và yRz thì xRz"
[i.e. 8x; y; z 2 A; (x; y) 2 R và (y; z) 2 R =) (x; z) 2 R ]
Một quan hệ R được gọi là
(i) Toàn phần nếu “ 8x; y 2 A; thì hoặc xRy hoặc yRx”
(ii) Quan hệ thứ tự nếu R phản xạ, phản đối xứng và truyền.
(iii) Quan hệ thứ tự toàn phần nếu R phản xạ, phản đối xứng, truyền và toàn phần.
(iv) Quan hệ tương đương nếu R phản xạ, đối xứng và truyền.
Bổ túc về suy luận Toán học
Các suy luận liên quan p ^ q; p _ q; p =) q; p () q:
Cho hai mệnh đề toán học p và q.
(i) Mệnh đề toán học p ^ q (đọc là p và q) chỉ đúng khi cả hai p và q cùng đúng:
p q p^q
Đ Đ Đ
Đ S S
S Đ S
S S S
(ii) Mệnh đề toán học p _ q (đọc là p hay q) chỉ sai khi cả hai p và q cùng sai:
p q p_q
Đ Đ Đ
Đ S Đ
S Đ Đ
S S S
(iii) Mệnh đề toán học p =) q (đọc là p suy ra q, p dẫn đến q) chỉ sai khi p đúng và q sai:
p q p =) q
Đ Đ Đ
Đ S S
S Đ Đ
S S Đ
(iv) Mệnh đề toán học p () q (đọc là p tương đương q) chỉ đúng khi cả hai p và q cùng đúng hoặc
cùng sai:
Chương 1. TẬP HỢP VÀ ÁNH XẠ 7

p q p () q
Đ Đ Đ
Đ S S
S Đ S
S S Đ
(v) Mệnh đề toán học ~p (đọc là phủ định p) chỉ đúng khi p sai:
p ~p
Đ S
S Đ
Ví dụ 8: (Xem như Bài tập).
(i) ~(~p) p
(ii) ~(p ^ q) (~p) _ (~q)
(iii) ~(p _ q) (~p) ^ (~q)
(iv) p =) q (~p) _ q
(v) ~(p =) q) p ^ (~q)
(vi) p () q (p =) q) ^ (q =) p)
Giải Ví dụ 8.
p ~p ~(~p) Kết luận
(i) Đ S Đ ~(~p) p
S Đ S
Kết luận: ~(~p) p
p q ~p ~q (~p) _ (~q) p ^ q ~(p ^ q)
Đ Đ S S S Đ S
(ii) Đ S S Đ Đ S Đ
S Đ Đ S Đ S Đ
S S Đ Đ Đ S Đ
Kết luận: ~(p ^ q) (~p) _ (~q)
p q ~p ~q (~p) ^ (~q) p _ q ~(p _ q)
Đ Đ S S S Đ S
(iii) Đ S S Đ S Đ S
S Đ Đ S S Đ S
S S Đ Đ Đ S Đ
Kết luận: ~(p _ q) (~p) ^ (~q)
p q ~p (~p) _ q p =) q
Đ Đ S Đ Đ
(iv) Đ S S S S
S Đ Đ Đ Đ
S S Đ Đ Đ
Kết luận: (p =) q) (~p) _ q
p q ~q p ^ (~q) p =) q ~(p =) q)
Đ Đ S S Đ S
(v) Đ S Đ Đ S Đ
S Đ S S Đ S
S S Đ S Đ S
Kết luận: ~(p =) q) p ^ (~q)
p q p =) q q =) p (p =) q) ^ (q =) p) p () q
Đ Đ Đ Đ Đ Đ
(vi) Đ S S Đ S S
S Đ Đ S S S
S S Đ Đ Đ Đ
Kết luận: (p =) q) ^ (q =) p) (p () q)
Chương 1. TẬP HỢP VÀ ÁNH XẠ 8

Cách viết một mệnh đề.


Cho X là một tập khác trống, 6= A X, P (x) là mệnh đề toán học phụ thuộc vào x 2 A.
(i) Xét mệnh đề (mđ1) "P (x) đúng với mọi x 2 A "
Ta viết mệnh đề phủ định của (mđ1) như sau:
Mệnh đề (mđ1) được viết lại như sau:
(mđ1) "8x 2 X; [ x 2 A =) P (x) đúng ] "
"8x 2 X; p(x) =) q(x)",
trong đó p và q là hai mệnh đề sau p(x) " x 2 A " và q(x) " P (x) đúng ".
Theo bài tập trên thì ~(p(x) =) q(x)) p(x) ^ (~q(x));
tức là, ~(mđ1) "9x 2 X : ~[x 2 A =) P (x) đúng]"
9x 2 X : (x 2 A) ^ ~((P (x) đúng)
9x 2 A và P (x) sai

~[P (x) đúng 8x 2 A] 9x 2 A và P (x) sai

(ii) Xét mệnh đề (mđ2) " tồn tại x 2 A sao cho P (x) đúng"
Mệnh đề (mđ2) được viết lại như sau:
(mđ2) "9x 2 X : [ x 2 A =) P (x) đúng ] "
"9x 2 X : [p(x) =) q(x)]",
trong đó p và q là hai mệnh đề sau p(x) " x 2 A " và q(x) " P (x) đúng ".
Khi đó, ~(mđ2) "8x 2 X : ~[x 2 A =) P (x) đúng]"
8x 2 X : (x 2 A) ^ ~((P (x) đúng)
8x 2 A và P (x) sai "P (x) sai 8x 2 A"

~[9x 2 A : P (x) đúng] P (x) sai 8x 2 A:

Cho X là một tập khác trống, 6= A; B X, P (x; y) là mệnh đề toán học phụ thuộc vào x 2 A;
y 2 B.
(iii) Xét mệnh đề (mđ3) "8x 2 A; 9y 2 B : P (x; y) đúng"
Ta viết mệnh đề phủ định của (mđ3) như sau.

~[8x 2 A; 9y 2 B : P (x; y) đúng]


[9x 2 A : 8y 2 B =) P (x; y) sai ] (mđ3*)
[9x 2 A : P (x; y) sai 8y 2 B]:

Mệnh đề (mđ3*) được viết lại như sau:


(mđ3*) "9x 2 A : 8y; [ y 2 B =) P (x; y) sai ] "
"9x 2 A : 8y; [ p(y) =) q(x; y) ] ",
trong đó p(y) và q(x; y) là hai mệnh đề sau p(y) " y 2 B " và q(x; y) " P (x; y) sai ".
Khi đó, ~(mđ3*) "8x 2 A : 9y; [ y 2 B và P (x; y) đúng ]"
8x 2 A; 9y 2 B : P (x; y) đúng.
(iv) Xét mệnh đề (mđ4) "9x 2 A : P (x; y) đúng 8y 2 B":
Tương tự, phủ định của (mđ4) như sau

~[9x 2 A : P (x; y) đúng 8y 2 B] ~[9x 2 A : 8y 2 B =) P (x; y) đúng]


[8x 2 A; 9y 2 B : P (x; y) sai].

1.3 Ánh xạ
Định nghĩa 5. Cho X và Y là hai tập khác trống. Giả sử với mọi x 2 X ta xác định duy nhất một phần tử
f (x) 2 Y; ta nói ta xác định được một ánh xạ f từ X vào Y . Lúc đó X được gọi là miền xác định của ánh xạ f ,
(hoặc X còn gọi là tập đi ), Y là tập đến, hoặc tập giá trị của f ).
Chương 1. TẬP HỢP VÀ ÁNH XẠ 9

Ký hiệu f : X ! Y để chỉ f là ánh xạ từ X vào Y; và ký hiệu x 7 ! f (x) để chỉ cách xác định ánh
xạ f; ta cũng viết lại như sau
f :X !Y
x 7 ! f (x):
Định nghĩa 6. Cho ánh xạ f : X ! Y:
(i) Tập Gf = f(x; f (x)) 2 X Y : x 2 Xg gọi là đồ thị của f:
(ii) Cho D là một tập con khác trống trong X: Tập f (D) = ff (x) : x 2 Dg được gọi là tập hợp ảnh
của D bởi f .
Đặc biệt D = X; tập f (X) gọi là tập ảnh của f; ký hiệu là Im(f ); vậy

Im(f ) = f (X) = ff (x) : x 2 Xg:

Đặc biệt, nếu D = ; ta đặt f ( ) = :


(iii) Cho E là một tập con khác trống trong Y: Tập fx 2 X : f (x) 2 Eg được gọi là tập hợp ảnh
ngược của E bởi f , ký hiệu là f 1 (E); vậy
1
f (E) = fx 2 X : f (x) 2 Eg:

Đặc biệt, nếu E = ; ta đặt f 1 ( ) = :


(iv) Cho D là một tập con khác trống trong X: Với mọi x 2 D; ta đặt g(x) = f (x); khi đó g : D ! Y
và ta nói g là ánh xạ thu hẹp của ánh xạ f trên D và ký hiệu g là f jD.

f :X !Y
x 7 ! f (x):
f jD : D ! Y
x 7 ! (f jD)(x) = f (x):

Ví dụ 9: Cho ánh xạ f : R ! R như sau

x; x > 0;
f (x) =
0; x 0:

Đặt D1 = (0; +1) và D2 = ( 1; 0]; ta có (f jD1 )(x) = x; 8x 2 D1 và (f jD2 )(x) = 0; 8x 2 D2 :


Định nghĩa 7. Cho X; Y và Z là ba tập khác trống, f : X ! Y và g : Y ! Z: Ta đặt
h(x) = g(f (x)) với mọi x 2 X. Khi đó h : X ! Z và được gọi là ánh xạ hợp của f và g và được ký
hiệu là g f . Vậy (g f )(x) = g(f (x)) với mọi x 2 X.

f g
X ! Y ! Z
x 7 ! f (x) 7 ! g(f (x)) = (g f )(x)
g f
X ! ! ! Z
& f g %
& %
Y

Ví dụ 10: Cho hai ánh xạ f : (0; +1) ! R và g : R ! R như sau

f (x) = ln3 x; 8x 2 (0; +1);


g(x) = 1 + sin2 x; 8x 2 R:

Khi đó g f : (0; +1) ! R được xác định bởi

(g f )(x) = g(f (x)) = 1 + sin2 ln3 x ; 8x 2 (0; +1):


Chương 1. TẬP HỢP VÀ ÁNH XẠ 10

Còn việc xác định f g như thế nào? Theo công thức

(f g)(x) = f (g(x)); 8x 2 R;

thì để cho f (g(x)) xác định thì g(x) phải nằm trong miền xác định của f , tức là g(x) > 0 với mọi 8x 2 R;
mà điều này luôn thỏa vì g(x) = 1 + sin2 x 1 > 0; 8x 2 R; do đó ta có f g : R ! R được xác định
như sau
(f g)(x) = f (g(x)) = ln3 1 + sin2 x ; 8x 2 R:
Ví dụ 11: Cho hai ánh xạ f : [0; +1) ! R và g : R ! R như sau
p
f (x) = x; 8x 2 [0; +1);
g(x) = x x2 ; 8x 2 R:

Dễ thấy rằng g f : [0; +1) ! R được xác định bởi


p
(g f )(x) = g(f (x)) = f (x) (f (x))2 = x x; 8x 2 [0; +1):

Tuy nhiên, việc xác định f g thì cần chú ý g(x) phải nằm trong miền xác định của ý, tức là g(x) 0;
mà điều này tương đương với g(x) = x x2 0; hay 0 x 1: Do đó, ta có f g : [0; 1] ! R được xác
định như sau p p
(f g)(x) = f (g(x)) = g(x) = x x2 ; 8x 2 [0; 1]:
Định nghĩa 8. Cho ánh xạ f : X ! Y:
(i) Ta nói f là một đơn ánh nếu

8x; y 2 X; f (x) = f (y) =) x = y;

hay diễn đạt bằng cách khác


8x; y 2 X; x 6= y =) f (x) 6= f (y):
hoặc
8x; y 2 X; ta có f (x) 6= f (y) khi x 6= y:
Chú ý ánh xạ f : X ! Y không đơn ánh được diễn đạt bằng mệnh đề phủ định như sau

~[8x; y 2 X; x 6= y =) f (x) 6= f (y)];

hay tương đương với


9x; y 2 X : x 6= y và f (x) = f (y):
(ii) Ta nói f là một toàn ánh nếu f (X) = Y
hay diễn đạt bằng cách khác:

f (X) = Y
() [f (X) Y và Y f (X)] () Y f (X)
() [8y 2 Y =) y 2 f (X)]
() [8y 2 Y =) (9x 2 X : f (x) = y)]
() [8y 2 Y; 9x 2 X : f (x) = y] :

(iii) Ta nói f là một song ánh nếu f đơn ánh và toàn ánh.
Cả 3 loại ánh xạ trên, ta có thể làm nó tương đương với các mệnh đề sau:

f : X ! Y toàn ánh () 8y 2 Y; phương trình f (x) = y có nghiệm trong X;


f : X ! Y đơn ánh () 8y 2 Y; phương trình f (x) = y có nhiều nhất là một nghiệm trong X;
f : X ! Y song ánh () 8y 2 Y; phương trình f (x) = y có duy nhất một nghiệm trong X:
Chương 1. TẬP HỢP VÀ ÁNH XẠ 11

Định nghĩa 9. Cho f : X ! Y là một song ánh. Với mọi y 2 Y ta có duy nhất một x 2 X sao cho
f (x) = y, ta đặt g(y) = x. Ta thấy rằng g : Y ! X có tính chất sau: (g f )(x) = x và (f g)(y) = y;
8x 2 X và 8y 2 Y: Ta nói g là ánh xạ ngược của f và ký hiệu là f 1 .
Vậy
(f 1 f )(x) = x và (f f 1 )(y) = y; 8x 2 X và 8y 2 Y:
Xét ánh xạ IX : X ! X cho bởi (IX )(x) = x; 8x 2 X: Ta gọi IX là ánh xạ đồng nhất trên X. Khi
đó, ta có
f 1 f = IX và f f 1 = IY :

1.4 Bài tập


1. Cho A; B X và 6= : Chứng minh rằng:
(i) X = A [ (X r A);
(ii) (X r A) [ (X r B) [ (A \ B) = X;
(iii) Nếu X = A [ B và A \ B = ; thì X = (X r A) [ (X r B) và X r A = B; X r B = A;
2. Cho f (x) = x2 : Chứng minh rằng f : R ! R không toàn ánh, không đơn ánh.
3. Cho f : X ! Y và A; B X: Chứng minh rằng:
(i) Nếu A B; thì f (A) f (B);
(ii) f (A [ B) = f (A) [ f (B);
(iii) f (A \ B) f (A) \ f (B);
(iv) Nếu f đơn ánh thì f (A \ B) = f (A) \ f (B):
(v) Nếu f (A \ B) = f (A) \ f (B); 8A; B X thì f đơn ánh.
4. Tìm một song ánh từ [0; 1] vào (0; 1):
5. Tìm một song ánh từ [0; 1] vào [0; 1):
6. Cho f : X ! Y và A; B Y: Chứng minh rằng:
(i) f 1 (A [ B) = f 1 (A) [ f 1 (B);
(ii) f 1 (A \ B) = f 1 (A) \ f 1 (B):
7. Cho f : S X ! Y vàSfAi gi2I ; là họ các tập con của X: Chứng minh rằng:
(i) f ( Ti2I Ai ) = Ti2I f (Ai );
(ii) f ( i2I Ai ) i2I f (ATi ); T
(iii) Nếu f đơn ánh thì f ( i2I Ai ) = i2I f (Ai ):
8. Cho f : XS ! Y và fB S i gi2I ; 1là họ các tập con của Y: Chứng minh rằng:
1
(i) f ( Ti2I Bi ) = Ti2I f (Bi );
(ii) f 1 ( i2I Bi ) = i2I f 1 (Bi );
9. Cho hai ánh xạ f : X ! Y và g : Y ! Z: Chứng minh rằng:
(i) Nếu f; g đơn ánh thì g f đơn ánh.
(ii) Nếu f; g toàn ánh thì g f toàn ánh.
(iii) Nếu f; g song ánh thì g f song ánh.
(iv) Nếu f; g song ánh thì (g f ) 1 = f 1 g 1 .
10. [Sẽ làm sau khi học xong chương tập số thực] Cho a 2 R sao cho jaj < "; 8" > 0: Chứng minh
rằng a = 0:
1 1 1
11. [Sẽ làm sau khi học xong chương tập số thực] Với mỗi n 2 N; đặt An = ( ; ) = fx 2 R : <
n n n
1
x < g:
n
T
1
(i) Chứng minh rằng An = f0g;
n=1
S
1
(ii) Xác định tập An :
n=1
1 1 1
12. [Sẽ làm sau khi học xong chương tập số thực] Với mỗi n 2 N; đặt An = ( 2 ; ) = fx 2 R : 2 <
n n n
1
x < g: Chứng minh rằng
n
Chương 1. TẬP HỢP VÀ ÁNH XẠ 12

T
1 S
1
(i) An = f0g; (ii) An = A1 :
n=1 n=1
Chương 2

TẬP CÁC SỐ (THỰC, TỰ NHIÊN,


NGUYÊN, HỮU TỈ, VÔ TỈ)

2.1 Quan hệ thứ tự


Định nghĩa 1. Cho tập A và từ đó có tập tích Descartes A A = A2 = f(x; y) : x; y 2 Ag. Một tập
con khác rỗng R A A được gọi là một quan hệ thứ tự trên A, nếu
i) R phản xạ: (x; x) 2 R; 8x 2 A,
ii) R truyền (bắc cầu): 8x; y; z 2 A, (x; y) 2 R và (y; z) 2 R =) (x; z) 2 R,
iii) R phản đối xứng: 8x; y 2 A, (x; y) 2 R và (y; x) 2 R =) x = y.
Nếu R là một quan hệ thứ tự trên A, ta dùng ký hiệu
x y thay cho (x; y) 2 R, đọc là x nhỏ hơn hay bằng y:
y x thay cho (x; y) 2 R, đọc là y lớn hơn hay bằng x:
x < y thay cho "(x; y) 2 R và x 6= y", đọc là x nhỏ hơn y:
y > x thay cho "(x; y) 2 R và x 6= y", đọc là y lớn hơn x:
Người ta cũng đồng nhất R và viết x y thay cho (x; y) 2 R.
Nếu 8x; y 2 A, ta có x y hay x > y được nghiệm đúng thì ta nói R là một quan hệ thứ tự toàn
phần, tập A cũng được gọi là tập thứ tự toàn phần.
Một tập A được gọi là tập thứ tự nếu nó có một quan hệ thứ tự trên A.
Nếu a 2 A, sao cho a x, 8x 2 A, thì ta nói a là phần tử nhỏ nhất của A, ký hiệu a = min A.
Nếu b 2 A, sao cho x b, 8x 2 A, thì ta nói b là phần tử lớn nhất của A, ký hiệu b = max A.

2.2 Chận trên nhỏ nhất (sup) và chận dưới lớn nhất (inf)
Định nghĩa 2. Cho A là một tập hợp thứ tự với quan hệ thứ tự " " và cho X A.
Nếu a 2 X, sao cho a x, 8x 2 X, thì ta nói a là phần tử nhỏ nhất của X, ký hiệu a = min X.
Nếu b 2 X, sao cho x b, 8x 2 X, thì ta nói b là phần tử lớn nhất của X, ký hiệu b = max X.
Ta nói:
X bị chận trên nếu tồn tại b 2 A sao cho x b, 8x 2 X, phần tử b được gọi là một chận trên của
X.
X bị chận dưới nếu tồn tại a 2 A sao cho a x, 8x 2 X, phần tử a được gọi là một chận dưới
của X.
X bị chận nếu X bị chận trên và bị chận dưới, tức là tồn tại a; b 2 A sao cho a x b, 8x 2 X.
Cho X A bị chận trên. Nếu tồn tại ^b 2 A là một chận trên của X sao cho ^b b với mọi chận
trên b của X, thì ta nói ^b là một chận trên nhỏ nhất của X; tức là,
^b = minfb 2 A : b là chận trên của Xg = minfb 2 A : x b; 8x 2 Xg:

Ta có thể nghiệm lại rằng, nếu chận trên nhỏ nhất của X tồn tại thì chận trên nhỏ nhất ấy là duy
nhất.

13
Chương 2. TẬP CÁC SỐ (THỰC, TỰ NHIÊN, NGUYÊN, HỮU TỈ, VÔ TỈ) 14

Thật vậy, giả sử ^b1 và ^b2 là hai chận trên nhỏ nhất của X; khi đó, do ^b2 là một chận trên của X; ta
có b1 ^b2 :
^
Mặt khác, do ^b1 là một chận trên của X; ta có ^b2 ^b1 : Do tính phản đối xứng của quan hệ thứ tự
ta có ^b1 = ^b2 :
Ta ký hiệu chận trên nhỏ nhất của X là ^b = sup X.
Tóm tắt đều nầy bởi: Nếu chận trên nhỏ nhất của X tồn tại thì

sup X = minfb 2 A : x b; 8x 2 Xg:

Cho X A bị chận dưới. Nếu tồn tại a ^ 2 A là một chận dưới của X sao cho a a
^ với mọi chận
dưới a của X, thì ta nói a
^ là một chận dưới lớn nhất của X; tức là,

a
^ = maxfa 2 A : a là chận dưới của Xg = maxfa 2 A : a x; 8x 2 Xg:

Ta có thể nghiệm lại rằng (lý luận tương tự như trên), nếu chận dưới lớn nhất của X tồn tại thì chận
dưới lớn nhất ấy là duy nhất. Ta ký hiệu chận dưới lớn nhất của X là a^ = inf X.
Vậy, nếu chận dưới lớn nhất của X tồn tại thì

inf X = maxfa 2 A : a x; 8x 2 Xg:

2.3 Định nghĩa tập số thực


Định nghĩa 3. Cho tập R 6= có một quan hệ thứ tự toàn phần " ", có trang bị hai ánh xạ (+)
(phép toán cộng) và ( ) (phép toán nhân), như sau đây:
Ánh xạ (+):
+:R R!R
(a; b) 7 ! +(a; b) = a + b;
mà ánh xạ (+) này gọi là phép toán cộng (+);
Ánh xạ ( ):
:R R!R
(a; b) 7 ! (a; b) = a b;
mà ánh xạ ( ) này gọi là phép toán nhân ( ).
Một tập R 6= được gọi là tập các số thực nếu R có một quan hệ thứ tự toàn phần " ", có trang bị
hai phép toán cộng (+) và nhân ( ) như trên thỏa các tính chất sau:
(1) a + b = b + a; 8a; b 2 R;
(2) (a + b) + c = a + (b + c); 8a; b; c 2 R;
(3) 90 2 R : a + 0 = a; 8a 2 R;
(4) 8a 2 R; 9( a) 2 R : a + ( a) = 0;
(5) a b = b a; 8a; b 2 R;
(6) (a b) c = a (b c); 8a; b; c 2 R;
(7) 91 2 R; 1 6= 0 : 1 a = a; 8a 2 R;
(8) 8a 2 R; a 6= 0; 9a 1 2 R : a 1 a = 1;
(9) a (b + c) = a b + a c; 8a; b; c 2 R;
(10) Quan hệ thứ tự toàn phần" ", có các tính chất sau:
(i) 8a; b 2 R; a > 0; b > 0 =) a + b > 0 và a b > 0;
(ii) 8a; b 2 R; a < b () b + ( a) > 0;
(iii) Mọi tập con 6= và bị chận trên của R, đều có chận trên nhỏ nhất (có sup).
Đặt tên gọi và viết ký hiệu:
Phần tử của R được gọi là số thực.
Phần tử a b của R nếu không sợ nhầm lẫn ta viết gọn lại là ab (bỏ dấu chấm).
Phần tử 0 trong (3) được gọi là số không (zêrô).
Phần tử 1 trong (7) được gọi là số một (phần tử đơn vị).
Phần tử ( a) trong (4) ký hiệu lại là a; và được gọi là trừ a, hay số đối của a.
Chương 2. TẬP CÁC SỐ (THỰC, TỰ NHIÊN, NGUYÊN, HỮU TỈ, VÔ TỈ) 15

Phần tử a 1 trong (8) được gọi là nghịch đảo của a, hay a nghịch đảo, hay a lũy thừa trừ một.
Cũng có thể viết nó như là a 1 = a1 được gọi là một chia a, hay là a 1 = 1=a được gọi là một trên a.
Ta cũng có thể viết các ký hiệu gọn hơn như sau:
Phần tử a + ( b) = a b, đọc là a trừ b.
Phần tử ab 1 = ab , đọc là a chia b hay ab 1 = a=b, đọc là a trên b.
a > 0 : ta nói a là số thực dương (hay số dương),
a < 0 : ta nói a là số thực âm (hay số âm),
a 0 : ta nói a là số thực không âm (hay số không âm),
a 0 : ta nói a là số thực không dương (hay số không dương).
Ví dụ 10: (Xem như Bài tập).
(i) Phần tử 0 trong tiên đề (3) là duy nhất.
(ii) Phần tử đối của a trong tiên đề (4) là duy nhất.
(iii) ( a) b = (a b):
(iv) ( 1) a = a:
(v) ( 1) ( 1) = ( 1) = 1:
(vi) 1 > 0:
(vii) a > 0 =) a 1 > 0:
(viii) a b =) m a m b; 8m > 0:
Giải Ví dụ 10.
(i) Giả sử có 00 2 R sao cho a + 00 = a; 8a 2 R: Ta chứng minh rằng 00 = 0:
Lấy a = 0 2 R; ta có: 00 = 00 + 0 = 0 + 00 = 0:
(ii) Giả sử có a0 2 R sao cho a + a0 = 0: Ta chứng minh rằng a0 = a:
a0 = a0 + 0 = a0 + (a + ( a)) = (a0 + a) + ( a) = 0 + ( a) = ( a) = a:
(iii) [a + ( a)] b = 0 b = 0 () a b + ( a) b = 0:
Vậy ( a) b = 0 là phần tử đối của a b; tức là ( a) b = (a b):
(iv) [1 + ( 1)] a = 0 a = 0 () 1 a + ( 1) a = 0
() a + ( 1) a = 0:
Vậy ( 1) a là phần tử đối của a; tức là ( 1) a = a:
(v) ( 1) [1 + ( 1)] = 0 () ( 1) 1 + ( 1) ( 1) = 0
() 1 ( 1) + ( 1) ( 1) = 0
() ( 1) + ( 1) ( 1) = 0:
Vậy ( 1) ( 1) là phần tử đối của ( 1); tức là ( 1) ( 1) = ( 1) = 1:
(vi) a = 1 6= 0:
Nếu 1 < 0; thì ( 1) = 0 + ( 1) > 0:
Theo tiên đề (10; (i)), ta có 1 = ( 1) ( 1) > 0: Vô lý, vậy 1 > 0:
(vii) a > 0:
Nếu a 1 < 0; thì ( a 1 ) = a 1 > 0
Theo tiên đề (10; (i)), ta có 1 = a 1 a = ( a 1 ) a > 0:
Suy ra 1 < 0: Vô lý, vậy a 1 > 0:
(viii) a b =) m a m b; 8m > 0:
a = b; m > 0 : Hiển nhiên m a = m b m b:
Xét a < b; m > 0; ta có b a > 0; m > 0 =) m (b a) > 0
hay m b + m ( a) > 0
hay m b m a > 0
hay m b > m a
hay m a < m b:
Vài ký hiệu về khoảng số thực. Cho a; b 2 R, a < b, ta định nghĩa
[a; b] = fx 2 R : a x bg : đoạn a; b:
(a; b) = fx 2 R : a < x < bg : khoảng a; b:
[a; b) = fx 2 R : a x < bg : nửa khoảng (phải) a; b:
Chương 2. TẬP CÁC SỐ (THỰC, TỰ NHIÊN, NGUYÊN, HỮU TỈ, VÔ TỈ) 16

(a; b] = fx 2 R : a < x bg : nửa khoảng (trái) a; b:


[a; +1) = fx 2 R : x ag : tia.
(a; +1) = fx 2 R : x > ag : tia.
( 1; b] = fx 2 R : x bg : tia.
( 1; b) = fx 2 R : x < bg : tia.
( 1; +1) = R.
Định nghĩa 4. Cho x0 2 (a; b), 9" > 0 : (x0 "; x0 + ") (a; b). Khoảng (x0 "; x0 + ") còn được
gọi là khoảng tâm x0 bán kính ", hay còn được gọi là một " lân cận của x0 .
Trị số tuyệt đối của a 2 R là một số thực không âm xác định bởi:

a; nếu a 0,
jaj =
a; nếu a < 0:

Tính chất:
(i) jaj 0, 8a 2 R,
(ii) jaj = 0 () a = 0,
(iii) jabj = jaj jbj , 8a; b 2 R,
a jaj
(iv) b = jbj , 8a, b 2 R, b 6= 0,
(v) ja + bj jaj + jbj , 8a; b 2 R.
Định lý 1. Cho X 6= là tập con bị chận trên của R. Khi đó :

x M , 8x 2 X,
M = sup X ()
8" > 0, 9x 2 X : x > M ":

Định lý 2. Cho X 6= là tập con bị chận dưới của R. Khi đó :

m x, 8x 2 X,
m = inf X ()
8" > 0, 9x 2 X : x < m + ":

2.4 Tập các số nguyên, tự nhiên, hữu tỉ, vô tỉ


Định nghĩa 5. Tập Z R; là tập con nhỏ nhất chứa 1 thỏa 4 tính chất sau
(i) a + b = b + a; 8a; b 2 Z;
(ii) (a + b) + c = a + (b + c); 8a; b; c 2 Z;
(iii) 90 2 Z : a + 0 = a; 8a 2 Z;
(iv) 8a 2 Z; 9( a) 2 Z : a + ( a) = 0;
được gọi là tập các số nguyên. Phần tử của Z được gọi là số nguyên.
Định nghĩa 6.
N = fx 2 Z : x > 0g được gọi là tập các số tự nhiên. Phần tử của N được gọi là số tự nhiên.
Z+ = N [ f0g = fx 2 Z : x 0g được gọi là tập các số nguyên không âm. Phần tử của Z+ được gọi
là số nguyên không âm.
Q = f ab : a 2 Z, b 2 Ng được gọi là tập các số hữu tỉ. Phần tử của Q được gọi là số hữu tỉ.
R r Q được gọi là tập các số vô tỉ. Phần tử của R r Q được gọi là số vô tỉ.
Định lý 3. Tập Z+ R; là tập con nhỏ nhất thỏa 2 tính chất sau
(i) 0 2 Z+ ,
(ii) 8n 2 Z+ =) n + 1 2 Z+ .

Chứng minh Định lý 3.


* Hiển nhiên Z+ thỏa 2 tính chất (i) và (ii).
* Giả sử X R thỏa 2 tính chất (i) và (ii), ta sẽ chứng minh rằng Z+ X.
Chương 2. TẬP CÁC SỐ (THỰC, TỰ NHIÊN, NGUYÊN, HỮU TỈ, VÔ TỈ) 17

Ta có: 0 2 X, 0 + 1 = 1 2 X, 1 + 1 2 X, (1 + 1) + 1 2 X,
Tiếp tục quá trình nầy ta có: n 2 X =) n + 1 2 X.
Gọi X 0 = f0, 1, 1 + 1, (1 + 1) + 1, ((1 + 1) + 1) + 1, g. Khi đó X 0 là tập con nhỏ nhất thỏa 2 tính
chất (i) và (ii), (vì X 0 chứa trong mọi tập con của R thỏa 2 tính chất (i) và (ii)). Ta cần chứng minh
rằng X 0 = Z+ .
Đặt Z 0 = X 0 [ f n : n 2 X 0 g. Ta có Z 0 Z vì X 0 Z+ .
Vì Z 0 R thỏa 4 tính chất trong định nghĩa của Z nên Z Z 0 . Vậy Z = Z 0 và cuối cùng ta có
0
X = Z+ .
Chứng minh Định lý 3 hoàn tất.
Chú ý: Z+ = X 0 = f0, 1, 1| {z
+ 1}, (1 + 1) + 1, ((1 + 1) + 1) + 1, g.
| {z } | {z }
2 2+1 3 3+1 4
Về tên gọi:
0 số không
0+1=1 số một
1+1=2 số hai
2+1=3 số ba
2+1=3 số bốn
4+1=5 số năm,
Phép chứng minh qui nạp.
Giả sử với mỗi k 2 Z+ ta có mệnh đề toán học Pk . Đặt A = fk 2 Z+ : Pk đúngg. Khi đó

Pk đúng 8k 2
Z+ () A = Z+
(i) 0 2 A = Z+ ,
()
(ii) 8k 2 A = Z+ =) k + 1 2 A = Z+ .
(i) P0 đúng,
()
(ii) 8k 2 Z+ ; Pk đúng =) Pk+1 đúng.

Phép chứng minh qui nạp dạng khác.


Giả sử với mỗi k 2 Z+ ; k k0 ta có mệnh đề toán học Pk . Khi đó

(i) Pk0 đúng,


Pk đúng 8k 2 Z+ ; k k0 ()
(ii) 8k 2 Z+ ; k k0 ; Pk đúng =) Pk+1 đúng.

Áp dụng trường hợp cho các mệnh đề toán học Qk = Pk+k0 ; k 2 Z+ . Khi đó

Pk đúng 8k 2 Z+ ; k k0 () Qk đúng 8k 2 Z+ :

Ví dụ 11: (Xem như Bài tập). Chứng minh rằng


n(1 + n)
(i) 1+2+ +n= 8n 2 N:
2
n(1 + n)(2n + 1)
(ii) 12 + 22 + + n2 = 8n 2 N:
4
n2 (1 + n)2
(iii) 13 + 23 + + n3 = = (1 + 2 + + n)2 8n 2 N:
4
Định lý 4. Ta có các tính chất sau
1/ Tính chất Archimède: 8x > 0; y > 0; 9n 2 N : nx > y:
2/ Tính trù mật của Q trong R :
8x 2 R; 8" > 0; 9q 2 Q : x " < q < x + ":
3/ Tính trù mật của R r Q trong R :
8x 2 R; 8" > 0; 9r 2 R r Q : x " < r < x + ":
Chương 2. TẬP CÁC SỐ (THỰC, TỰ NHIÊN, NGUYÊN, HỮU TỈ, VÔ TỈ) 18

Chứng minh Định lý 4.


1/ Chứng minh tính chất Archimède. 8x > 0; y > 0; 9n 2 N : nx > y:
(j) Nếu x > y; thì n = 1; ta có nx = x > y:
(jj) Nếu x y; thì ta sẽ chứng minh rằng:

9n 2 N : nx > y: (*)

Giả sử điều (*) không đúng, i.e.


nx y; 8n 2 N: (**)
Đặt A = fn 2 N : nx yg: Ta có A R; A 6= ; vì 1 2 A: Mặt khác A bị chặn trên bởi y x 1 = yx 1 :
Theo tiên đề (10; (iii)), tồn tại c = sup A:
1 1
Với " = = 1 2 1 > 0; 9n0 2 A : c < n0 c:
2 2
1 1
Nếu n0 + 1 2 A thì n0 + 1 sup A = c; do đó c + 1 < n0 + 1 c, mà điều này dẫn đến c + < c:
2 2
Vô lý, vậy n0 + 1 2
= A; tức là (n0 + 1) x > y: Điều này dẫn đến điều mâu thuẫn với (**). Vậy (*) đúng.
2/ Chứng minh tính trù mật của Q trong R; tức là phải chứng minh

8x 2 R; 8" > 0; 9q 2 Q : x " < q < x + ":

(j) Giả sử b > a > 1; b a > 1. Khi đó, ta sẽ chứng minh rằng:

9m 2 N : a < m < b: (#)

Giả sử điều (#) không đúng, i.e.


m2
= (a; b); 8m 2 N: (##)
Đặt A = fm 2 N : m 2 = (a; b)g: Ta có A R; A 6= ; vì 9m 2 N : m > b: Mặt khác A bị dưới bởi 1:
Theo tiên đề (10; (iii)), tồn tại d = inf A:
1 1
Với " = > 0; 9m0 2 A : d m0 < d + :
2 2
1 1 1
Nếu m0 1 2 A thì d = inf A m0 1 < d + 1=d ; mà điều này dẫn đến d < d . Vô lý,
2 2 2
vậy m0 1 2 = A; tức là a < m0 1 < b: Điều này dẫn đến điều mâu thuẫn với (##). Vậy (#) đúng.
(jj) Giả sử b > a > 0; tồn tại n1 ; n2 2 N sao cho: n1 a > 1 và n2 (b a) > 1: Đặt n = maxfn1 ;
n2 g, ta có
n 2 N; na > 1, n(b a) > 1:
m
Áp dụng trường hợp (j), ta có m 2 N : na < m < nb: Do đó a < < b:
n
m
(jjj) Giả sử a < b < 0; khi đó 0 < b < a. Áp dụng trường hợp (jj), ta có q = sao cho
n
m
b<q= < a:
n
b m
(4j) Giả sử a 0 < b; khi đó a 0 < < b. Áp dụng trường hợp (jj), ta có q = sao cho
2 n
b m
a 0< <q= < b:
2 n
a m
(5j) Giả sử a < 0 b; khi đó a < < 0 b. Áp dụng trường hợp (jjj), ta có q = sao cho
2 n
m a
a<q= < < 0 b:
n 2
m
(6j) Giả sử a < b; từ các trường hợp (jj)-(5j) ta cũng suy ra rằng tồn tại q = sao cho
n
m
a<q= < b:
n
m
(7j) Giả sử 8x 2 R; 8" > 0: Áp dụng trường hợp (6j) với a = x ", b = x + "; ta có q = sao
n
cho a = x " < q < b = x + ":
Vậy, tính trù mật của Q trong R được chứng minh.
Chương 2. TẬP CÁC SỐ (THỰC, TỰ NHIÊN, NGUYÊN, HỮU TỈ, VÔ TỈ) 19

Chú ý: Ta có thể làm một chứng minh khác theo cách sau. Ta có
[
R+ = [0; +1) = [n; n + 1):
n2Z+

Cho x 2 R+ ; ta có n0 2 Z+ : n0 x < n0 + 1:
1 k m0 n0 + k
Cho " > 0; tồn tại m0 2 N : < ": Đặt xk = n0 + = 2 Q; k = 0; 1; ; m0 : Khi đó,
[m0 m0 m0 m0
x 2 [n0 ; n0 + 1) = [xk 1 ; xk ); ta có k 2 f1; 2; ; m0 g sao cho xk 1 x < xk :
k=1
Như vậy
1
jxk xj < jxk xk 1j = < ":
m0
Tương tự cho x < 0; ta có x > 0; theo trường hợp trên, với " > 0; ta có x
~k 2 Q sao cho
j~
xk ( x)j < ":
3/ Chứng minh tính trù mật của R r Q trong R; tức là phải chứng minh

8x 2 R; 8" > 0; 9r 2 R r Q : x " < r < x + ":

Giả sử mệnh đề này không đúng, tức là 9x 2 R; 9" > 0 : @r 2 R r Q : x " < r < x + ";
hay 9a; b 2 R; a < b : @r 2 R r Q : a < r < a;
hay 9a; b 2 R; a < b : 8r 2 R r Q =) r 2 = (a; b);
hay 9a; b 2 R; a < b : 8r 2= Q =) r 2
= (a; b);
hay 9a; b 2 R; a < b : (a; b) Q:
= Q; ta chọn 2 số hữu tỉ a0 ; b0 2 Q : a < a0 < b0 < b)
Giả sử a; b 2 Q; a < b : (a; b) Q (nếu a; b 2
Do R r Q 6= ; nên (R r Q) \ [( 1; a) [ (b; +1)] 6= : Mà (R r Q) \ [( 1; a) [ (b; +1)] =
[(R r Q) \ ( 1; a)] [ [(R r Q) \ (b; +1)] ; nên (R r Q) \ [( 1; a) [ (b; +1)] 6= tương đương với
(R r Q) \ ( 1; a) 6= hay (R r Q) \ (b; +1) 6= :
Ta xét hai trường hợp:
(j) Trường hợp 1: (R r Q) \ ( 1; a) 6= : Đặt A = fr 2 R r Q : r < ag. Ta có 6= A R và A
và bị chận trên bởi a, do đó tồn tại rmax = sup A:
1 1
Chọn m 2 N : m(b a) > 1: Với " = > 0; tồn tại a1 2 A : rmax < a1 rmax :
m m
1 1 1 1 1
Nếu a1 + 2 A thì a1 + rmax < a1 + : Điều này vô lý. Vậy a1 + 2
= A; tức là a1 + a:
m m m m m
1 1 1
Do a1 + 2 R r Q và a 2 Q; nên a1 + 6= a: Vậy a1 + > a:
m m m
1 1 1 1
Mặt khác, do a1 rmax a; ta có a < a1 + a+ < b: Vậy a1 + 2 R r Q và a1 + 2 (a; b):
m m m m
Vô lý vì (R r Q) \ (a; b) = :
(jj) Trường hợp 2: (R r Q) \ (b; +1) 6= : Đặt B = fr 2 R r Q : r > bg. Ta có 6= B R và
B và bị chận dưới bởi b, do đó tồn tại rmin = inf B:
1 1
Chọn m 2 N : m(b a) > 1: Với " = > 0; tồn tại b1 2 B : rmin b1 < rmin + :
m m
1 1 1 1 1
Nếu b1 2 B thì b1 rmin > b1 : Điều này vô lý. Vậy b1 2
= B; tức là b1 b:
m m m m m
1 1 1
Do b1 2 R r Q và b 2 Q; nên b1 6= b: Vậy b1 < b:
m m m
1 1 1 1
Mặt khác, do b rmin b1 ; ta có a < b b1 < b: Vậy b1 2 R r Q và b1 2 (a; b):
m m m m
Vô lý vì (R r Q) \ (a; b) = :
Các vô lý này dẫn đến tính trù mật của R r Q trong R được chứng minh.
Ví dụ 12: (Xem như Bài tập). Cho m > 0 và 6= A R và A bị chặn trên. Đặt mA = fmx : x 2 Ag:
Chứng minh sup(mA) = m sup A:
Giải Ví dụ 12. Đầu tiên, A là tập con khác rỗng và bị chận trên của R, nên tồn tại sup A. Tương tự
sup(mA) cũng tồn tại.
Chương 2. TẬP CÁC SỐ (THỰC, TỰ NHIÊN, NGUYÊN, HỮU TỈ, VÔ TỈ) 20

8x 2 A; ta có, mx m sup A; do đó sup(mA) m sup A:


Mặt khác, ta có, x = m 1 mx m 1 sup(mA); 8x 2 A; ta suy ra rằng sup A m 1 sup(mA);
hay m sup A sup(mA): Từ bất đẳng thức trên ta suy ra đẳng thức sup(mA) = m sup A:
Ví dụ 13: (Xem như Bài tập). Cho m > 0 và 6= A R và A bị chặn dưới. Đặt mA = fmx : x 2 Ag:
Chứng minh inf(mA) = m inf A:
Giải Ví dụ 13. Đầu tiên, A là tập con khác rỗng và bị chận dưới của R, nên tồn tại inf A. Tương tự
inf(mA) cũng tồn tại.
8x 2 A; ta có, mx m inf A; do đó inf(mA) m inf A:
Mặt khác, ta có, x = m 1 mx m 1 inf(mA); 8x 2 A; ta suy ra rằng inf A m 1 inf(mA);
hay m inf A inf(mA): Từ bất đẳng thức trên ta suy ra đẳng thức inf(mA) = m inf A:
Ví dụ 13a: (Xem như Bài tập). Cho hai tập con khác trống A và B; bị chận trong R. Giả sử rằng
x y; 8x 2 A; 8y 2 B.
Chứng minh sup A inf B:
Giải Ví dụ 13a. 8x 2 A; ta có x y; 8y 2 B. Do đó x là một chặn dưới của B, suy ra x inf B: Mà
điều này đúng với mọi 8x 2 A:
Do đó inf B là một chặn trên của A, Do đó sup A inf B:
Ví dụ 13b: (Xem như Bài tập). Cho 6= A R và A bị chặn trên. Đặt B = f x : x 2 Ag.
Chứng minh B bị chặn dưới và sup A = inf B:
Ví dụ 13c: (Xem như Bài tập). Cho 6= A R và A bị chặn dưới. Đặt B = f x : x 2 Ag.
Chứng minh B bị chặn trên và sup B = inf A:
Ví dụ 13d: (Xem như Bài tập). Cho 6= A; B R và A và B bị chặn trên.
Chứng minh rằng sup(A [ B) = maxfsup A; sup Bg:
Ví dụ 13e: (Xem như Bài tập). Cho 6= A; B R và A và B bị chặn dưới.
Chứng minh rằng inf(A [ B) = minfinf A; inf Bg:
Ví dụ 13f : (Xem như Bài tập). Cho 6= A B R và B bị chặn trên.
Chứng minh rằng sup A sup B:
Ví dụ 13g: (Xem như Bài tập). Cho 6= A B R và B bị chặn dưới.
Chứng minh rằng inf A inf B:
Ví dụ 13h: (Xem như Bài tập). Cho m 2 R và 6= A; B R: Chứng minh rằng:
(i) Nếu A bị chặn trên thì supfmx : x 2 Ag = m sup A; 8m > 0:
(ii) Nếu A bị chặn trên thì inffmx : x 2 Ag = m sup A; 8m < 0:
Đặc biệt: inff x : x 2 Ag = sup A:
(iii) Nếu A bị chặn dưới thì inffmx : x 2 Ag = m inf A; 8m > 0:
(iv) Nếu A bị chặn dưới thì supfmx : x 2 Ag = m inf A; 8m < 0:
Đặc biệt: supf x : x 2 Ag = inf A:
(v) Nếu A bị chặn trên thì supfm + x : x 2 Ag = m + sup A:
(vi) Nếu A bị chặn dưới thì inffm + x : x 2 Ag = m + inf A:
(vii) Nếu và A và B bị chặn trên thì supfx + y : (x; y) 2 A Bg sup A + sup B:
(viii) Nếu và A và B bị chặn dưới thì inffx + y : (x; y) 2 A Bg inf A + inf B:
Ví dụ 13k: (Xem như Bài tập). Cho a 2 (0; 1): Chứng minh rằng, 8" > 0; 9n 2 N : an < ":
Ví dụ 13l: (Xem như Bài tập). Cho a > 1: Chứng minh rằng, 8M > 0; 9n 2 N : an > M:
Hướng dẫn Ví dụ 13k. A = fan : n 2 Ng =
6 và bị chặn dưới bởi 0. Đặt m = inf A:
8n 2 N; ta có an+1 2 A; do đó an+1 inf A = m:
m m
Từ đó, suy ra an 8n 2 N: Do đó m = inf A : Do a 2 (0; 1); ta suy ra m = 0:
n
a n a
Cho " > 0; 9a 2 A : a < m + " = ":
Chương 2. TẬP CÁC SỐ (THỰC, TỰ NHIÊN, NGUYÊN, HỮU TỈ, VÔ TỈ) 21

1 1
Hướng dẫn Ví dụ 13l. Đặt b = ; ta có b 2 (0; 1): Cho M > 0, với " = > 0; ta có n 2 N : bn < ":
a M
1 1
[Do Ví dụ 13k], hay n = bn < " = ; hay an > M:
a M
Định nghĩa 7. Cho A là một tập 6= , ta nói
(i) A có n phần tử nếu có một song ánh f : f1; 2; ; ng ! A:
Khi đó ta nói tập A có hữu hạn phần tử.
(ii) A là một tập vô hạn đếm được (hoặc vắn tắt là đếm được) nếu có một song ánh f : N ! A.
(iii) A là một tập quá lắm đếm được nếu A có hữu hạn phần tử hoặc vô hạn đếm được.
(iv) A là một tập vô hạn không đếm được (hoặc vắn tắt là không đếm được) nếu A không hữu
hạn và không vô hạn đếm được.
Chú ý
(i) Tập A hữu hạn
(j) Tập A vô hạn đếm được (đếm được)
(ii) Tập A vô hạn:
(jj) Tập A vô hạn không đếm được (không đếm được)
(iii) Tập A quá lắm đếm được nếu A hữu hạn hoặc đếm được.

Ví dụ 14: (Xem như Bài tập). Chứng minh rằng họ P(N) tất cả các tập con của N là một tập vô
hạn không đếm được.
Giải Ví dụ 14. P(N) là vô hạn không đếm được () P(N) không hữu hạn và P(N) không vô hạn
đếm được.
Ta chia bài toán thành hai trường hợp để chứng minh:
Trường hợp 1: P(N) không hữu hạn;
Trường hợp 2: P(N) không vô hạn đếm được.
Chứng minh trường hợp 1: P(N) không hữu hạn: Do P(N) ff1g; f2g; ; fng; g = A mà A là
tập không hữu hạn, do đó P(N) cũng vậy.
Chứng minh trường hợp 2: P(N) không vô hạn đếm được:
Giả sử có một song ánh f : N ! P(N). đặt f (k) = Ak ; 8k 2 N: ta có f (N) = fA1 ; A2 ; ; An ; g=
P(N):
Ta sẽ chứng minh rằng f : N ! P(N) không toàn ánh, nghĩa là:

Tồn tại C 2 P(N) : C 6= f (i); 8i 2 N: (*)

Chọn C = fi 2 N : i 2= Ai g: Ta sẽ chứng minh (*) đúng. Giả sử (*) không đúng, tức là: 9i0 2 N : C =
Ai0 = f (i0 ):
Nếu i0 2 Ai0 = C thì i0 2= Ai0 : Vô lý.
Nếu i0 2
= Ai0 = C thì i0 2 Ai0 : Vô lý.
Vậy (*) đúng, nghĩa là f : N ! P(N) không song ánh, mà điều này dẫn đến P(N) không vô hạn đếm
được.
Kết quả là P(N) là một tập vô hạn không đếm được.

2.5 Bài tập bổ sung


13. Cho A = fa; bg: (i) Viết các phần tử của tập P (A): (ii) Chứng minh rằng tập P (A) hữu hạn:
14. Cho x 2 R; và A = fn 2 Z : n xg: Ta có A 6= và bị chận trên bởi x; do đó tồn tại c = sup A:
Chứng minh rằng:
(i) c 2 Z và c 2 A:
(ii) c x < c + 1:
Hướng dẫn bài 14: (i)
- Nếu x 2 Z; thì do x 2 Z; x x; nên x 2 A; vậy x c x: Suy ra c = x 2 Z; do đó c 2 A:
Chương 2. TẬP CÁC SỐ (THỰC, TỰ NHIÊN, NGUYÊN, HỮU TỈ, VÔ TỈ) 22

- Nếu x 2
= Z : x 6= n; 8n 2 Z: Do đó c = sup A = supfn 2 Z : n < xg: Ta chứng minh rằng c 2 Z.
1 1
Thật vậy, nếu c 2 = Z; thì với " = > 0; 9n0 2 A : c n0 > c :
2 2
Ta khẳng định rằng n0 + 1 2 = A; vì nếu ngược lại thì n0 + 1 2 A; do đó n0 + 1 < c; dẫn đến
1 1
n0 + 1 < c < n0 + : Mà điều này dẫn đến n0 + 1 < n0 + là điều vô lý. Vậy n0 + 1 2 = A; tức là n0 + 1 x:
2 2
Do x 2 = Z; nên n0 + 1 > x: Từ đây suy ra n0 x: Như thế thì c x < n0 c: Mà điều này dẫn đến vô
lý. Vậy c 2 Z và từ đó c 2 A:
Hướng dẫn bài 14: (ii). Theo như (i) thì ta có:
- Nếu x 2 Z; thì c = x;
- Nếu x 2 = Z; thì do c 2 A; ta có c 2 Z và c < x; ta chứng minh rằng x < c + 1: Thật vậy, nếu
x c + 1; thì x > c + 1 (do c + 1 2 Z), như vậy c + 1 2 A; sẽ dẫn đến c + 1 sup A = c: Mà điều này
dẫn đến vô lý. Vậy x < c + 1:
Chú thích. Ta ký hiệu c = [x] là phần nguyên của x:
Tóm lại [x] = supfn 2 Z : n xg = maxfn 2 Z : n xg = là số nguyên lớn nhất nhỏ hơn hay bằng
x: Số này có tính chất
(i) [x] x < [x] + 1; 8x 2 R; (là phần nguyên của x).
(ii) fxg = x [x] 2 [0; 1); 8x 2 R; (là phần lẻ của x).
(iii) fxg = x [x] = 0 () x 2 Z:
Chương 3

DÃY SỐ THỰC

3.1 Các định nghĩa


Định nghĩa 1. Dãy số thực là một ánh xạ x : N ! R: Ta đặt xn = x(n); với mọi n 2 N, ta có thể
dùng một trong các ký hiệu sau để chỉ dãy số thực x :

fxn g; fxn gn2N ; fxn ; n 2 Ng; (xn ); (xn )n2N ; (xn ; n 2 N):

1 cos(n3 )
Ví dụ 1. ; ; fcos(n3 )g; fn + 3n3 g; fcos(n3 )g; là các dãy số thực.
n2 n
Ví dụ 2. Đặt x1 = 1; xn = 3xn 1 + 2; n = 2; 3; khi đó fxn g là một dãy số thực.
Chú ý về việc xác định dãy. Đôi khi người ta cho dãy bắt đầu từ số hạng nào đó trở đi, ví dụ
fxn g; n = 10; 11; ; hay ký hiệu nó là fxn gn 10 hay fx10 ; x11 ; x12 ; g; : Nếu hiểu đúng định nghĩa
dãy số thực là một ánh xạ từ N vào R thì cách xác định fxn gn 10 như trên chưa thể gọi là dãy, vì các
số hạng x1 ; x2 ; ; x10 chưa xác định. Tuy nhiên, nếu đặt x ~n = xn+10 ; n = 1; 2; ; khi đó ta xác định
được một dãy số thực f~ xn g. Như vậy theo thông lệ người ta vẫn nói fxn gn 10 là một dãy số thực theo
nghĩa f~xn g là dãy xác định từ các giá trị x10 ; x11 ; x12 ; :
3n
Ví dụ. xn = ; n = 3; 4; : Như đã nói trên fxn g chưa được gọi là một dãy vì hai số
(n 1)(n 2)
3(n + 2)
hạng x1 ; x2 chưa xác định. Nếu x ~n = xn+2 = ; n = 1; 2; ; khi đó f~xn g là một dãy số thực
(n + 1)n
f~
xn g được xác định được từ các số hạng x3 ; x4 ; trở đi. Các khái niệm sau này (chẳng hạn như sự hội
tụ, tính bị chặn, ) không phụ thuộc vào một số hữu hạn các số hạng đầu tiên.
Định nghĩa 2. Cho fxn g là một dãy số thực. Ta nói dãy fxn g là hội tụ nếu tồn tại a 2 R sao cho:
Với mọi " > 0, tồn tại N 2 N sao cho

8n 2 N; n > N =) jxn aj < ": (1)

Nếu dãy fxn g hội tụ thì số thực a ở trên là duy nhất. Thật vậy, giả sử có hai số thực a; a0 thỏa mệnh
đề (1), tức là, Với mọi " > 0; tồn tại N; N 0 2 N sao cho

8n 2 N; n > N =) jxn aj < "; (2)


0 0
8n 2 N; n > N =) xn a < ":

1
Nếu a 6= a0 ; ta chọn " = ja a0 j > 0, ta có hai số N; N 0 2 N thỏa hai mệnh đề. Với n > maxfN; N 0 g;
2
ta có từ (2) rằng
a a0 ja xn j + xn a0 < " + " = a a0 ;
tức là ja a0 j < ja a0 j : Điều này không thể xảy ra. Vậy a = a0 :

23
Chương 3. DÃY SỐ THỰC 24

Khi dãy fxn g hội tụ, số thực a ở trên gọi là giới hạn của dãy fxn g và ký hiệu nó hoặc viết là

a = lim xn ;
n!1
hay a = lim xn ;
hay xn ! a khi n ! 1;
hay xn ! a:

Dùng các ký hiệu logic ta có thể diễn đạt định nghĩa trên như sau:

xn ! a () 8" > 0; 9N 2 N : 8n 2 N; n > N =) jxn aj < ":

Chú ý rằng, số N tồn tại trên đây nói chung phụ thuộc vào "; do đó ta có thể viết N = N ("): Hơn
cũng không cần thiết N phải là số tự nhiên. Thật vậy, nếu N không là số tự nhiên, dùng tính chất
Archimède ta sẽ thay nó bởi một số tự nhiên lớn hơn nó.
Theo đó, ta viết lại mệnh đề định nghĩa ở trên như sau

xn ! a () 8" > 0; 9N 2 R : 8n 2 N; n > N =) jxn aj < ":

Ta nói dãy fxn g là phân kỳ nếu fxn g không hội tụ.


Chú thích.
(i) Từ định nghĩa ta cũng thấy rằng

xn ! a () xn a ! 0;
jxn j ! 0 () xn ! 0:

(ii) Với dãy hằng số: xn = C; 8n 2 N; ta luôn có xn ! C:


(iii) Từ định nghĩa ta cũng thấy rằng, tính chất hội tụ và giới hạn của một dãy số thực không phụ
thuộc vào một số hữu hạn các số hạng đầu tiên. Để thấy rõ hơn, ta xét dãy số hội tụ fxn g như trên. Cho
trước q 2 N; ta xét một dãy số thực mới f~ xn g được xác định bởi x
~n = xn+q ; n = 1; 2; ; khi đó ta có
thể chứng minh được rằng dãy f~ xn g cũng hội tụ và có cùng giới hạn a với dãy số thực f~xn g. Như vậy
giới hạn a này không phụ thuộc vào q, và cũng không phụ thuộc vào q số hạng đầu tiên x1 ; x2 ; ; xq :
(iv) Nhận xét tương tự cho tính chất phân kỳ cũng vậy.
Định nghĩa 2. Cho fxn g là một dãy số thực. Ta nói
(i) fxn g bị chận trên nếu tập fxn : n 2 Ng bị chận trên, i.e., 9M 2 R : xn M; 8n 2 N;
(ii) fxn g bị chận dưới nếu tập fxn : n 2 Ng bị chận dưới, i.e., 9m 2 R : xn m; 8n 2 N;
(iii) fxn g bị chận nếu fxn g bị chận trên và bị chận dưới, i.e.,
9m; M 2 R : m xn M; 8n 2 N;
hay tương đương với 9M 2 R : jxn j M; 8n 2 N;
(iv) fxn g là dãy tăng nếu xn < xn+1 ; 8n 2 N;
(v) fxn g là dãy giảm nếu xn > xn+1 ; 8n 2 N;
(vi) fxn g là dãy không giảm nếu xn xn+1 ; 8n 2 N;
(vii) fxn g là dãy không tăng nếu xn xn+1 ; 8n 2 N:
(viii) Dãy fxn g thoả 1 trong 4 tính chất (iv)-(vii) được gọi là dãy đơn điệu.
Để phân biệt người ta còn gọi tên đi kèm chữ đơn điệu chẳng hạn như:
đơn điệu tăng, đơn điệu giảm, đơn điệu không giảm, đơn điệu không tăng.
Chú thích. Bốn định nghĩa (iv)-(vii) là 4 khái niệm khác nhau và độc lập, tức là phủ định một
trong 4 định nghĩa (iv)-(vii) không phải là một trong ba định nghĩa còn lại. Ví dụ, dãy không tăng không
có nghĩa là không phải là dãy tăng. Bởi vì dãy fxn g không phải là dãy tăng có nghĩa là: 9n0 2 N :
xn0 xn0 +1 ; trong khi đó để dãy fxn g không tăng thì bất đẳng thức xn xn+1 phải đúng 8n 2 N:

3.2 Các tính chất và các phép tính về giới hạn của dãy số hội tụ
Định lý 1. Giả sử xn ! a:
Chương 3. DÃY SỐ THỰC 25

(i) Nếu a > M; thì 9N 2 N : xn > M; 8n > N ;


(ii) Nếu xn > M; 8n 2 N; thì a M;
(iii) Nếu a < M; thì 9N 2 N : xn < M; 8n > N ;
(iv) Nếu xn < M; 8n 2 N; thì a M;
(v) fxn g bị chận.
Chứng minh Định lý 1 .
(i) Với a > M; chọn " sao cho 0 < " < a M thì a " > M: Với số " này thì

9N 2 N : 8n > N =) a " < xn < a + " =) xn > a " > M:

(ii) Giả sử ngược lại a < M: Khi đó theo (iii), thì 9N 2 N : xn < M; 8n > N: Đặc biệt với n0 = N +1,
ta có xn0 < M: Điều này mâu thuẫn với giả thiết (ii). Vậy (ii) đúng.
(iii) Chứng minh tương tự với (i).
(iv) Chứng minh tương tự với (ii).
(v) Chọn " = 1; 9N 2 N : 8n > N =) jxn aj < 1; từ đó jxn j jxn aj + jaj < 1 + jaj
maxf1 + jaj ; jx1 j ; jx2 j ; ; jxN jg = M; 8n 2 N:
Định lý 1 được chứng minh xong.
Định lý 2. Cho hai dãy hội tụ xn ! a và yn ! b:
(i) Nếu xn < yn 8n 2 N; thì a b:
(ii) Nếu a < b; thì 9N 2 N : 8n 2 N; n > N =) xn < yn :
Chứng minh Định lý 2.
a+b
(i) Giả sử ta có a > b: Lấy một số M = ta có a > M > b: Khi đó theo Định lý 1 (i), với a > M
2
ta có N1 2 N : xn > M; 8n > N1 :
Mặt khác, theo Định lý 1 (iii), với b < M ta có N2 2 N : yn < M; 8n > N2 :
Chọn n0 2 N và n0 > maxfN1 ; N2 g; ta có xn0 > M > yn0 : Điều nầy mâu thuẫn với giả thiết (ii). Do
đó a b:
b a
(ii) " = > 0; N1 2 N : jxn aj < "; jyn bj < "; 8n > N1 :
2
b a b a
Vậy, 8n > N1 ; ta có xn < a + " < a + =b = b " < yn :
2 2
Định lý 2 được chứng minh xong.
Định lý 3. Cho ba dãy fxn g; fyn g và fzn g sao cho
(i) xn yn zn 8n 2 N;
(ii) xn ! a; zn ! a:
Khi đó dãy fyn g cũng hội tụ và yn ! a:
Chứng minh Định lý 3. Do xn ! a; zn ! a; theo định nghĩa sự hội tụ, 8" > 0; ta có N1 ; N2 2 N sao
cho

8n > N1 =) jxn aj < " =) a " < xn < a + ";


8n > N2 =) jzn aj < " =) a " < zn < a + ":

Khi đó, 8n > maxfN1 ; N2 g =) a " < xn yn zn < a + " =) jyn aj < ": Vậy yn ! a:
Định lý 3 được chứng minh xong.
Định lý 4. Cho hai dãy hội tụ xn ! a và yn ! b: Cho k 2 R: Khi đó, các dãy fxn + yn g; fxn yn g;
fkxn g; fjxn jg cũng hội tụ và
(i) xn + yn ! a + b;
(ii) xn yn ! ab;
(iii) kxn ! ka;
(iv) jxn j ! jaj ;
xn a
(v) Nếu b 6= 0 thì tồn tại N 2 N : yn 6= 0; 8n > N và ! :
yn b
Chương 3. DÃY SỐ THỰC 26

Chứng minh Định lý 4.


Chứng minh (i). Theo định nghĩa giới hạn, 8" > 0; ta có N1 ; N2 2 N sao cho

8n > N1 =) jxn aj < "=2;


8n > N2 =) jyn bj < "=2:

Khi đó, 8n > maxfN1 ; N2 g dẫn đến

j(xn + yn ) (a + b)j jxn aj + jyn bj < "=2 + "=2 = ":

Vậy xn + yn ! a + b:
Chứng minh (ii). Ta có

jxn yn abj = jxn (yn b) + b (xn a)j


jxn j jyn bj + jbj jxn aj :

Do fxn g hội tụ, nên fxn g bị chận, ta có M > 0 : jxn j M; 8n 2 N:


Cho " > 0; do xn ! a; ta có N1 2 N sao cho
"
8n > N1 =) jxn aj < :
2 (1 + jbj)

Mặt khác, do yn ! b; ta có N2 2 N sao cho


"
8n > N2 =) jyn aj < :
2 (1 + M )

Khi đó, 8n > maxfN1 ; N2 g dẫn đến

jxn yn abj jxn j jyn


bj + jbj jxn aj
" "
M + jbj
2 (1 + M ) 2 (1 + jbj)
M jbj "
= + < ":
1+M 1 + jbj 2

Vậy xn yn ! ab:
Chứng minh (iii). Xét yn = k; 8n 2 N: Ta có yn ! k: Khi đó theo (ii), thì kxn ! ka:
Chứng minh (iv). Ta có
0 jjxn j jajj jxn aj ! 0:
Do Định lý 3, ta có jxn j jaj ! 0; i.e., jxn j ! jaj :
jbj
Chứng minh (v). Do b 6= 0 và yn ! b; ta có jyn j ! jbj > > 0; do Định lý 1 (i), ta có N1 2 N sao
2
cho
jbj
jyn j > ; 8n > N1 :
2
Với mọi n > N1 ; ta có

xn a b(xn a) + a(b yn )
=
yn b byn
1 jaj
jxn aj + jb yn j
jyn j jbj
2 jaj
jxn aj + jb yn j
jbj jbj
2 jaj
= jxn aj + 2 2 jb yn j :
jbj b
Chương 3. DÃY SỐ THỰC 27

Với mọi " > 0; xn ! a và yn ! b; tồn tại N2 ; N3 2 N; sao cho


2 "
8n > N2 =) jxn aj < ;
jbj 2
b2 "
8n > N3 =) jyn aj < :
4 (1 + jaj)

Khi đó, 8n > maxfN1 ; N2 ; N3 g dẫn đến

xn a 2 jaj
jxn aj + 2 2 jb yn j
yn b jbj b
" jaj 2
b "
+2 2 < ":
2 b 4 (1 + jaj)
xn a
Vậy ! :
yn b
Định lý 4 được chứng minh xong.
Định lý 5 (Tiêu chuẩn Weierstrass). Cho dãy số thực fxn g thỏa một trong 4 điều kiện dưới đây:
(i) fxn g tăng và bị chận trên;
(ii) fxn g không giảm và bị chận trên;
(iii) fxn g giảm và bị chận dưới;
(iv) fxn g không tăng và bị chận dưới;
Khi đó, dãy fxn g hội tụ.
Chứng minh Định lý 5.
Chứng minh (i). Cho fxn g là dãy tăng và bị chận trên. Khi đó tập A = fxn : n 2 Ng 6= và bị chận
trên. Do đó, tồn tại a = sup A: Ta sẽ chứng minh rằng xn ! a:
Cho " > 0; do a = sup A; ta có xn0 2 A : xn0 > a ":
Với mọi n > n0 ; do fxn g là dãy tăng ta có xn > xn0 ; do đó ta suy ra

a " < xn0 < xn a < a + ";

điều này dẫn đến


jxn aj < ":
Vậy xn ! a:
Chứng minh (ii): Một cách tương tự, chứng minh xn ! sup A:
Chứng minh (iii) và (iv): Một cách tương tự, chứng minh xn ! inf A:
Định lý 5 được chứng minh xong.
Chú thích.
(i) Mọi dãy fxn g tăng (hoặc không giảm) và bị chận trên thì hội tụ về supxn :
n2N
(ii) Mọi dãy fxn g giảm (hoặc không tăng) và bị chận dưới thì hội tụ về inf xn :
n2N
Ví dụ 3: (Xem như Bài tập). Cho 6= A R và bị chặn trên. Chứng minh có một dãy fxn g A
hội tụ về sup A.
Giải Ví dụ 3. Do 6= A R và bị chặn trên, nên tồn tại sup A:
1 1
Với mỗi n 2 N; lấy " = ; ta có phần tử xn 2 A (phụ thuộc vào n) sao cho xn > a : Từ đây ta
n n
suy ra
1 1
a < xn a < a + ;
n n
1
tức là jxn aj < ! 0: Vậy xn ! a:
n
Ví dụ 4: (Xem như Bài tập). Cho 6= A R và bị chặn dưới. Chứng minh có một dãy fxn g A
hội tụ về inf A.
Chương 3. DÃY SỐ THỰC 28

3.3 Dãy con, dãy Cauchy


Định nghĩa 3. Cho dãy số thực fxn g và cho fnk g : N ! N là dãy tăng các số tự nhiên, tức là
nk < nk+1 ; 8k 2 N: Ánh xạ hợp fxn g fnk g : N ! R là một dãy số thực được xác định bởi yk = xnk
8k 2 N: Dãy số thực fyk g được gọi là một dãy con của dãy fxn g tương ứng với dãy fnk g: Dãy con fyk g
được ký hiệu lại là fxnk g:
Chú thích.
(i) Bản thân fxn g cũng là dãy con của chính nó.
(ii) Dãy con của dãy con cũng là dãy con của dãy ban đầu, nghĩa là nếu fyk g = fxnk g là dãy con của
dãy fxn g và fzj g = fykj g cũng là dãy con của fxn g. Ta chú ý là zj = xnkj = xqj . Mà qj = nkj là dãy các
số tự nhiên tăng, bởi vì kj < kj+1 , dẫn đến qj = nkj < nkj+1 = qj+1 .
1
Ví dụ 5: Cho dãy số thực fxn g; với xn = ; xét hai dãy fyk g và fzk g như sau:
n
1
yk = x2k = ;
2k
1
zk = x2k 1 = ; 8k 2 N:
2k 1

Ta có fyk g là một dãy con của dãy fxn g tương ứng với dãy nk = 2k; và fzk g là một dãy con của dãy
fxn g tương ứng với dãy nk = 2k 1:
Định lý 6. Mọi dãy con của một dãy hội tụ thì cũng hội tụ và có cùng một giới hạn.
Chứng minh Định lý 6. Cho xn ! a và fxnk g là một dãy con của dãy fxn g: Ta sẽ chứng minh rằng
xnk ! a:
Thật vậy, cho " > 0; do xn ! a; ta có N 2 N : jxn aj < "; 8n > N:
Chú ý rằng nk k; 8k 2 N; ta chọn k0 2 N sao cho k0 > N; khi đó nk k > k0 > N; 8k > k0 :
Do đó, ta suy ra jxnk aj < "; 8k > k0 :
Vậy xnk ! a:
Định lý 6 được chứng minh xong.
Định nghĩa 4. Cho dãy số thực fxn g: Ta nói fxn g là dãy Cauchy nếu

8" > 0; 9N 2 N : 8m; n 2 N; m; n > N =) jxm xn j < ":

Định lý 7. Cho fxn g là một dãy số thực. Khi đó

fxn g hội tụ () fxn g là dãy Cauchy.

Chứng minh Định lý 7.


Chứng minh phần thuận (=)): Cho xn ! a; ta sẽ chứng minh rằng fxn g là dãy Cauchy.
"
Thật vậy, cho " > 0; do xn ! a; ta có N 2 N : jxn aj < ; 8n > N:
2
Do đó, 8m; n 2 N; m; n > N; ta có
" "
jxm xn j jxm aj + ja xn j < + = ":
2 2
Vậy fxn g là dãy Cauchy.
Chứng minh phần đảo ((=): Giả sử fxn g là dãy Cauchy. Ta chứng minh fxn g hội tụ.
Dùng Ví dụ 6 dưới đây, ta chỉ cần chứng minh dãy fxn g chứa một dãy con hội tụ.
Trước hết chúng ta chứng minh dãy fxn g bị chận.
Do fxn g là dãy Cauchy, với " = 1; ta có N 2 N : jxm xn j < 1; 8m; n > N:
Suy ra

jxm j jxm xN +1 j + jxN +1 j 1 + jxN +1 j


max(1 + jxN +1 j ; jx1 j ; jx2 j ; ; jxN jg = M; 8m 2 N:
Chương 3. DÃY SỐ THỰC 29

Dùng định lý Bolzano-Weierstrass (Sẽ chứng minh ở Định lý 8), ta có một dãy con fxnk g của dãy
fxn g hội tụ về một giới hạn a 2 [ M; M ].
Sử dụng Bài tập dưới đây ta có Định lý 7 được chứng minh xong.
Ví dụ 6: (Xem như Bài tập). Cho fxn g là một dãy số thực Cauchy. Giả sử fxn g có một dãy con hội
tụ về a. Chứng minh fxn g hội tụ về a.
Giải Ví dụ 6. Giả sử fxnk g là một dãy con của dãy fxn g sao cho xnk ! a: Ta sẽ chứng minh rằng
xn ! a:
Thật vậy, cho " > 0:
"
Do fxn g là một dãy số thực Cauchy, nên có N 2 N : jxm xn j < ; 8m; n > N:
2
"
Do xnk ! a; ta có k0 > N : jxnk aj < ; 8k > k0 :
2
Đặc biệt lấy một số tự nhiên k1 = k0 + 1 > k0 ; ta có
"
xnk1 a < ;
2
nk1 k1 > N:

Với mọi m > N , ta suy ra


" "
jxm aj xm xnk1 + xnk1 a < + = ":
2 2
Vậy xn ! a:
Giải Ví dụ 6 giải xong.
Ví dụ 7: (Xem như Bài tập). Cho hai dãy số thực fan g và fbn g sao cho [an ; bn ] [am ; bm ]; 8m; n 2 N;
n m.
(i) Đặt amax = supan ; bmin = inf bm . Chứng minh rằng
n2N m2N
\
[amax ; bmin ] [ak ; bk ]:
k2N

(ii) Nếu thêm điều kiện bn an ! 0; hãy chứng minh rằng amax = bmin :
Giải Ví dụ 7.
Chứng minh (i). Viết lại điều kiện

am an bn bm ; 8m; n 2 N; m n:

Lấy m = 1
a1 an bn b1 ; 8n 2 N:
Suy ra:
fan g là dãy (tăng) không giảm và bị chận trên, do đó an ! amax supan ;
n2N
fbn g là dãy (giảm) không tăng và bị chận dưới, do đó bn ! bmin inf bn .
n2N
Cố định m 2 N; do am an bn bm ; 8n m. Cho n ! 1; ta thu được

am amax bmin bm ; 8m 2 N:

Do đó
[amax ; bmin ] [am ; bm ]; 8m 2 N:
Vậy \
[amax ; bmin ] [am ; bm ]:
m2N

Chứng minh (ii). Từ các bất đẳng thức am amax bmin bm ; 8m 2 N; ta suy ra

0 bmin amax bm am ; 8m 2 N:
Chương 3. DÃY SỐ THỰC 30

Cho m ! 1; ta thu được bmin amax = 0; hay supan = inf bm :


n2N m2N
Ví dụ 8: (Xem như Bài tập). Cho ba dãy số thực fan g; fbn g và fxn g sao cho [an ; bn ] [am ; bm ];
8m; n 2 N; m n.
(i) [an ; bn ] [am ; bm ]; 8m; n 2 N; m n;
(ii) bn an ! 0;
(iii) an xn bn ; 8n 2 N:
Chứng minh fxn g là một dãy hội tụ.
Giải Ví dụ 8. Theo Ví dụ 7, thì (i), (ii) dẫn đến an ! amax ; bn ! bmin và amax = bmin :
Từ các điều kiện (iii), dẫn đến xn ! amax = bmin :
Định lý 8 (Bolzano-Weierstrass). Cho a; b 2 R; a < b và fxn g một dãy số thực sao cho a xn b;
8n 2 N: Khi đó có một dãy con của dãy fxn g hội tụ về x 2 [a; b].
Chứng minh Định lý 8.
a+b
Chia đoạn [a; b] thành hai đoạn bởi trung điểm c = : Đặt
2
J1 = fn 2 N : xn 2 [a; c]g;
J1+ = fn 2 N : xn 2 [c; b]g;
ta thu được J1 [ J1+ = N:
Do N vô hạn phần tử nên một trong hai tập J1 ; J1+ phải vô hạn phần tử, chẳng hạn như J1+ vô hạn
phần tử. Chọn n1 2 J1+ .
a1 + b1
Ta đặt lại [a1 ; b1 ] = [c; b]; và chia đoạn [a1 ; b1 ] thành hai đoạn bởi trung điểm c1 = ; như vậy
2
J1 J1+ = fn 2 N : xn 2 [a1 ; b1 ]g; xn1 2 [a1 ; b1 ];
J2 = fn 2 J1 : xn 2 [a1 ; c1 ]g;
J2+ = fn 2 J1 : xn 2 [c1 ; b1 ]g;
ta thu được
J2 [ J2+ = J1 ;
b a
b1 a1 = :
2
Do J1 vô hạn phần tử nên một trong hai tập J2 ; J2+ phải vô hạn phần tử, chẳng hạn như J2+ vô hạn
phần tử. Chọn n2 2 J2+ ; n2 > n1 .
a2 + b2
Ta đặt lại [a2 ; b2 ] = [c1 ; b1 ]; và chia đoạn [a2 ; b2 ] thành hai đoạn bởi trung điểm c2 = ; như vậy
2
J2 J2+ = fn 2 J1 : xn 2 [a2 ; b2 ]g; xn2 2 [a2 ; b2 ];
J3 = fn 2 J2 : xn 2 [a2 ; c2 ]g;
J3+ = fn 2 J2 : xn 2 [c2 ; b2 ]g;
ta thu được
J3 [ J3+ = J2 ;
b1 a1 b a
b2 a2 = = :
2 22
Do J2 vô hạn phần tử nên một trong hai tập J3 ; J3+ phải vô hạn phần tử, chẳng hạn như J3+ vô hạn
phần tử. Chọn n3 2 J3+ ; n3 > n2 .
a3 + b3
Ta đặt lại [a3 ; b3 ] = [c2 ; b2 ]; và chia đoạn [a3 ; b3 ] thành hai đoạn bởi trung điểm c3 = ; như vậy
2
J3 J3+ = fn 2 J2 : xn 2 [a3 ; b3 ]g; xn3 2 [a3 ; b3 ];
J4 = fn 2 J3 : xn 2 [a3 ; c3 ]g;
J4+ = fn 2 J3 : xn 2 [c3 ; b3 ]g;
Chương 3. DÃY SỐ THỰC 31

ta thu được

J4 [ J4+ = J3 ;
b2 a2 b a
b3 a3 = = ;
2 23
Do J3 vô hạn phần tử nên một trong hai tập J4 ; J4+ phải vô hạn phần tử, chẳng hạn như J4+ vô hạn
phần tử. Chọn n4 2 J4+ ; n4 > n3 .
a4 + b43
Ta đặt lại [a4 ; b4 ] = [c3 ; b3 ]; và chia đoạn [a3 ; b3 ] thành hai đoạn bởi trung điểm c4 = ; như
2
vậy

J4 J4+ = fn 2 J3 : xn 2 [a4 ; b4 ]g; xn4 2 [a4 ; b4 ];


J5 = fn 2 J4 : xn 2 [a4 ; c4 ]g;
J5+ = fn 2 J4 : xn 2 [c4 ; b4 ]g;

ta thu được

J5 [ J5+ = J4 ;
b3 a3 b a
b4 a4 = = :
2 24
Tiếp tục quá trình trên ta chọn được tập con vô hạn Jk fn 2 Jk 1 : xn 2 [ak ; bk ]g; của N; gồm
những n sao cho xn 2 [ak ; bk ]; với

Jk Jk 1; [ak ; bk ] [ak 1 ; bk 1 ];
bk 1 ak 1 b a
bk ak = = = k ; 8k 2 N:
2 2
Chọn nk 2 Jk ; ta có

xnk 2 [ak ; bk ] [ak ; bk ] [ak 1 ; bk 1 ];


b a
bk ak = ! 0; khi k ! 1:
2k
Theo Ví dụ 8, ta thu được dãy fxnk g hội tụ.
Định lý 8 được chứng minh xong.

3.4 Dãy số tiến ra vô cùng


Định nghĩa 5. Ta nói dãy số thực fxn g tiến ra +1 nếu

8A 2 R; 9N 2 N : 8n 2 N; n > N =) xn > A:

Khi dãy fxn g tiến ra +1; ta có thể nói dãy fxn g có giới hạn +1 và ta có thể viết một theo các cách
sau

lim xn = +1;
n!1
hay lim xn = +1;
hay xn ! +1 khi n ! 1;
hay xn ! +1:

Định nghĩa tương tự cho giới hạn 1 cho dãy fxn g như sau
Định nghĩa 6. Ta nói dãy số thực fxn g tiến ra 1 nếu

8A 2 R; 9N 2 N : 8n 2 N; n > N =) xn < A:
Chương 3. DÃY SỐ THỰC 32

Khi dãy fxn g tiến ra 1; ta có thể nói dãy fxn g có giới hạn 1 và ta có thể viết một theo các cách
sau

lim xn = 1;
n!1
hay lim xn = 1;
hay xn ! 1 khi n ! 1;
hay xn ! 1:

Chú thích.
(i) Chú ý rằng dãy fxn g với hai trường hợp xn ! +1 và xn ! 1 (không có giới hạn hữu hạn)
được xếp vào loại dãy phân kỳ (không hội tụ).
(ii) Trong trường hợp dãy fxn g tăng và không bị chận trên, khi đó ta có xn ! +1:
Thật vậy, do dãy fxn g không bị chận trên, ta có

8M > 0; 9n(M ) 2 N : xn(M ) > M:

Do dãy fxn g tăng, nên, 8m 2 N; m > n(M ) =) xm > xn(M ) > M:


Vậy, xn ! +1:
(iii) Với chú ý là nếu dãy fxn g giảm và không bị chận dưới, ta có xn ! 1:
(iv) Các phép tính giới hạn của dãy cũng được mở rộng cho các phép tính với giới hạn vô cực như
sau.

(+1) (+1) = +1; ( 1) ( 1) = +1; ( 1) (+1) = 1; (+1) ( 1) = 1;


a (+1) = +1; a ( 1) = 1; ( a) ( 1) = +1; ( a) (+1) = 1; a > 0;
a
= 0; a 2 R:
1

Ngoài ra cũng có các dạng vô định (không xác định)


0 1
; ; 0 ( 1) ; ( 1) 0; 11 ; 00 :
0 1

Ví dụ 9: (Xem như Bài tập). Cho dãy số thực fxn g không bị chận trên. Chứng minh rằng tồn tại
một dãy con fxnk g của fxn g sao cho xnk ! +1:
Ví dụ 10: (Xem như Bài tập). Cho dãy số thực fxn g không bị chận dưới. Chứng minh rằng tồn tại
một dãy con fxnk g của fxn g sao cho xnk ! 1:
Giải Ví dụ 9.

fxn g bị chận trên () 9k 2 R : xn k; 8n 2 N;


fxn g không bị chận trên () 8k 2 N; 9nk 2 N : xnk > k:

Từ đây ta thấy xnk > k ! +1; do đó xnk ! +1: Nhưng ở đây cần đòi hỏi fxnk g phải là dãy con
của fxn g; tức là nk < nk+1 ; 8k 2 N:
Do fxn g không bị chận trên, nên với k = 1; 9n1 2 N : xn1 > 1:
Do tập A1 = fxn : n > n1 g không bị chận trên, nên với k = 2; 9n2 > n1 : xn2 > 2:
Tiếp tục quá trình trên, tập Ak = fxn : n > nk g không bị chận trên,
nên với k + 1 2 N; 9nk+1 > nk : xnk+1 > k + 1:

3.5 Giới hạn trên (limsup) và giới hạn dưới (liminf)


Định nghĩa giới hạn trên (limsup) và giới hạn dưới (liminf).
Cho một dãy số thực fxn g.
Chương 3. DÃY SỐ THỰC 33

Đặt An = fxk : k ng; ta có dãy fAn g giảm dần các tập con khác trống của N:

Am An A1 ; 8m; n 2 N; m n:

Định nghĩa limsup.


(i) Nếu A1 không bị chận trên (i.e. fxn g không bị chận trên), ta đặt: lim supxn = +1:
n!1
(ii) Nếu A1 bị chận trên (i.e. fxn g bị chận trên), ta đặt yn = sup An ; ta có fyn g là dãy giảm:

ym yn y1 ; 8m; n 2 N; m n:

(ii1) Nếu fyn g không bị chặn dưới, khi đó yn ! 1; ta đặt: lim supxn = 1 = lim yn :
n!1 n!1
(ii2) Nếu fyn g bị chặn dưới, khi đó fyn g có giới hạn là lim yn = inf yn ; ta đặt
n!1 n 1

lim supxn = lim yn :


n!1 n!1

Chú thích 1. Trường hợp (ii2) này, fyn g giảm và bị chặn dưới, ta có yn ! inf yn khi n ! 1:
n 1
Do đó, ta viết lại theo một cách như sau
!
lim supxn = lim yn = inf yn = inf supxk :
n!1 n!1 n 1 n 1 k n

Chú thích 2. Cũng còn gọi tên là giới hạn trên của dãy fxn g, và người ta có thể dùng thêm một ký
hiệu khác lim xn thay cho lim supxn :
n!1 n!1
Định nghĩa liminf.
(i) Nếu A1 không bị chận dưới (i.e. fxn g không bị chận dưới), ta đặt: lim inf xn = 1:
n!1
(ii) Nếu A1 bị chận dưới (i.e. fxn g bị chận dưới), ta đặt: zn = inf An , ta có fzn g là dãy tăng:

z1 zn zm ; 8m; n 2 N; m n:

(ii1) Nếu fzn g không bị chặn trên, khi đó zn ! +1; ta đặt: lim inf xn = +1 = lim zn :
n!1 n!1
(ii2) Nếu fzn g bị chặn trên, khi đó fzn g có giới hạn là lim zn = supzn ; ta đặt
n!1 n 1

lim inf xn = lim zn :


n!1 n!1

Chú thích 3. Trường hợp (ii2) này, fzn g tăng và bị chặn trên, ta có zn ! supzn khi n ! 1:
n 1
Do đó, ta viết lại theo một cách như sau

lim inf xn = lim zn = supzn = sup inf xk :


n!1 n!1 n 1 n 1 k n

Chú thích 4a. Cũng còn gọi tên là giới hạn dưới của dãy fxn g, và người ta có thể dùng thêm một
ký hiệu khác lim xn thay cho lim inf xn :
n!1 n!1
Chương 3. DÃY SỐ THỰC 34

Tóm tắt Định nghĩa.


(
+1; nếu fxn g không bị chận trên,
yn = supxk ; nếu fxn g bị chận trên,
k n
8
>
> +1; nếu fxn g không bị chận trên,
>
>
>
> 1; nếu fxn g bị chận trên, fyn g không bị chận dưới,
<
lim supxn = lim y n = inf y n
n!1 n 1 !
n!1 >
>
>
> nếu fxn g bị chận trên, fyn g bị chận dưới,
>
> = inf supxk ;
: n 1 k n
(
1; nếu fxn g không bị chận dưới,
zn = inf xk ; nếu fxn g bị chận dưới,
k n
8
> 1; nếu fxn g không bị chận dưới,
>
>
>
< +1;
> nếu fxn g bị chận dưới, fzn g không bị chận trên,
lim inf xn = lim z n = sup z n
n!1
n!1 >
> n 1
>
> nếu fxn g bị chận dưới, fzn g bị chận trên.
>
: = sup inf xk ;
n 1 k n

Chú thích 4b.


(i) Nếu fxn g không bị chận trên, ta định nghĩa lim supxn = +1:
n!1
(ii) Nếu fxn g không bị chận dưới, ta định nghĩa lim inf xn = 1:
n!1
Chú thích 4c. Nếu ta đặt

supxk = +1; nếu fxn g không bị chận trên,


k n

inf xk = 1; nếu fxn g không bị chận dưới,


k n

khi đó không chia các trường hợp chúng ta có thể định nghĩa lim supxn bằng công thức
n!1
!
lim supxn = inf supxk ;
n!1 n 1 k n

và lim inf xn bằng công thức


n!1

lim inf xn = sup inf xk :


n!1 n 1 k n

Ví dụ 11: Cho xn = ( 1)n ; với mọi n 2 N:


Giải Ví dụ 11. Đặt
An = fxk : k ng = f( 1)k : k ng = f 1; 1g:
(i) Tính lim inf xn . Ta có:
A1 bị chặn dưới, ta đặt
zn = inf An = 1:
Do fzn g bị chặn trên, ta có
lim inf xn = lim zn = 1:
n!1 n!1

Ta thấy rằng dãy fxn g là phân kỳ nhưng vẫn có lim inf xn = 1:


n!1
(ii) Tính lim sup xn . Ta có A1 bị chặn trên

yn = sup An = 1:
Chương 3. DÃY SỐ THỰC 35

Do fyn g bị chặn dưới, ta có


lim supxn = lim yn = 1:
n!1 n!1

Trong trường hợp này, ta có

lim supxn = 1 6= 1 = lim inf xn :


n!1 n!1

Ví dụ 12: Cho xn = ( 1)n n; với mọi n 2 N:


Giải Ví dụ 12. Đặt
An = fxk : k ng = f( 1)k k : k ng;
ta có
A1 = fxk : k 1g = f( 1)k k : k 1g f 2k 1:k 1g:
(i) Tính lim inf xn . Ta có A1 không bị chặn dưới, vậy lim inf xn = 1:
n!1
(ii) Tính lim sup xn . Ta có

A1 = fxk : k 1g = f( 1)k k : k 1g f2k : k ng:

Do đó A1 không bị chặn trên, vậy lim supxn = +1:


n!1
Ví dụ 13: Cho xn = n; với mọi n 2 N:
Giải Ví dụ 13. Đặt

An = fxk : k ng = f k : k ng
= fk 2 Z : k ng fk : k 1g = A1 :

(i) Tính lim sup xn . Ta có A1 bị chặn trên, yn = sup An = n:


Do fyn g không bị chặn dưới, ta có lim supxn = 1:
n!1
(ii) Tính lim inf xn . Ta có A1 không bị chặn dưới, vậy lim inf xn = 1:
n!1
Ví dụ 14: Cho xn = n; với mọi n 2 N:
Giải Ví dụ 14. Đặt

An = fxk : k ng = fk : k ng
A1 = fxk : k 1g = N:

(i) Tính lim sup xn . Ta có, A1 không bị chặn trên, vậy lim supxn = +1:
n!1
(ii) Tính lim inf xn . Ta có, A1 bị chận dưới, ta đặt zn = inf An = n:
Do fzn g không bị chặn trên, ta có lim inf xn = +1:
n!1
Ví dụ 15: (Xem như Bài tập). Cho một dãy số thực fxn g. Giả sử lim supxn và lim inf xn đều là các
n!1 n!1
số thực. Chứng minh lim inf xn lim supxn :
n!1 n!1
Giải Ví dụ 15.

An = fxk : k ng;
Am An A1 ; 8m; n 2 N; m n:

yn = sup An :
lim supxn = lim yn = lim (sup An )
n!1 n!1 n!1

zn = inf An :
lim inf xn = lim zn = lim (inf An )
n!1 n!1 n!1

zn = inf An xn sup An = yn ; 8n 2 N:
Chương 3. DÃY SỐ THỰC 36

Do zn ! lim inf xn và yn ! lim supxn ; ta có


n!1 n!1

lim inf xn = lim zn lim yn = lim sup xn :


n!1 n!1 n!1 n!1

Ví dụ 16: (Xem như Bài tập). Cho một dãy số thực fxn g. Chứng minh rằng

lim sup xn = lim inf ( xn ):


n!1 n!1

Giải Ví dụ 16. Ta có
!
lim sup xn = inf sup xk = inf inf ( xk )
n!1 n 1 k n n 1 k n

= sup inf ( xk ) = lim inf ( xn ):


n 1 k n n!1

Ví dụ 17: (Xem như Bài tập). Cho một dãy số thực fxn g. Giả sử lim sup xn = lim inf xn = a 2 R:
n!1 n!1
Chứng minh rằng xn ! a:
Ví dụ 18: (Xem như Bài tập). Cho một dãy số thực fxn g. Giả sử xn ! a: Chứng minh rằng lim sup
n!1
xn = lim inf xn = a:
n!1
Giải Ví dụ 17. Ta có
zn = inf xk xn sup xk = yn ; 8n 2 N:
k n k n

Do zn ! lim inf xn = a và yn ! lim sup xn = a; ta có xn ! a:


n!1 n!1
Giải Ví dụ 18.
(i) Chứng minh lim supxn = a: Cho " > 0; do xn ! a ta có N 2 N :
n!1

8n > N =) jxn aj < "=2:

Do yn = supxk ; 9kn (") n : yn "=2 < xkn (") yn : Điều này dẫn đến
k n

xkn (") yn < "=2:

8n > N; ta có kn (") n > N; do đó

xkn (") a < "=2:

Vậy
jyn aj yn xkn (") xkn (") a < "=2 + "=2 = ":
Suy ra yn ! a; tức là lim supxn = lim yn = a:
n!1 n!1
(ii) Chứng minh lim inf xn = a:
n!1
Do zn = inf xk ; 9qn (") n : zn xqn (") < zn + "=2: Điều này dẫn đến
k n

xqn (") zn < "=2:

8n > N; ta có qn (") n > N; do đó

xqn (") a < "=2:

Vậy
jzn aj zn xqn (") xqn (") a < "=2 + "=2 = ":
Chương 3. DÃY SỐ THỰC 37

Suy ra zn ! a; tức là lim inf xn = lim zn = a:


n!1 n!1
Ví dụ 19: (Xem như Bài tập). Cho một dãy số thực fxn g bị chận. Đặt A = fa 2 R : a là giới hạn
của một dãy con của fxn g g. Chứng minh rằng

lim supxn = sup A 2 A;


n!1
lim inf xn = inf A 2 A:
n!1

Giải Ví dụ 19. Cho a 2 A = fa 2 R : a là giới hạn của một dãy con của fxn g g. Khi đó, ta có một
dãy con fxnj g fxn g sao cho xnj ! a:
Ta sẽ chứng minh rằng
lim inf xn a lim supxn :
n!1 n!1

Từ bất đẳng thức


znj = inf xk xnj sup xk = ynj ;
k nj k nj

kết hợp với sự hội tụ zn = inf xk ! lim inf xn và yn = supxk ! lim supxn ; ta suy ra znj = inf xk !
k n n!1 k n n!1 k nj
lim inf xn và ynj = sup xk ! lim supxn ; do đó ta có
n!1 k nj n!1

lim inf xn a lim supxn :


n!1 n!1

Ta sẽ chứng minh rằng lim supxn = M 2 A; và lim inf xn = m 2 A:


n!1 n!1
(i) Chứng minh lim supxn = M 2 A:
n!1 !
Ta có M = lim supxn = inf yn = inf supxk :
n!1 n 1 n 1 k n
1 1
Với mọi j 2 N; " = ; ta có nj 2 N : M ynj < M + :
j j
1
Với ynj = sup xk ; ta có kj nj sao cho ynj < xkj ynj :
k nj j
Mà xkj sup xk = ykj ynj ; do đó
k kj

1 1 1
M ynj < xkj ynj < M +
j j j
1
Ta suy ra xkj M < ! 0; vậy xkj ! M: nghĩa là M 2 A và M = max A.
j
(ii) Chứng minh lim inf xn = m 2 A:
n!1

Ta có m = lim inf xn = supzn = sup inf xk :


n!1 n 1 n 1 k n
1 1
Với mọi j 2 N; " = ; ta có nj 2 N : m < znj m:
j j
1
Với znj = inf xk ; ta có kj nj sao cho znj xkj < znj + :
k nj j
Mà xkj inf xk = zkj znj ; do đó
k kj

1 1 1 1
m < znj xkj < znj + zk j + m+
j j j j
1
Ta suy ra xkj m <! 0; vậy xkj ! m: nghĩa là m 2 A và m = min A.
j
Ví dụ 20: (Xem như Bài tập). Cho một dãy số thực fxn g = f1; 2; 3; 1; 2; 3; 1; 2; 3; g = fx1 ; x2 ; x3 ; x4 ; g:
Chương 3. DÃY SỐ THỰC 38

(i) Chứng minh rằng dãy fxn g bị chận.


(ii) Không cần chỉ ra dãy con cụ thể hãy chứng tỏ rằng tồn tại một dãy con hội tụ của fxn g.
(iii) Chứng minh rằng có một dãy con của fxn g hội tụ về lim supxn :
n!1
(iv) Chứng minh rằng có một dãy con của fxn g hội tụ về lim inf xn :
n!1
Giải Ví dụ 20. Để hình dung ra dãy fxn g trên đây ta viết lại
fxn g = f1; 2; 3; 1; 2; 3; 1; 2; 3; g = f
x1 = 1; x2 = 2; x3 = 3; x4 = 1; x5 = 2; x6 = 3; x7 = 1; x8 = 2; x9 = 3; x10 = 1; x11 = 2; x12 = 3;
x13 = 1; x14 = 2; x15 = 3;
hay viết lại
x3n 2 = 1; n 2 N; x3n 1 = 2; n 2 N; x3n = 3; n 2 N;
n 1 n 1
r=n 3 = n 3 phần nguyên của :
3 3
n 1 0
n=1:r=n 3 =1 3 = 1:
3 3
n 1 1
n=2:r=n 3 =2 3 = 2:
3 3
n 1 2
n=3:r=n 3 =3 3 = 3:
3 3
n 1 3
n=4:r=n 3 =4 3 = 1:
3 3
n 1 4
n=5:r=n 3 =5 3 = 2:
3 3
n 1 5
n=6:r=n 3 =6 3 = 3:
3 3
n 1 6
n=7:r=n 3 =7 3 = 1:
3 3
n 1 7
n=8:r=n 3 =8 3 = 2:
3 3
n 1 8
n=9:r=n 3 =9 3 = 3:
3 3
n 1 3k 3
n = 3k 2 : r = n 3 = 3k 2 3 = 1:
3 3
n 1 3k 2
n = 3k 1 : r = n 3 = 3k 1 3 = 2:
3 3
n 1 3k 1
n = 3k : r = n 3 = 3k 3 = 3:
3 3
n 1
Dãy trên viết lại thành một công thức như sau xn = n 3 ; n 2 N:
3
(i) Chứng minh rằng dãy fxn g bị chận.
Dãy trên đây chỉ lấy 3 giá trị 1; 2; 3: xn 2 f1; 2; 3g; 8n 2 N: Do đó 1 xn 3; 8n 2 N: Như vậy dãy
fxn g bị chận dưới bởi 1 và bị chận trên bởi 3; do đó dãy fxn g bị chận.
(ii) Do định lý Bolzano-Weierstrass, tồn tại một dãy con hội tụ của fxn g.
(iii) Chỉ ra một dãy con của fxn g hội tụ về lim supxn :
n!1
(iv) Chỉ ra một dãy con của fxn g hội tụ về lim inf xn :
n!1
Ta lấy hai dãy con xnk = x3k = 3; xmk = x3k 2 = 1; 8k 2 N: Đây là hai dãy hằng do đó lần lượt hội
tụ về 3 và 1. Ta kiểm tra lại lim supxn = 3 và lim inf xn = 1:
n!1 n!1
Ta có 1 xn 3; 8n 2 N; do đó 1 lim inf xn < lim supxn 3; 8n 2 N:
n!1 n!1
Đặt A = fa 2 R : a là giới hạn của một dãy con của fxn g g. Khi đó A [1; 3]: Mà 3 2 A;
max A 3 max A; vậy lim supxn = max A = 3:
n!1
Tương tự, 1 2 A; min A 1 min A; vậy lim inf xn = min A = 1:
n!1
Chương 3. DÃY SỐ THỰC 39

!
Chú thích: Có thể tính lim supxn qua công thức lim supxn = inf supxk :
n!1 n!1 n 1 k n
!
Thật vậy, dễ thấy supxk = maxf1; 2; 3g = 3; 8n 2 N: Do đó lim supxn = inf supxk = 3:
k n n!1 n 1 k n

Tương tự, ta có inf xk = minf1; 2; 3g = 1; 8n 2 N: Do đó lim inf xn = sup inf xk = 1:


k n n!1 n 1 k n

3.6 Bài tập bổ sung


1. Cho dãy số thực fxn g hội tụ về a:
p p
(i) Giả sử xn 0; 8n 2 N: Chứng minh rằng a 0 và xn ! a:
p p
(ii) Chứng minh rằng 3 xn ! 3 a:
(iii) Cho 2 (0; 1): Chứng minh rằng jxn j ! jaj :
x1 + x2 + + xn
(iv) Chứng minh rằng ! a:
n
x1 + x2 x1 + x2 + + xn
x1 + + +
(v) Chứng minh rằng 2 n ! a:
n
(vi) Giả sử xn > 0; 8n 2 N và a > 0: Chứng minh rằng (x1 x2 xn )1=n ! a:
Hướng dẫn: p
p p
(i) Dùng bất đẳng thức x y jx yj; 8x; y 0:
1 1
(ii) Dùng bất đẳng thức jxj x jyj y 21 jx yj ; 8x; y 0; 8 2 (0; 1):
(iii) Dùng bất đẳng thức jjxj jyj j jx yj ; 8x;X
y 2 R; 8 2 (0; 1):
n
x1 + x2 + + xn jxi aj
i=1
(iv) Dùng bất đẳng thức a :
n n
(v) Áp dụng kết quả (iv). Xn
jln xi ln aj
(vi) Dùng bất đẳng thức ln (x1 x2 xn )1=n ln a i=1
:
n
2. Cho dãy số thực fxn g xác định bởi công thức qui nạp
(
x1 = 1;
1
xn = xn 1 1; n = 2; 3;
2
Chứng minh rằng fxn g hội tụ. Tính lim xn :
n!1
3. Cho dãy số thực fxn g xác định bởi công thức qui nạp
x1 = 1;
p
xn = xn 1 + 2; n = 2; 3;
Chứng minh rằng fxn g hội tụ. Tính lim xn :
n!1
4. Cho 6= A R và bị chặn trên. Chứng minh rằng có một dãy fxn g A sao cho xn ! sup A.
5. Cho 6= A R và bị chặn dưới. Chứng minh rằng có một dãy fxn g A sao cho xn ! inf A.
6. Cho dãy số thực fxn g tăng (hoặc không giảm) và không bị chận trên. Chứng minh rằng xn ! +1:
7. Cho dãy số thực fxn g giảm (hoặc không tăng) và không bị chận dưới. Chứng minh rằng xn ! 1:
8. Cho dãy số thực fxn g không bị chận trên. Chứng minh rằng có một dãy con fxnk g của fxn g sao
cho xnk ! +1:
9. Cho dãy số thực fxn g không bị chận dưới. Chứng minh rằng có một dãy con fxnk g của fxn g sao
cho xnk ! 1:
1 1
10. Cho dãy số thực fxn g xác định bởi công thức xn = 1 + + + ; n 2 N: Chứng minh rằng
2 n
fxn g phân kỳ và xn ! +1:
11. Cho x 2 R: Chứng minh rằng tồn tại hai dãy số fxn g và fyn g sao cho:
(i) fxn g Q; 8n 2 N : xn ! x;
(ii) fyn g (R n Q); 8n 2 N : yn ! x:
Chương 3. DÃY SỐ THỰC 40

12. Cho dãy số thực fxn g: Giả sử x2n ! a và x2n 1 ! a: Chứng minh rằng fxn g hội tụ về a:
13. Cho dãy số thực fxn g xác định bởi công thức qui nạp
(
x1 = 1;
1 + xn 1
xn = ; n = 2; 3;
2
Chứng minh rằng fxn g là dãy Cauchy. Tính lim xn :
n!1
14. Cho a; p; q 2 R; jqj < 1 và dãy số thực fxn g xác định bởi công thức qui nạp
x1 = a;
xn = q sin xn 1 + p; n = 2; 3; ;
Chứng minh rằng fxn g hội tụ.
15. Cho dãy số thực fxn g thỏa điều kiện 2 cos2 xn + sin xn 1 + 4 = 0; n = 2; 3; : Chứng minh rằng
dãy fxn g phân kỳ.
16. Cho f : [a; b] ! [a; b] thỏa điều kiện, tồn tại hằng số 2 [0; 1) sao cho jf (x) f (y)j jx yj ;
8x; y 2 [a; b]:
Cho x0 2 [a; b]; xét dãy số thực fxn g xác định bởi công thức qui nạp xn = f (xn 1 ); n = 1; 2; :
Chứng minh rằng:
(i) fxn g hội tụ về một giới hạn x 2 [a; b]:
(ii) x là nghiệm duy nhất của phương trình x = f (x ):
b a n
(iii) jxn x j ; 8n 2 N:
1
17. (Bài này sẽ làm trong chương hàm số liên tục). Cho f : [a; b] ! [a; b] thỏa điều kiện jf (x) f (y)j <
jx yj ; 8x; y 2 [a; b]; x 6= y:
Chứng minh rằng tồn tại duy nhất x 2 [a; b] sao cho x = f (x ):
Chương 4

GIỚI HẠN HÀM SỐ

4.1 Điểm tụ
Định nghĩa 1. Cho 6= D R và a 2 R. Ta nói a là một điểm tụ của D nếu

(a "; a + ") \ (Dnfag) 6= ; 8" > 0:

Điều được diễn đạt như sau

((a "; a + ")nfag) \ D 6= ; 8" > 0:

hay
8" > 0; 9x 2 D và 0 < jx aj < ";
i.e., mọi khoảng tâm a bán kính " chứa ít nhất một điểm của D khác với a.
Tập hợp tất cả các điểm tụ của D được ký hiệu là D :
Chú ý rằng

(a "; a + ") \ (Dnfag) = ((a "; a + ")nfag) \ D


= [(a "; a) [ (a; a + ")] \ D:

Chú ý rằng:
- Điểm tụ của D không nhất thiết thuộc D (Xem ví dụ 3 dưới đây).
- Một điểm thuộc D không nhất thiết là điểm tụ của D (Xem ví dụ 5 dưới đây).
Ví dụ 1: (Xem như Bài tập). Chứng minh hai mệnh đề sau đây tương đương với nhau a 2 D ()
a 2 (Dnfag) .
Giải Ví dụ 1. Đặt D1 = Dnfag: Ta chứng minh rằng a 2 D () a 2 D1 :
(i) Giả sử a 2 D : Ta chứng minh rằng a 2 D1 ; i.e.,

(a "; a + ") \ (D1 nfag) 6= ; 8" > 0 ( )

Mà D1 nfag = Dnfag; do đó (*) đúng.


(ii) Giả sử a 2 D1 : Ta chứng minh rằng a 2 D ; i.e.,

(a "; a + ") \ (Dnfag) 6= ; 8" > 0 ( )

Tương tự, do D1 nfag = Dnfag; do đó (**) đúng.


Ví dụ 2: (Xem như Bài tập). Chứng minh rằng a 2 D () Tồn tại một dãy fxn g Dnfag;
8n 2 N : xn ! a:
Giải Ví dụ 2.
1
(i) Chứng minh =): Giả sử a 2 D : Lấy " = > 0; ta có
n
1 1
(a ; a + ) \ (Dnfag) 6= ; 8n 2 N:
n n

41
Chương 4. GIỚI HẠN HÀM SỐ 42

1 1 1
Vậy tồn tại xn 2 (a ; a + ) \ (Dnfag) ; 8n 2 N; điều này dẫn tới xn 2 Dnfag; jxn aj < ;
n n n
8n 2 N; tức là fxn g Dnfag; 8n 2 N : xn ! a:
(ii) Chứng minh (=: Đảo lại, giả sử có một dãy fxn g Dnfag; 8n 2 N : xn ! a: Ta chứng minh

(a "; a + ") \ (Dnfag) 6= ; 8" > 0 ( )

Cho " > 0; do xn ! a; ta có n0 2 N : jxn0 aj < ": Vậy xn0 2 (a "; a + ") \ (Dnfag) ; do đó
(a "; a + ") \ (Dnfag) 6= :
Ví dụ 3: (Xem như Bài tập). Cho D = (0; 1) và a = 0. Chứng minh a = 0 là một điểm tụ của D:
Ví dụ 4: (Xem như Bài tập). Cho D = [0; 1] và a = 0. Chứng minh a = 0 là một điểm tụ của D:
Ví dụ 5: (Xem như Bài tập). Cho D = [0; 1] [ f2g và a = 2. Chứng minh a = 2 không là một điểm
tụ của D:

4.2 Giới hạn hàm số tại điểm tụ


Định nghĩa 2. Cho 6= D R và a 2 D . Cho f : D ! R: Ta nói f có giới hạn tại a nếu:
Tồn tại L 2 R sao cho

8" > 0; 9 > 0 : 8x 2 D; 0 < jx aj < =) jf (x) Lj < ":

Điều được diễn đạt như sau

9L 2 R : 8" > 0; 9 > 0 : 8x 2 (a ; a + ) \ (Dnfag) =) jf (x) Lj < ":

Nếu f có giới hạn tại a thì số thực L tồn tại ở trên là duy nhất.
1
Thật vậy, giả sử có hai số thực L; L0 thỏa mệnh đề trên. Nếu L 6= L0 ; ta chọn " = jL L0 j > 0, ta
2
có hai số 1 > 0; 2 >0:

8x 2 (a 1; a + 1) \ (Dnfag) =) jf (x) Lj < ";


8x 2 (a 2; a + 2) \ (Dnfag) =) f (x) L0 < "; :

Chọn = minf 1 ; 2 g; và x1 2 (a ; a + ) \ (Dnfag) ; ta có

L L0 jL f (x1 )j + f (x1 ) L0 < " + " = L L0 ;

tức là jL L0 j < jL L0 j : Điều này không thể xảy ra. Vậy L = L0 :


Khi f có giới hạn tại a, số thực L ở trên gọi là giới hạn tại a của hàm f và ký hiệu nó hoặc viết là

L = lim f (x);
x!a
hay f (x) ! L khi x ! a:

Định nghĩa 3 (Định nghĩa tương đương giới hạn hàm theo ngôn ngữ dãy).
Định lý 1 (Định nghĩa tương đương giới hạn hàm theo ngôn ngữ dãy). Cho 6= D R và a 2 D .
Cho f : D ! R: Khi đó:

Với mọi dãy fxn g Dnfag;


L = lim f (x) () =) f (xn ) ! L :
x!a và xn ! a

Chứng minh Định lý 1.


Chứng minh phần thuận =): Giả sử L = lim f (x) và cho dãy fxn g Dnfag; và xn ! a: Ta chứng
x!a
minh rằng f (xn ) ! L:
Chương 4. GIỚI HẠN HÀM SỐ 43

Cho " > 0; do L = lim f (x);


x!a

9 > 0 : 8x 2 (a ; a + ) \ (Dnfag) =) jf (x) Lj < ":

Do xn ! a;
9N 2 N : 8n 2 N; n > N =) jxn aj < :
Ta suy ra
xn 2 (a ; a + ) \ (Dnfag) ; 8n > N:
Điều này dẫn đến
jf (xn ) Lj < "; 8n > N:
Vậy f (xn ) ! L:
Chứng minh phần đảo (=: Giả sử " Với mọi dãy fxn g Dnfag và xn ! a =) f (xn ) ! L". Ta
chứng minh rằng L = lim f (x):
x!a
Nếu L 6= lim f (x); ta có "0 > 0 :
x!a

8 > 0; 9x 2 (a ; a + ) \ (Dnfag) và jf (x ) Lj "0 :


1 1 1
Với mỗi n 2 N; = > 0; 9xn 2 (a ; a + ) \ (Dnfag) và jf (xn ) Lj "0 ;
n n n
1
i.e., 9fxn g (Dnfag) ; jxn aj < ; và jf (xn ) Lj "0 :
n
Điều này dẫn đến, ta có một dãy fxn g (Dnfag) ; xn ! a và f (xn ) 9 L:
Điều này mâu thuẫn với giả thiết phần đảo. Vậy L = lim f (x):
x!a
Định lý 1 được chứng minh xong.
Định nghĩa 4. Định nghĩa giới hạn bên phải.
Cho 6= D R và a 2 R sao cho (a; a + ) \ D 6= ; 8 > 0: Cho f : D ! R: Ta nói f có giới
hạn bên phải tại a nếu:
Tồn tại L 2 R sao cho

8" > 0; 9 > 0 : 8x 2 D; 0 < x a< =) jf (x) Lj < ":

Điều được diễn đạt như sau

9L 2 R : 8" > 0; 9 > 0 : 8x 2 (a; a + ) \ D =) jf (x) Lj < ":

Nếu f có giới hạn bên phải tại a thì số thực L tồn tại ở trên là duy nhất (kiểm tra tương tự như
trên). Số thực L này gọi là giới hạn bên phải tại x0 của hàm f và ký hiệu nó hoặc viết là

L = lim f (x);
x!a+
hay L = lim f (x);
x!a+0
hay f (x) ! L khi x ! a+ :

Chú thích. Do (a ; a + ) \ [D \ (a; +1)] = (a; a + ) \ D; nên điều kiện (a; a + ) \ D 6= ; 8 > 0
tương đương với a là một điểm tụ của D \ (a; +1): Để chỉ a là một điểm tụ của D \ (a; +1); ta viết
a 2 D+ : Như vậy khi định nghĩa lim f (x) chúng ta đã qui ước rằng a là một điểm tụ của D \ (a; +1)
x!a+
(tức là a 2 D+ ) mà không cần phải nhắc đến điều kiện của a:
Định nghĩa 5. Định nghĩa giới hạn bên trái.
Định nghĩa tương tự với 6= D R và a 2 R sao cho (a ; a) \ D 6= ; 8 > 0 (tức là a là một
điểm tụ của D \ ( 1; a)). Ta nói f có giới hạn bên trái tại a nếu:
Tồn tại L 2 R sao cho

8" > 0; 9 > 0 : 8x 2 D; a < x < a =) jf (x) Lj < ":


Chương 4. GIỚI HẠN HÀM SỐ 44

Điều được diễn đạt như sau

9L 2 R : 8" > 0; 9 > 0 : 8x 2 (a ; a) \ D =) jf (x) Lj < ":

Nếu f có giới hạn bên trái tại a thì số thực L tồn tại ở trên là duy nhất và gọi là giới hạn bên trái
tại a của hàm f và ký hiệu nó hoặc viết là

L = lim f (x);
x!a
hay L = lim f (x);
x!a 0
hay f (x) ! L khi x ! a :

Chú thích. Nếu a là một điểm tụ của D \ ( 1; a); ta viết a 2 D : Cũng giống như chú thích ở
trên, khi định nghĩa lim f (x) chúng ta đã qui ước rằng a là một điểm tụ của D \ ( 1; a) mà không
x!a
cần phải nhắc đến điều kiện của a:
Tương tự như trên, các định lý sau cho Định nghĩa tương đương giới hạn bên phải, bên trái của hàm
theo ngôn ngữ dãy.
Định lý 2. Cho 6= D R và f : D ! R: Khi đó:
(i) Nếu a là một điểm tụ của D \ (a; +1) (tức là a 2 D+ ) ta có

8fxn g D; xn > a; 8n 2 N
L = lim f (x) () =) f (xn ) ! L ;
x!a+ và xn ! a

(ii) Nếu a là một điểm tụ của D \ ( 1; a) (tức là a 2 D ) ta có

8fxn g D; xn < a; 8n 2 N
L = lim f (x) () =) f (xn ) ! L :
x!a và xn ! a

Chứng minh (xem như bài tập).


Định lý 3. Cho 6= D R và a 2 D+ \ D . Cho f : D ! R: Khi đó:

lim f (x) tồn tại () Cả hai lim f (x); lim f (x) tồn tại và bằng nhau.
x!a x!a+ x!a

Khi đó
lim f (x) = lim f (x) = lim f (x).
x!a x!a+ x!a

Định lý 4. Cho 6= D R và a 2 D . Cho f : D ! R: Khi đó:

8" > 0; 9 > 0 :


lim f (x) tồn tại () =) f (x) f (x0 ) < "
x!a 8x; x0 2 (a ; a + ) \ (Dnfag)

Chứng minh Định lý 4.


(i) Chứng minh phần thuận =): Giả sử tồn tại L = lim f (x) tồn tại. Cho " > 0; ta có > 0 sao cho
x!a

"
8x 2 (a ; a + ) \ (Dnfag) =) jf (x) Lj < :
2
Do đó, 8x; x0 2 (a ; a + ) \ (Dnfag) ; ta có
" "
jf (x) Lj < và f (x0 ) L < :
2 2
Điều này dẫn tới
" "
f (x) f (x0 ) jf (x) Lj + L f (x0 ) < + = ":
2 2
Chương 4. GIỚI HẠN HÀM SỐ 45

(ii) Chứng minh phần đảo (=: Đảo lại, giả sử


8" > 0; 9 > 0 :
=) f (x) f (x0 ) < ":
8x; x0 2 (a ; a + ) \ (Dnfag)

Ta sẽ chứng minh rằng lim f (x) tồn tại.


x!a
Cho 8" > 0; 9 1 > 0 :
"
8x; x0 2 (a 1; a + 1) \ (Dnfag) =) f (x) f (x0 ) < : ( )
2
Cho fxn g Dnfag; và xn ! a: Ta sẽ chứng minh rằng dãy ff (xn )g hội tụ về L và lim f (x) = L.
x!a
Do fxn g Dnfag; và xn ! a; ta có N1 2 N :
8n 2 N; n > N1 =) xn 2 (a 1; a + 1) \ (Dnfag) :
Do đó 8m; n 2 N; m; n > N1 ; ta có
xm ; xn 2 (a 1; a + 1) \ (Dnfag) ;
mà điều này dẫn tới
"
8m; n 2 N; m; n > N1 =) jf (xm ) f (xn )j < < ": ( )
2
Suy ra ff (xn )g là dãy Cauchy, do đó ff (xn )g là dãy hội tụ về L:
Cố định m = m1 > N1 và cho n ! +1 trong (**), ta có
"
xm1 2 (a 1; a + 1) \ (Dnfag) và jf (xm1 ) Lj :
2
Khi đó, 8x 2 (a 1; a + 1) \ (Dnfag), ta có
" "
jf (x) Lj < jf (x) f (xm1 )j + jf (xm1 ) Lj < + = ":
2 2
Vậy lim f (x) tồn tại và lim f (x) = L: Định lý 4 được chứng minh.
x!a x!a
Tương tự ta cũng có
Định lý 5. Cho 6= D R và f : D ! R: Khi đó:
(i) Nếu a 2 D+ ta có
8" > 0; 9 > 0 :
lim f (x) tồn tại () =) f (x) f (x0 ) < " :
x!a+ 8x; x0 2 (a; a + ) \ D

(ii) Nếu a 2 D ta có
8" > 0; 9 > 0 :
lim f (x) tồn tại () =) f (x) f (x0 ) < " :
x!a 8x; x0 2 (a ; a) \ D

4.3 Giới hạn hàm số tại vô cùng


4.3.1 Giới hạn hàm số tại +1.
Định nghĩa giới hạn tại +1. Cho 6= D R sao cho tồn tại fxn g D; 8n 2 N; xn ! +1: Cho
f : D ! R: Ta nói f (x) có giới hạn tại +1; nếu: Tồn tại L 2 R sao cho
8" > 0; 9N 2 R : 8x 2 D; x > N =) jf (x) Lj < ":
Nếu f có giới hạn tại +1 thì số thực L tồn tại ở trên là duy nhất (Kiểm tra lại như bài tập). Số
thực L ở trên gọi là giới hạn tại +1 của hàm f và ký hiệu nó hoặc viết là
L = lim f (x);
x!+1
hay f (x) ! L khi x ! +1:
Chương 4. GIỚI HẠN HÀM SỐ 46

4.3.2 Giới hạn hàm số tại 1.


Định nghĩa giới hạn tại 1. Cho 6= D R sao cho tồn tại fxn g D; 8n 2 N; xn ! 1: Cho
f : D ! R:
Tương tự ta cũng nói f (x) có giới hạn tại 1; nếu: Tồn tại L 2 R sao cho

8" > 0; 9N 2 R : 8x 2 D; x < N =) jf (x) Lj < ":

Nếu f có giới hạn tại 1 thì số thực L tồn tại ở trên là duy nhất (Kiểm tra lại như bài tập). Số
thực L ở trên gọi là giới hạn tại 1 của hàm f và ký hiệu nó hoặc viết là

L = lim f (x);
x! 1
hay f (x) ! L khi x ! 1:

Tương tự ta cũng có định nghĩa giới hạn hàm tại vô cùng theo ngôn ngữ dãy.
Định lý 6. Với D như trên phù hợp cho định nghĩa giới hạn hàm tại vô cùng và cho f : D ! R; ta

Với mọi dãy fxn g D;
L = lim f (x) () =) f (xn ) ! L ;
x!+1 và xn ! +1
Với mọi dãy fxn g D;
L = lim f (x) () =) f (xn ) ! L :
x! 1 và xn ! 1

Chứng minh như bài tập.


Tương tự ta cũng có định lý về sự tồn tại giới hạn hàm tại vô cùng
Định lý 7. Với D như trên phù hợp cho định nghĩa giới hạn hàm tại vô cùng và cho f : D ! R; ta

8" > 0; 9N 2 R :
lim f (x) tồn tại () =) f (x) f (x0 ) < " ;
x!+1 8x; x0 2 R; x; x0 > N
8" > 0; 9N 2 R :
lim f (x) tồn tại () =) f (x) f (x0 ) < " :
x! 1 8x; x0 2 R; x; x0 < N

Chứng minh như bài tập.


Các phép tính về giới hạn hàm số.
Định lý 8. Cho 6= D R và a 2 D . Cho f; g : D ! R: Giả sử f và g có giới hạn tại a lần lượt
là L = lim f (x); và m = lim g(x): Khi đó
x!a x!a
(i) Nếu f (x) > 0; 8x 2 D thì L 0;
(ii) Nếu L > 0; thì 9 > 0 : f (x) > 0; 8x 2 (a ; a + ) \ (Dnfag) ;
(iii) Nếu f (x) < g(x); 8x 2 D thì L m;
(iv) Nếu L < m; thì 9 > 0 : f (x) < g(x); 8x 2 (a ; a + ) \ (Dnfag) ;
(v) lim (f (x) + g(x)) = L + m;
x!a
(vi) lim f (x)g(x) = Lm;
x!a
(vii) lim cf (x) = cL; 8c 2 R;
x!a
f (x) L
(viii) Nếu m 6= 0; thì lim = :
x!a g(x) m
Định lý 9. Cho 6= D R và a 2 D . Cho ba hàm f; g; h : D ! R sao cho:
(i) f (x) g(x) h(x); 8x 2 Dnfag;
(ii) lim f (x) = lim h(x) = L:
x!a x!a

Khi đó, tồn tại lim g(x) và lim g(x) = L:


x!a x!a

Các tính chất và các phép tính trên vẫn còn đúng cho các giới hạn bên phải, bên trái tại a:
Chương 4. GIỚI HẠN HÀM SỐ 47

Định nghĩa 6. Cho hàm số f : D ! R: Ta nói


(i) Hàm số f bị chận trên D nếu tập ff (x) : x 2 Dg bị chận trên,
i.e., 9M 2 R : f (x) M; 8x 2 D;
(ii) Hàm số f bị chận dưới nếu tập ff (x) : x 2 Dg bị chận dưới,
i.e., 9m 2 R : f (x) m; 8x 2 D;
(iii) Hàm số f bị chận nếu f bị chận trên và bị chận dưới,
i.e., 9m; M 2 R : m f (x) M; 8x 2 D;
hay tương đương với 9M 2 R : jf (x)j M; 8x 2 D;
Định nghĩa 7. Cho hàm số f : (a; b) ! R: Ta nói
(i) f là hàm tăng trên (a; b) nếu 8x; x0 2 (a; b); x < x0 =) f (x) < f (x0 );
(ii) f là hàm giảm trên (a; b) nếu 8x; x0 2 (a; b); x < x0 =) f (x) > f (x0 );
(iii) f là hàm không giảm trên (a; b) nếu 8x; x0 2 (a; b); x < x0 =) f (x) f (x0 );
(iv) f là hàm không tăng trên (a; b) nếu 8x; x0 2 (a; b); x < x0 =) f (x) f (x0 );
(v) Hàm f thoả 1 trong 4 tính chất (iv)-(vii) được gọi là hàm số đơn điệu.
Để phân biệt người ta còn gọi tên đi kèm chữ đơn điệu chẳng hạn như: đơn điệu tăng, đơn điệu giảm,
đơn điệu không giảm, đơn điệu không tăng.
Định nghĩa về hàm đơn điệu trên [a; b]; [a; b); (a; b]; (a; +1); [a; +1); ( 1; b); ( 1; b] thay cho
(a; b):
Chú thích 2. Cũng có một số gọi tên khác như sau:
Hàm tăng = tăng ngặt, đồng biến nghiêm cách;
Hàm giảm = giảm ngặt, nghịch biến nghiêm cách;
Hàm không giảm = đồng biến;
Hàm không tăng = nghịch biến.
Chú thích 3. Bốn định nghĩa (i)-(iv) là 4 khái niệm khác nhau và độc lập, tức là phủ định một trong
4 định nghĩa (i)-(iv) không phải là một trong ba định nghĩa còn lại. Ví dụ, hàm không tăng trên (a; b)
không có nghĩa là không phải là hàm tăng trên (a; b). Bởi vì hàm f không phải là hàm tăng trên (a; b);
có nghĩa là: 9x; x0 2 (a; b); x < x0 và f (x) f (x0 ); trong khi đó để dãy hàm f không tăng trên (a; b) thì
bất đẳng thức f (x) f (x0 ) phải đúng 8x; x0 2 (a; b); x < x0 :
Định lý 10. Cho f : (a; b) ! R: Giả sử f là hàm tăng (không giảm) trên (a; b) bị chận trên. Khi đó,
tồn tại lim f (x):
x!b
Hơn nữa, lim f (x) = supff (x) : a < x < bg:
x!b

Chứng minh Định lý 10. Do f bị chận trên, tập ff (x) : a < x < bg 6= và cũng bị chận trên, do đó
tồn tại L = supff (x) : a < x < bg: Ta sẽ chứng minh rằng lim f (x) = L:
x!b
Cho " > 0; do L = supff (x) : a < x < bg; ta có x 2 (a; b) : f (x ) > L ":
Do f là hàm tăng trên (a; b); nên mọi x 2 (x ; b) dẫn đến L " < f (x ) f (x) L < L + "; tức là
jf (x) Lj < ":
Chương 4. GIỚI HẠN HÀM SỐ 48

Vậy lim f (x) = L:


x!b

Chú ý =b x > 0; b = x < x < b =) L " < f (x ) f (x) L =) jf (x) Lj < ":
Tương tự ta cũng có
Định lý 11. Cho f : (a; b) ! R: Giả sử f là hàm giảm (không tăng) trên (a; b) bị chận dưới. Khi
đó, tồn tại lim f (x) = infff (x) : a < x < bg:
x!b

Chứng minh Định lý 11. Do f bị chận dưới, tập ff (x) : a < x < bg 6= và cũng bị chận dưới, do đó
tồn tại m = infff (x) : a < x < bg: Ta sẽ chứng minh rằng lim f (x) = m:
x!b
Cho " > 0; do m = infff (x) : a < x < bg; ta có x 2 (a; b) : f (x ) < m + ":
Do f là hàm giảm trên (a; b); nên mọi x 2 (x ; b) dẫn đến m " < m f (x) f (x ) < m + "; tức
là jf (x) mj < ":
Vậy lim f (x) = m:
x!b
Tương tự, ta cũng có
Định lý 12. Cho f : (a; b) ! R: Giả sử f là hàm tăng (không giảm) trên (a; b) bị chận dưới. Khi
đó, tồn tại lim f (x):
x!a+
Hơn nữa, lim f (x) = infff (x) : a < x < bg:
x!a+

Định lý 13. Cho f : (a; b) ! R: Giả sử f là hàm giảm (không tăng) trên (a; b) bị chận trên. Khi đó,
tồn tại lim f (x):
x!a+
Hơn nữa, lim f (x) = supff (x) : a < x < bg:
x!a+
Chương 4. GIỚI HẠN HÀM SỐ 49

4.4 Giới hạn vô cùng tại điểm tụ


Định nghĩa 8. Cho 6= D R và a 2 D . Cho f : D ! R:

4.4.1 Giới hạn vô cùng tại điểm tụ


(i) Ta nói f (x) tiến ra cộng vô cùng khi x ! a nếu:

8A 2 R; 9 > 0 : 8x 2 D; 0 < jx aj < =) f (x) > A:

Ta còn nói f (x) có giới hạn cộng vô cùng tại a; và viết thành một trong các cách sau:

lim f (x) = +1; hay f (x) ! +1 khi x ! a:


x!a

(ii) Ta nói f (x) tiến ra trừ vô cùng khi x ! a nếu:

8A 2 R; 9 > 0 : 8x 2 D; 0 < jx aj < =) f (x) < A:

Ta còn nói f (x) có giới hạn trừ vô cùng tại a; và viết thành một trong các cách sau:

lim f (x) = 1; hay f (x) ! 1 khi x ! a:


x!a

4.4.2 Giới hạn vô cùng bên phải tại điểm tụ


(i) Ta nói f (x) tiến ra cộng vô cùng khi x ! a+ nếu:

8A 2 R; 9 > 0 : 8x 2 D; 0 < x a< =) f (x) > A:

Ta còn nói f (x) có giới hạn cộng vô cùng bên phải tại a; và viết thành một trong các cách sau:

lim f (x) = +1; hay f (x) ! +1 khi x ! a+ :


x!a+

(ii) Ta nói f (x) tiến ra trừ vô cùng khi x ! a+ nếu:

8A 2 R; 9 > 0 : 8x 2 D; 0 < x a< =) f (x) < A:

Ta còn nói f (x) có giới hạn trừ vô cùng bên phải tại a; và viết thành một trong các cách sau:

lim f (x) = 1; hay f (x) ! 1 khi x ! a+ :


x!x+
0

4.4.3 Giới hạn vô cùng bên trái tại điểm tụ


(i) Ta nói f (x) tiến ra cộng vô cùng khi x ! a nếu:

8A 2 R; 9 > 0 : 8x 2 D; 0 < a x< =) f (x) > A:

Ta còn nói f (x) có giới hạn cộng vô cùng bên trái tại a; và viết thành một trong các cách sau:

lim f (x) = +1; hay f (x) ! +1 khi x ! a :


x!a

(ii) Ta nói f (x) tiến ra trừ vô cùng khi x ! a nếu:

8A 2 R; 9 > 0 : 8x 2 D; 0 < a x< =) f (x) < A:

Ta còn nói f (x) có giới hạn trừ vô cùng bên trái tại a; và viết thành một trong các cách sau:

lim f (x) = 1; hay f (x) ! 1 khi x ! a :


x!a
Chương 4. GIỚI HẠN HÀM SỐ 50

Tóm tắt lại 6 định nghĩa trên như sau:


lim f (x) = +1 () 8A 2 R; 9 > 0 : 8x 2 D; 0 < jx aj < =) f (x) > A;
x!a
lim f (x) = 1 () 8A 2 R; 9 > 0 : 8x 2 D; 0 < jx aj < =) f (x) < A;
x!a
lim f (x) = +1 () 8A 2 R; 9 > 0 : 8x 2 D; 0 < x a< =) f (x) > A;
x!a+
lim f (x) = 1 () 8A 2 R; 9 > 0 : 8x 2 D; 0 < x a< =) f (x) < A;
x!a+
lim f (x) = +1 () 8A 2 R; 9 > 0 : 8x 2 D; 0 < a x< =) f (x) > A;
x!a
lim f (x) = 1 () 8A 2 R; 9 > 0 : 8x 2 D; 0 < a x< =) f (x) < A:
x!a

4.5 Giới hạn vô cùng tại vô cùng


4.5.1 Định nghĩa giới hạn vô cùng tại +1.
Cho 6= D R sao cho tồn tại fxn g D; 8n 2 N; xn ! +1:
(i) Ta nói f (x) tiến ra cộng vô cùng khi x ! +1 nếu:
8A 2 R; 9N 2 R : 8x 2 D; x > N =) f (x) > A:
Ta còn nói f (x) có giới hạn cộng vô cùng tại +1; và viết thành một trong các cách sau:
lim f (x) = +1; hay f (x) ! +1 khi x ! +1:
x!+1

(ii) Ta nói f (x) tiến ra trừ vô cùng khi x ! +1 nếu:


8A 2 R; 9N 2 R : 8x 2 D; x > N =) f (x) < A:
Ta còn nói f (x) có giới hạn trừ vô cùng tại +1; và viết thành một trong các cách sau:
lim f (x) = 1; hay f (x) ! 1 khi x ! +1:
x!+1

4.5.2 Định nghĩa giới hạn vô cùng tại 1.


Cho 6= D R sao cho tồn tại fxn g D; 8n 2 N; xn ! 1:
(i) Ta nói f (x) tiến ra cộng vô cùng khi x ! 1 nếu:
8A 2 R; 9N 2 R : 8x 2 D; x < N =) f (x) > A:
Ta còn nói f (x) có giới hạn cộng vô cùng tại 1; và viết thành một trong các cách sau:
lim f (x) = +1; hay f (x) ! +1 khi x ! 1:
x! 1

(ii) Ta nói f (x) tiến ra trừ vô cùng khi x ! 1 nếu:


8A 2 R; 9N 2 R : 8x 2 D; x < N =) f (x) < A:
Ta còn nói f (x) có giới hạn trừ vô cùng tại 1; và viết thành một trong các cách sau:
lim f (x) = 1; hay f (x) ! 1 khi x ! 1:
x! 1

Tóm lại tắt 4 định nghĩa trên như sau:


lim f (x) = +1 () 8A 2 R; 9N 2 R : 8x 2 D; x > N =) f (x) > A;
x!+1
lim f (x) = 1 () 8A 2 R; 9N 2 R : 8x 2 D; x > N =) f (x) < A;
x!+1
lim f (x) = +1 () 8A 2 R; 9N 2 R : 8x 2 D; x < N =) f (x) > A;
x! 1
lim f (x) = 1 () 8A 2 R; 9N 2 R : 8x 2 D; x < N =) f (x) < A:
x! 1
Chương 4. GIỚI HẠN HÀM SỐ 51

Chú thích.
(i) Chú ý rằng giới hạn vô cùng của hàm f tại điểm tụ, ở bên phải, bên trái của điểm tụ, tại vô cùng
trên đây được xếp vào loại hàm không tồn tại giới hạn (không có giới hạn hoặc không có giới hạn hữu
hạn).
(ii) Trong trường hợp hàm f : [a; +1) ! R là tăng (không giảm) và không bị chận trên, khi đó ta
có lim f (x) = +1:
x!+1
Thật vậy, do hàm f không bị chận trên, ta có

8A > 0; 9xA 2 [a; +1) : f (xA ) > A:

Do hàm f tăng, nên, 8x > xA =) f (x) > f (xA ) > A:


Vậy, lim f (x) = +1:
x!+1
Tương tự,
(iii) Nếu hàm f : [a; +1) ! R là giảm (không tăng) và không bị chận dưới, thì lim f (x) = 1:
x!+1
(iv) Nếu hàm f : ( 1; b] ! R là tăng (không giảm) và không bị chận dưới, thì lim f (x) = 1:
x! 1
(v) Nếu hàm f : ( 1; b] ! R là giảm (không tăng) và không bị chận trên, thì lim f (x) = +1:
x! 1
(vi) Tương tự như dãy số, các phép tính giới hạn của hàm số cũng được mở rộng cho các phép tính
với giới hạn vô cực như sau.

(+1) (+1) = +1; ( 1) ( 1) = +1; ( 1) (+1) = 1; (+1) ( 1) = 1;


a (+1) = +1; a ( 1) = 1; ( a) ( 1) = +1; ( a) (+1) = 1; a > 0;
a
= 0; a 2 R:
1

Ngoài ra cũng có các dạng vô định (không xác định)


0 1
; ; 0 ( 1) ; ( 1) 0; 11 ; 00 :
0 1

4.6 Tổng kết các loại giới hạn (15 loại)


(i) lim f (x) = L 2 R; lim f (x) = +1; lim f (x) = 1;
x!a x!a x!a
(ii) lim f (x) = L 2 R; lim f (x) = +1; lim f (x) = 1;
x!a+ x!a+ x!a+
(iii) lim f (x) = L 2 R; lim f (x) = +1; lim f (x) = 1;
x!a x!a x!a
(iv) lim f (x) = L 2 R; lim f (x) = +1; lim f (x) = 1;
x!+1 x!+1 x!+1
(v) lim f (x) = L 2 R; lim f (x) = +1; lim f (x) = 1:
x! 1 x! 1 x! 1

4.7 Một số hàm sơ cấp

x , (x 2 R+ ; 2 R; (x; ) 6= (0; 0)),


sin x, cos x, tg x, cotg x,
ex , ln x;
ax , loga x, (0 < a 6= 1);
arcsin x, arccos x, arctg x, arccotg x,
ex + e x ex e x
chx = , shx = ,
2 x x
2
shx e e
thx = = x ,
chx e +e x
1 e +e x
x
coth x = = x :
thx e e x
Chương 4. GIỚI HẠN HÀM SỐ 52

4.8 Bài tập bổ sung


1. Cho các ví dụ về tập con khác rỗng A của R sao cho:
(i) Tồn tại sup A và max A;
(ii) Tồn tại sup A và không tồn tại max A;
(iii) Tồn tại inf A và min A;
(iv) Tồn tại inf A và không tồn tại min A:
2. Cho hai tập con khác rỗng A; B của R: Giả sử sup; inf, max, min của hai tập A và B tồn tại. Giả
sử A B:
Hãy so sánh các giá trị sup A; inf A, max A, min A; sup B; inf B, max B, min B với nhau.
Hướng dẫn: Biện luận trường hợp max, min tồn tại và không tồn tại.
(i) max A, min A; max B, min B tồn tại: sup A = max A, inf A = min A; sup B = max B, inf B =
min B:
max A max B; min A min B:
(ii) max A, min A; max B, min B không tồn tại: sup A sup B; inf A inf B:
3. Cho hai tập con khác rỗng A; B của R và A \ B 6= : Giả sử sup; inf, max, min của hai tập A
và B tồn tại. Hãy cho các công thức sup A [ B; inf A [ B, max A [ B, min A [ B; sup A \ B; inf A \ B,
max A \ B, min A \ B liên hệ các giá trị sup A; inf A, max A, min A; sup B; inf B, max B, min B:
Hướng dẫn: Biện luận trường hợp max, min tồn tại và không tồn tại.
(a) A [ B
(i) max A, min A; max B, min B tồn tại: max A [ B = maxfmax A; max Bg; min A [ B =
minfmin A; min Bg;
(ii) max A, min A; max B, min B không tồn tại: sup A [ B = maxfsup A; sup Bg; inf A [ B =
minfinf A; inf Bg;
(iii) max A tồn tại, max B không tồn tại: sup A [ B = maxfmax A; sup Bg;
(iv) min A tồn tại, min B không tồn tại: inf A [ B = minfmin A; inf Bg:
(b) A \ B
(i) max A, max B tồn tại: max(A \ B) minfmax A, max Bg:
Dấu bằng không luôn luôn đúng, bởi vì: A = f1; 2g; B = f1; 3); ta có max(A \ B) = maxf1g = 1;
max A = 2, max B = 3; minfmax A, max Bg = minf2; 3g = 2:
(ii) max A, max B không tồn tại: sup(A \ B) minfsup A, sup Bg:
(iii) max A tồn tại, max B không tồn tại: sup(A \ B) minfmax A, sup Bg:
(iv) min A, min B tồn tại: min(A \ B) maxfmin A, min Bg:
Dấu bằng không luôn luôn đúng, bởi vì: A = f1; 3g; B = f2; 3); ta có min(A \ B) = minf3g = 3;
min A = 1, min B = 2; maxfmin A, min Bg = maxf1; 2g = 2:
(v) min A, min B không tồn tại: inf(A \ B) maxfinf A, inf Bg:
(v) min A tồn tại, min B không tồn tại: inf(A \ B) maxfmin A, inf Bg:
4. Cho hai tập con khác rỗng A; B của R:
(i) Giả sử A và B bị chận trên. Chứng minh A [ B cũng bị chận trên.
(ii) Giả sử A và B bị chận dưới. Chứng minh A [ B cũng bị chận dưới.
(iii) Giả sử A và B bị chận. Chứng minh A [ B cũng bị chận.
5. Cho dãy số thực fxn g và cho N 2 N: Đặt AN = fxn : n N g: Chứng minh rằng:
(i) Nếu AN = fxn : n N g bị chận trên thì dãy fxn g bị chận trên.
(ii) Nếu AN = fxn : n N g bị chận dưới thì dãy fxn g bị chận dưới.
(iii) Nếu AN = fxn : n N g bị chận thì dãy fxn g bị chận.
6. Cho f (x) = 3x + 7: Chứng minh rằng lim f (x) = 10:
x!1
3x + 5
7. Cho f (x) = : Chứng minh rằng lim f (x) = +1:
x 2 x!2+
8. Chứng minh rằng lim x 3 4x = +1:
x!+1
x3 + 1
9. Chứng minh rằng lim = +1:
x!+1 x2 + 1
10. Cho f (x) = sin(1=x): Chứng minh rằng không tồn tại lim f (x):
x!0
Chương 4. GIỚI HẠN HÀM SỐ 53

11. Chứng minh rằng lim x sin(1=x) tồn tại và tính lim x sin(1=x):
x!0 x!0
x sin x + cos x x sin x + cos x
12. Chứng minh rằng lim tồn tại và tính lim :
x!+1 x2 + x + 1 x!+1 x2 + x + 1
2
13. Cho f (x) = sin(x ): Chứng minh rằng:
(i) Không tồn tại lim f (x):
x!+1
1
(ii) Tồn tại một dãy số thực fxn g sao cho xn ! +1 và f (xn ) ! :
2
(iii) Không tồn tại một dãy số thực fxn g sao cho xn ! +1 và f (xn ) ! 2:
14. Cho f (x) = cos2 (x3 ): Chứng minh rằng:
(i) Không tồn tại lim f (x):
x!+1
(ii) Với mọi L 2 [0; 1]; tồn tại một dãy số thực fxn g sao cho xn ! +1 và f (xn ) ! L:
(iii) Với mọi L > 1; không tồn tại một dãy số thực fxn g sao cho xn ! +1 và f (xn ) ! L:
15. Cho f (x) = x sin(x): Chứng minh rằng:
(i) Tồn tại một dãy số thực fxn g sao cho xn ! +1 và f (xn ) ! 0:
(ii) Tồn tại một dãy số thực fxn g sao cho xn ! +1 và f (xn ) ! +1:
(iii) Tồn tại một dãy số thực fxn g sao cho xn ! +1 và f (xn ) ! 1:
16. Cho 6= D R và a 2 D . Cho f : D ! R: Giả sử lim f (x) = L > 0: Chứng minh rằng tồn tại
x!a
> 0 sao cho f (x) > 0; 8x 2 (a ; a + ) \ (Dnfag) :
17. Cho hàm f : [a; b) ! R là tăng và không bị chận trên. Chứng minh rằng lim f (x) = +1:
x!b
Hướng dẫn bài 17:
Tập f ([a; b)) bị chận trên của R () 9A 2 R : f (x) A, 8x 2 [a; b):
Tập f ([a; b)) không bị chận trên của R () 8A 2 R; 9xA 2 [a; b) : f (xA ) > A:
Cho A 2 R; do f ([a; b)) không bị chận trên, nên 9xA 2 [a; b) : f (xA ) > A:
Chọn = b xA > 0; khi đó, do f là hàm tăng trên [a; b), nên f (x) > f (xA ) > A: Vậy lim f (x) = +1:
x!b

18. Cho hàm f : [a; b) ! R sao cho lim jf (x)j =


6 +1: Chứng minh rằng, tồn tại một dãy số thực
x!b
fxn g [a; b) sao cho xn ! b và dãy ff (xn )g bị chặn.
Chương 4. GIỚI HẠN HÀM SỐ 54

19. Cho hàm f : [a; +1) ! R là tăng và bị chận trên. Chứng minh rằng lim f (x) tồn tại và
x!+1
lim f (x) = sup f ([a; +1)):
x!+1
20. Cho hàm f : [a; +1) ! R là tăng và không bị chận trên. Chứng minh rằng lim f (x) = +1:
x!+1
21. Cho hàm f : [1; +1) ! R không bị chận trên sao cho f ([1; b]) bị chận trên với mọi b > 1: Chứng
minh rằng tồn tại một dãy fxn g [1; +1) sao cho xn ! +1 và f (xn ) ! +1:
22. Cho hàm f : [1; +1) ! R không bị chận trên. Chứng minh rằng:
(i) Tồn tại một dãy fxn g [1; +1) sao cho xn ! +1 và f (xn ) ! +1;
hoặc
(ii) Tồn tại một dãy fxn g hội tụ sao cho f (xn ) ! +1:
23. Cho hàm f : [a; +1) ! R là giảm và bị chận dưới. Chứng minh rằng lim f (x) tồn tại và
x!+1
lim f (x) = inf f ([a; +1)):
x!+1
24. Cho hàm f : [a; +1) ! R là giảm và không bị chận dưới. Chứng minh rằng lim f (x) = 1:
x!+1
25. Cho hàm f : [1; +1) ! R không bị chận dưới sao cho f ([1; b]) bị chận dưới với mọi b > 1: Chứng
minh rằng tồn tại một dãy fxn g [1; +1) sao cho xn ! +1 và f (xn ) ! 1:
26. Cho hàm f : [1; +1) ! R không bị chận dưới. Chứng minh rằng:
(i) Tồn tại một dãy fxn g [1; +1) sao cho xn ! +1 và f (xn ) ! 1;
hoặc
(ii) Tồn tại một dãy fxn g hội tụ sao cho f (xn ) ! 1:
27. Hãy cho ví dụ về một hàm f : [1; +1) ! R không bị chận trên sao cho không tồn tại một dãy
fxn g [1; +1) sao cho xn ! +1 và f (xn ) ! +1:
28. Hãy cho ví dụ về một hàm f : [1; +1) ! R không bị chận dưới sao cho không tồn tại một dãy
fxn g [1; +1) sao cho xn ! +1 và f (xn ) ! 1:

Hướng dẫn bài 19: Do f ([a; +1)) là tập con 6= và bị chận trên của R, nên tồn tại L = sup f ([a; +1)):
Ta chứng minh rằng lim f (x) = L:
x!+1
Cho " > 0; do L = sup f ([a; +1)); tồn tại x" a sao cho f (x" ) > L ":
Do hàm f tăng, nên 8x > x" ; ta có L " < f (x" ) < f (x) L < L+", điều này dẫn tới jf (x) Lj < ":
Vậy lim f (x) = L:
x!+1
Hướng dẫn bài 20: Do f ([a; +1)) không bị chận trên, nên 8A 2 R; 9xA a : f (xA ) > A:
Do hàm f tăng, nên 8x > xA ; ta có f (x) > f (xA ) > A: Vậy lim f (x) = +1:
x!+1
Hướng dẫn bài 21: Do f ([1; +1)) không bị chận trên, nên tồn tại x1 1 sao cho f (x1 ) > 1:
Đặt x ~1 = maxf2; x1 g và do f ([1; x
~1 ]) bị chận trên, nên tập f ([~
x1 ; +1)) không bị chận trên, nên tồn
tại x2 x ~1 sao cho f (x2 ) > 2:
Tiếp tục quá trình lý luận trên ta thu được dãy fxn g sao cho xn n và f (xn ) n; 8n 2 N: Do đó
xn ! +1 và f (xn ) ! +1:
Hướng dẫn bài 22: Do f ([1; +1)) không bị chận trên, nên tồn tại một dãy fxn g [1; +1) sao cho
f (xn ) n; 8n 2 N:
(i) Nếu fxn g không bị chận trên, nên tồn tại một dãy con fxnk g fxn g sao cho xnk ! +1: Mà
f (xnk ) nk k ! +1: Vậy chọn yk = xnk ; ta có yk ! +1 và f (yk ) ! +1:
(ii) Nếu fxn g bị chận trên, khi đó fxn g bị chận, theo Định lý Bolzano-Weierstrass, tồn tại một dãy
con fxnk g fxn g sao cho xnk ! a: Mà f (xnk ) nk k ! +1: Vậy chọn yk = xnk ; ta có fyk g hội tụ
và f (yk ) ! +1:
Hướng dẫn bài 23: Chứng minh tương tự bài 19.
Hướng dẫn bài 24: Chứng minh tương tự bài 20.
Hướng dẫn bài 25: Chứng minh tương tự bài 21.
Hướng dẫn bài 26: Chứng minh tương tự bài 22.
Chương 5

HÀM SỐ THỰC LIÊN TỤC

5.1 Các định nghĩa


5.1.1 Điểm tụ, điểm cô lập
Định nghĩa điểm tụ. Cho 6= D R và a 2 R. Ta nói a là một điểm tụ của D nếu

(a ; a + ) \ (Dnfag) 6= ; 8 > 0:

Định nghĩa điểm cô lập. Cho 6= D R, a 2 D được gọi là điểm cô lập của D nếu

9 > 0 : (a ; a + ) \ (Dnfag) = :

5.1.2 Hàm số liên tục tại một điểm


Định nghĩa 1. Cho 6= D R, f : D ! R và a 2 D:
Nếu a là điểm cô lập của D: Ta nói f liên tục tại a:
Nếu a là điểm tụ của D: Ta nói f liên tục tại a 2 D nếu lim f (x) = f (a):
x!a
Trong trường hợp, a 2 D là điểm tụ của D: Ta cũng có

f liên tục tại a () lim f (x) = f (a)


x!a
() 8" > 0; 9 > 0 : 8x 2 D; jx aj < =) jf (x) f (a)j < ":

Với a 2 D sao cho (a; a + ) \ D 6= ; 8 > 0: Ta định nghĩa hàm f liên tục bên phải tại a như sau:
Ta nói f liên tục bên phải tại a 2 D nếu lim f (x) = f (a); tức là,
x!a+

8" > 0; 9 > 0 : 8x 2 D \ [a; a + ) =) jf (x) f (a)j < ":

Tương tự, với a 2 D sao cho (a ; a) \ D 6= ; 8 > 0: Ta cũng có định nghĩa hàm f liên tục bên
trái tại a như sau:
Ta nói f liên tục bên trái tại a 2 D nếu lim f (x) = f (a); tức là,
x!a

8" > 0; 9 > 0 : 8x 2 D \ (a ; a] =) jf (x) f (a)j < ":

Hiển nhiên, điều kiện cần và đủ để hàm f liên tục tại a là f liên tục bên phải và bên trái tại a:

f liên tục tại a () f liên tục bên phải tại a và bên trái tại a:

55
Chương 5. HÀM SỐ THỰC LIÊN TỤC 56

5.1.3 Hàm số liên tục trong khoảng, trên đoạn, trên một tập
Định nghĩa 2.
(i) Hàm f : (a; b) ! R được gọi là liên tục trong khoảng (a; b) nếu f liên tục tại mọi điểm
x0 2 (a; b):
(ii) Hàm f : [a; b] ! R được gọi là liên tục trên đoạn [a; b] nếu f liên tục trong khoảng (a; b) và
liên tục bên phải tại a; liên tục bên trái tại b:
(iii) Cho 6= D R, hàm f : D ! R được gọi là liên tục trên tập D nếu f liên tục tại mọi điểm
a 2 D; hay
8a 2 D; 8" > 0; 9 > 0 : 8x 2 D \ (a ; a + ) =) jf (x) f (a)j < ":

5.2 Các phép toán trên các hàm số liên tục tại một điểm
Áp dụng các phép toán đơn giản về các hàm số có giới hạn ta có một số kết quả sau đây:
Định lý 1. Nếu hàm f là liên tục tại điểm a thì hàm jf j cũng liên tục tại a:
Định lý 2. Nếu các hàm f và g liên tục tại điểm a thì các hàm f + g; f g; cf (c là hằng số ) cũng
liên tục tại a:
f
Ngoài ra, nếu g(a) 6= 0 thì hàm liên tục tại a:
g
Định lý 3. Giả sử D; E R và f : D ! E; g : E ! R: Nếu hàm f liên tục tại điểm a và g liên tục
tại điểm b = f (a) 2 E; thì hàm hợp g f : D ! R cũng liên tục tại a:

5.3 Điểm gián đoạn. Phân loại


Định nghĩa 3. Hàm f được gọi là gián đoạn tại a nếu f không liên tục tại điểm a: Lúc đó a điểm
gián đoạn của f:
Chú thích. Nếu f gián đoạn tại a thì đồ thị của hàm y = f (x) không liền tại điểm M0 (a; f (a)); mà
bị ngắt quảng tại M0 :
Căn cứ vào định nghĩa ta thấy rằng hàm f gián đoạn tại a nếu gặp một trong các trường hợp sau:
i) Nếu tồn tại các giới hạn bên phải f+ (a) = lim f (x); giới hạn bên trái f (a) = lim f (x) và ba số
x!a+ x!a
thực f (a); f+ (a); f (a) không đồng thời bằng nhau, thì ta nói a là điểm gián đoạn loại một.
j) Nếu f+ (a) = f (a) 6= f (a); thì ta nói a là điểm gián đoạn bỏ được.
jj) Nếu f+ (a) 6= f (a); thì ta nói a là điểm nhảy. Hiệu số f+ (a) f (a) được gọi là bước nhảy
tại a của hàm f .
ii) Điểm gián đoạn không thuộc loại một được gọi là điểm gián đoạn loại hai.
Ví dụ: Xét hàm
x + 1; nếu x 0;
f (x) =
x 1; nếu x < 0:
Ta có: f+ (0) = lim f (x) = 1 6= f (0) = lim f (x) = 1:
x!0+ x!0
Vậy x = 0 là một điểm nhảy, với bước nhảy là f (+0) f ( 0) = 2:
Ví dụ: Xét hàm ( sin x
; nếu x 6= 0;
f (x) = x
2; nếu x = 0:
Vì lim f (x) = lim f (x) = 1 6= f (0) = 2; nên gián đoạn loại một tại x = 0: Hơn nữa, x = 0 là một
x!0+ x!0
điểm gián đoạn bỏ được.
Chương 5. HÀM SỐ THỰC LIÊN TỤC 57

Nếu xét hàm ( sin x


; nếu x 6= 0;
f~(x) = x
1; nếu x = 0:
thì f~ sẽ liên tục tại x = 0: Như vậy bằng cách chỉnh sửa giá trị hàm tại x = 0, ta đã chỉnh sửa từ một
hàm f không liên tục tại x = 0 thành một hàm f~ liên tục tại x = 0. Điều nầy giải thích từ "bỏ được"
tính bất liên tục của hàm số.
Ví dụ: Hàm f (x) = x1 có điểm gián đoạn loại hai tại x = 0; vì lim x1 = +1; lim x1 = 1:
x!0+ x!0

5.4 Tính liên tục của các hàm sơ cấp


Ta sẽ chỉ ra rằng các hàm sơ cấp đều liên tục trên tập xác định của chúng.
1/ Đa thức Pn (x) = a0 xn + a1 xn 1 + + an 1 x + an :
Vì hàm số y = C = hằng và hàm số y = x liên tục trên R nên hàm số

x 7 ! axk = a xx
| {z x}
k thừa số

trong đó a là một số tực không đổi và k là một số tự nhiên, liên tục trên R: Do đó hàm Pn (x) là tổng
hữu hạn các hàm thuộc dạng trên cũng liên tục trên R:
P
Hàm hữu tỉ ; trong đó P và Q là các đa thức, liên tục tại mọi điểm x 2 R tại đó Q(x) 6= 0:
Q
2/ Hàm mũ y = ax (a > 0) liên tục trên R:
Giả sử x0 2 R: Với mọi x 2 R; ta có ax = ax0 ax x0 :
Khi x ! x0 ta có x x0 ! 0 và ax x0 ! 1: Do đó lim ax = ax0 : Vậy hàm y = ax liên tục tại điểm
x!x0
x0 : Ta có:

lim ax = +1 và lim ax = 0 với a > 1;


x!+1 x! 1
x
lim a = 0 và lim ax = +1 với 0 < a < 1:
x!+1 x! 1

Tập các giá trị của hàm số y = ax là khoảng (0; +1):


3/ Hàm số Lôgarit y = loga x (0 < a 6= 1) liên tục trên (0; +1):(Xem mục 5.5)
x
Giả sử x0 > 0: Với mọi x 2 R; ta có loga x = loga x0 + loga :
x0
x x
Khi x ! x0 ta có ! 1 và loga ! 0: Do đó lim loga x = loga x0 :
x0 x0 x!x0
Vậy hàm y = loga x liên tục tại điểm x0 : Ta có:

lim loga x = 1 và lim loga x = +1 nếu a > 1;


x!0+ x!+1

lim loga x = +1 và lim loga x = 1 nếu 0 < a < 1:


x!0+ x!+1

4/ Hàm số lũy thừa y = x ( 2 R) liên tục trên (0; +1): Vì x = e ln x nên theo định lý về tính
liên tục của hàm số hợp, hàm số lũy thừa liên tục trên (0; +1):
5/ Các hàm số lượng giác liên tục trên tập xác định của chúng.
Thật vậy, Giả sử x0 2 R: Với mọi x 2 R; ta có
x + x0 x x0 x x0
jsin x sin x0 j = 2 cos sin 2 sin jx x0 j :
2 2 2

Từ đó suy ra lim sin x = sin x0 :


x!x0
Vậy hàm số y = sin x liên tục tại điểm x0 ; tức là liên tục trên R:
Vì cos x = sin( 2 x) với mọi x 2 R; nên theo định lý về tính liên tục của hàm số hợp, suy ra hàm
số y = cos x liên tục trên R:
Chương 5. HÀM SỐ THỰC LIÊN TỤC 58

sin x
Cũng theo tính chất hàm liên tục ta có hàm số y = tgx = liên tục tại mọi điểm x 2 R mà
cos x
+ k ; k 2 Z tập các số nguyên.
cos x 6= 0; tức là x 6= 2
cos x
Hàm số y = cotg x = liên tục tại mọi điểm x 2 R mà sin x 6= 0; tức là x 6= k ; k 2 Z:
sin x
6/ Người ta chứng minh được rằng các hàm lượng giác ngược liên tục trên tập xác định của chúng.
Cụ thể là
- Hàm số y = arcsin x liên tục và tăng trên từ [ 1; 1] lên [ 2 ; 2 ]:
- Hàm số y = arccos x liên tục và giảm trên từ [ 1; 1] lên [0; ]:
- Hàm số y = arctgx liên tục và tăng trên từ R lên ( 2 ; 2 ):
- Hàm số y = arccotg x liên tục và giảm trên từ R lên (0; ):

5.5 Tính chất của hàm liên tục trên một đoạn
Định lý 4. Cho f : [a; b] ! R liên tục trên đoạn [a; b]: Khi đó
(i) f bị chận trên đoạn [a; b], tức là 9M > 0 : jf (x)j M; 8x 2 [a; b]:
(ii) f đạt giá trị lớn nhất và giá trị nhỏ nhất trên đoạn [a; b]; tức là

9x ; x 2 [a; b] : f (x ) f (x) f (x ) 8x 2 [a; b],

nghĩa là 9x ; x 2 [a; b] :

f (x ) = min f ([a; b]) = minff (x) : x 2 [a; b]g;


f (x ) = max f ([a; b]) = maxff (x) : x 2 [a; b]g:

(iii) Nếu min f ([a; b]) k max f ([a; b]); thì 9c 2 [a; b] sao cho k = f (c):
(iv) Nếu f (a)f (b) < 0; thì 9c 2 (a; b) : f (c) = 0: (xem hình ở dưới)
(v) f ([a; b]) là một đoạn trong R, tức là 9c; d 2 R; c d : f ([a; b]) = [c; d]:
(vi) Nếu f là một song ánh từ f : [a; b] ! f ([a; b]): Khi đó ánh xạ ngược f 1 : f ([a; b]) ! [a; b] cũng
liên tục trên đoạn f ([a; b]):
(vii) Nếu f tăng (tương ứng giảm) trên đoạn [a; b]: Khi đó f là một song ánh từ [a; b] lên [f (a); f (b)]
(tương ứng [f (b); f (a)] ) và hàm số ngược f 1 : [f (a); f (b)] ! [a; b] (tương ứng f 1 : [f (b); f (a)] ! [a; b])
của hàm f là liên tục và tăng (tương ứng giảm).
Nhắc lại:
Một hàm f : [a; b] ! R được gọi là tăng (giảm) trên đoạn [a; b]; nếu

8x; x0 2 [a; b]; x < x0 =) f (x) < f (x0 ) (tương ứng f (x) > f (x0 )):

Một hàm f : [a; b] ! R được gọi là không giảm (không tăng) trên đoạn [a; b]; nếu

8x; x0 2 [a; b]; x < x0 =) f (x) f (x0 ) (tương ứng f (x) f (x0 )):

Chứng minh Định lý 4.


Chứng minh (i). Giả sử f không bị chận trên đoạn [a; b], tức là 8M > 0; 9xM 2 [a; b] : jf (xM )j > M .
Lấy M = n 2 N;
9xn 2 [a; b] : jf (xn )j > n; 8n 2 N:
Dùng Định lý Bolzano-Weierstrass, tồn tại một dãy con fxnk g fxn g và x0 2 [a; b] sao cho xnk ! x0 .
Do tính liên tục của f tại x0 , ta có jf (xnk )j ! jf (x0 )j :
Từ bất đẳng thức
jf (xnk )j nk ; 8k 2 N;
ta suy ra điều mâu thuẫn. Vậy f bị chận trên đoạn [a; b]: Vậy (i) được chứng minh xong.
Chứng minh (ii). Ta có 6= f ([a; b]) R và bị chặn trên. Do đó tồn tại M = sup f ([a; b]) = supff (x) :
x 2 [a; b]g:
Chương 5. HÀM SỐ THỰC LIÊN TỤC 59

1
Với mỗi n 2 N; lấy " = > 0; do M = sup f ([a; b]);
n
1
9xn 2 [a; b] : M < f (xn ) M; 8n 2 N:
n
Dùng Định lý Bolzano-Weierstrass, tồn tại một dãy con fxnk g fxn g và x 2 [a; b] sao cho xnk ! x .
Do tính liên tục của f tại x , ta có f (xnk ) ! f (x ):
Từ bất đẳng thức
1
M < f (xnk ) M; 8k 2 N;
nk
cho k ! +1; ta có
M f (x ) M:
Do đó
f (x ) = M = sup f ([a; b]) = max f ([a; b]):
Chứng minh tương tự ta cũng có x 2 [a; b] sao cho

f (x ) = inf f ([a; b]) = min f ([a; b]):

Vậy (ii) được chứng minh xong.


Chứng minh (iii). Do (ii), ta có

9x ; x 2 [a; b] : f (x ) f (x) f (x ) 8x 2 [a; b].

Nếu f (x ) = f (x ); hiển nhiên (iii) đúng.


Giả sử f (x ) < f (x ); ta có thể giả sử rằng x < x (nếu không ta thay f bởi f ).
Ta chỉ cần xét trường hợp f (x ) < k < f (x ): Ta có 6= A = fx 2 [x ; x ] : f (x) kg và bị chặn
trên. Do đó tồn tại c = sup A:
1
Với mỗi n 2 N; lấy " = > 0; do c = sup A; ta có
n
1
9xn 2 A : c < xn c; 8n 2 N:
n
hay
1
9xn 2 [x ; x ]; f (xn ) k:c < xn c; 8n 2 N:
n
Do đó ta suy ra xn ! c 2 [x ; x ]; và do tính liên tục của f tại c, ta có f (c) k:
Nếu c = x ; thì f (c) = f (x ) > k : Mâu thuẫn với f (c) k: Vậy c < x :
x c
Mặt khác, ta có yn = c + 2 [x ; x ] với n đủ lớn. Hơn nữa yn > c; dẫn đến yn 2
= A:
n
Do đó f (yn ) > k và yn ! c; dẫn đến f (c) k:
Vậy f (c) = k:
Vậy (iii) được chứng minh xong.
Chứng minh (iv). Áp dụng (iii) với min f ([a; b]) f (a) < k = 0 < f (b) max f ([a; b]):
Vậy (iv) được chứng minh xong.
Chứng minh (v). Đặt c = f (x ) = min f ([a; b]); d = f (x ) = max f ([a; b]):
Ta có f ([a; b]) [c; d]:
Mặt khác, ta sẽ chứng minh rằng [c; d] f ([a; b]): Mà điều này ta sẽ áp dụng (iii).
Vậy (v) được chứng minh xong.
Chứng minh (vi). g = f 1 : f ([a; b]) = [c; d] ! [a; b] cũng liên tục trên đoạn f ([a; b]) = [c; d]:

Cho fyn g [c; d]; yn ! y0 2 [c; d]: Ta chứng minh rằng g(yn ) ! g(y0 ): Giả sử g(yn ) 9 g(y0 ): Khi đó

9"0 > 0; 9fynk g fyn g và jg(ynk ) g(y0 )j "0 ; 8k 2 N:


Chương 5. HÀM SỐ THỰC LIÊN TỤC 60

Đặt x0 = g(y0 ); xnk = g(ynk ) 2 [a; b]; 8k 2 N:


Ta có
f (x0 ) = y0 ; jxnk x0 j "0 ; 8k 2 N:
Dùng Định lý Bolzano-Weierstrass, tồn tại một dãy con fxnkm g fxnk g và x
^ 2 [a; b] sao cho
xnkm ! x^. Ta có x^ 6= x0 ; do xnkm 9 x0 :
Do tính liên tục của f tại x ^, ta có ynkm = f (xnkm ) ! f (^
x): Ta có f (^
x) = y0 = f (x0 ): Do f là một
song ánh từ f : [a; b] ! [c; d]; ta có x
^ = x0 : Mâu thuẫn với x
^ 6= x0 :
Vậy (vi) được chứng minh xong.
Chứng minh (vii).

(vii) Nếu f tăng trên đoạn [a; b]: Khi đó f là một song ánh từ [a; b] lên [f (a); f (b)] = f ([a; b]): Áp
dụng (vi), ta có g = f 1 : f ([a; b]) = [f (a); f (b)] ! [a; b] cũng liên tục trên đoạn [f (a); f (b)]: Ta chỉ cần
kiểm tra g = f 1 tăng trên đoạn [f (a); f (b)]:
Cho y; y 0 2 [f (a); f (b)]; y < y 0 ; ta có x = g(y) < x0 = g(y 0 ): Thật vậy, Giả sử x = g(y) x0 = g(y 0 );
do f tăng, ta có
y = f (x) f (x0 ) = y 0 :
Mâu thuẫn với y < y 0 :
Chứng minh tương tự với f là hàm giảm trên đoạn [a; b]:
Vậy (vii) được chứng minh xong.
Định lý 4 được chứng minh xong.
Chương 5. HÀM SỐ THỰC LIÊN TỤC 61

5.6 Hàm số liên tục đều


Định nghĩa 4. Cho 6= D R, f : D ! R: Ta nói f liên tục đều trên D nếu

8" > 0; 9 > 0 : 8x; x0 2 D; x x0 < =) f (x) f (x0 ) < ":

Chú thích:
1/ f liên tục đều trên D thì liên tục tại mọi điểm thuộc D. Ngược lại không đúng ví dụ hàm f (x) = x2
liên tục trên R; nhưng không liên tục đều trên R:
p
2/ Ví dụ hàm f (x) = x liên tục đều trên R+ .
p p p
HD: Chứng minh bất đẳng thức x y jx yj; 8x; y 2 R+ :
Định lý 5. Cho f : [a; b] ! R liên tục trên đoạn [a; b]: Khi đó f liên tục đều trên [a; b]:
Chứng minh Định lý 5. Ta chỉ chứng minh phần thuận.
Cho f : [a; b] ! R liên tục trên đoạn [a; b]: Giả sử f không liên tục đều trên [a; b]: Khi đó

9"0 > 0; 8 > 0 : 9x ; y 2 [a; b]; jx y j< và jf (x ) f (y )j "0 :


1
Với mỗi n 2 N; lấy = > 0;
n
1
9xn ; yn 2 [a; b]; jxn yn j < và jf (xn ) f (yn )j "0 ; 8n 2 N:
n
Dùng Định lý Bolzano-Weierstrass, tồn tại một dãy con fxnk g fxn g và x0 2 [a; b] sao cho xnk ! x0 .
Chương 5. HÀM SỐ THỰC LIÊN TỤC 62

Mặt khác, ta có
1
jynk x0 j jynk xnk j + jxnk x0 j < + jxnk x0 j ! 0:
nk

Do tính liên tục của f tại x0 , ta có jf (xnk ) f (ynk )j ! jf (x0 ) f (y0 )j = 0:


Từ bất đẳng thức
jf (xnk ) f (ynk )j "0 ; 8k 2 N;
ta suy ra điều mâu thuẫn. Vậy f liên tục đều trên [a; b]:
Định lý 5 được chứng minh xong.
Ví dụ 1: (Xem như Bài tập). Cho f : R ! R liên tục trên R sao cho lim f (x) = lim f (x) = 0:
x!+1 x! 1
Chứng minh rằng f liên tục đều trên R:
Giải Ví dụ 1. Cho " > 0: Do lim f (x) = lim f (x) = 0; ta có N1 ; N2 > 0 :
x!+1 x! 1

"
8x < N1 =) jf (x)j < ;
2
"
8x > N2 =) jf (x)j < :
2
Do f liên tục đều trên đoạn [ N1 1; N2 + 1]; nên
"
9 > 0 : 8x; x0 2 [ N1 1; N2 + 1]; x x0 < =) f (x) f (x0 ) < :
2
Chọn 1 = minf1; g; và 8x; x0 2 R; jx x0 j < 1; ta phân biệt 5 trường hợp:
(i) x; x0 2 ( 1; N1 );
(ii) x; x0 2 (N2 ; +1);
(iii) x; x0 2 [ N1 ; N2 ];
(iv) x 2 ( 1; N1 ); x0 2 [ N1 ; N2 ];
(v) x 2 [ N1 ; N2 ]; x0 2 (N2 ; +1):
Do đó, nếu jx x0 j < 1 ; ta có
" "
(i) x; x0 2 ( 1; N1 ) : jf (x) f (x0 )j jf (x)j + jf (x0 )j <
+ = ":
2" 2"
(ii) x; x0 2 (N2 ; +1) : jf (x) f (x0 )j jf (x)j + jf (x0 )j < + = ":
2 2 "
(iii) x; x0 2 [ N1 ; N2 ] : x; x0 2 [ N1 1; N2 + 1] =) jf (x) f (x0 )j < < ":
2
"
(iv) x 2 ( 1; N1 ); x0 2 [ N1 ; N2 ] : x; x0 2 [ N1 1; N2 + 1] =) jf (x) f (x0 )j < < ":
2
"
(v) x 2 [ N1 ; N2 ]; x0 2 (N2 ; +1) : x; x0 2 [ N1 1; N2 + 1] =) jf (x) f (x0 )j < < ":
2
Cả 5 trường hợp trên đều dẫn đến jf (x) f (x0 )j < ":
Vậy f liên tục đều trên R:
Ví dụ 2: (Xem như Bài tập). Cho f : R ! R liên tục trên R sao cho lim f (x) = L1 2 R;
x! 1
lim f (x) = L2 2 R: Chứng minh rằng f liên tục đều trên R:
x!+1
Giải Ví dụ 2. Cho " > 0: Do lim f (x) = L1 2 R; lim f (x) = L2 2 R; ta có N1 ; N2 > 0 :
x! 1 x!+1

"
8x < N1 =) jf (x) L1 j < ;
2
"
8x > N2 =) jf (x) L2 j < :
2
Do f liên tục đều trên đoạn [ N1 1; N2 + 1]; nên
"
9 > 0 : 8x; x0 2 [ N1 1; N2 + 1]; x x0 < =) f (x) f (x0 ) < :
2
Chương 5. HÀM SỐ THỰC LIÊN TỤC 63

Chọn 1 = minf1; g; và 8x; x0 2 R; jx x0 j < 1; ta phân biệt 5 trường hợp:


(i) x0
x; 2 ( 1; N1 );
(ii) x; x0 2 (N2 ; +1);
(iii) x; x0 2 [ N1 ; N2 ];
(iv) x 2 ( 1; N1 ); x0 2 [ N1 ; N2 ];
(v) x 2 [ N1 ; N2 ]; x0 2 (N2 ; +1):
Do đó, nếu jx x0 j < 1 ; ta có
" "
(i) x; x0 2 ( 1; N1 ) : jf (x) f (x0 )j jf (x) L1 j + jL1 f (x0 )j <
+ = ":
2" 2"
(ii) x; x0 2 (N2 ; +1) : jf (x) f (x0 )j jf (x) L2 j + jL2 f (x0 )j < + = ":
2" 2
(iii) x; x0 2 [ N1 ; N2 ] : x; x0 2 [ N1 1; N2 + 1] =) jf (x) f (x0 )j < < ":
2
"
(iv) x 2 ( 1; N1 ); x0 2 [ N1 ; N2 ] : x; x0 2 [ N1 1; N2 + 1] =) jf (x) f (x0 )j < < ":
2
"
(v) x 2 [ N1 ; N2 ]; x0 2 (N2 ; +1) : x; x0 2 [ N1 1; N2 + 1] =) jf (x) f (x0 )j < < ":
2
Cả 5 trường hợp trên đều dẫn đến jf (x) f (x0 )j < ":
Vậy f liên tục đều trên R:

5.7 Bài tập bổ sung


1. Cho A = [0; 1] \ Q: Chứng minh rằng:
(i) Mọi điểm thuộc A đều là điểm tụ của A.
(ii) A không có điểm cô lập.
2. Cho dãy số thực fxn g hội tụ về a: Xét dãy số fyn g như sau yn = xn+10 ; n = 1; 2; : Chứng minh
rằng yn ! a:
3. Cho ví dụ về hai dãy số thực fxn g, fyn g phân kỳ mà fxn yn g hội tụ.
4. Cho ví dụ về hai dãy số thực fxn g, fyn g phân kỳ mà fxn yn g phân kỳ.
5. Cho ví dụ về hai dãy số thực fxn g, fyn g sao cho fxn g hội tụ fyn g phân kỳ và fxn yn g hội tụ.
6. Cho ví dụ về hai dãy số thực fxn g, fyn g sao cho fxn g hội tụ fyn g phân kỳ và fxn yn g phân kỳ.
7. Cho f : [0; 1] ! R liên tục trên [0; 1]: Chứng minh rằng nếu f (x) = 0; 8x 2 [0; 1] \ Q thì f (x) = 0;
8x 2 [0; 1]:
8. Cho hai hàm f; g : R ! R liên tục trên R: Chứng minh rằng nếu f (x) = g(x); 8x 2 Q thì
f (x) = g(x); 8x 2 R:
9. Cho 6= D R và f : D ! R: Cho a 2 D: Viết mệnh đề "f không liên tục tại a":
10. Cho 6= D R và f : D ! R: Cho a 2 D: là điểm tụ của D: Giả sử f liên tục tại a: Chứng minh
rằng nếu f (a) > 0; thì p tồn tại số thực > 0 sao cho f (x) > 0; 8x 2 D \ (a ; a + ):
3
x sin(1=x); x 6= 0;
11. Cho f (x) =
0; x = 0:
Xét sự liên tục của8 hàm f:
< x; x < 1;
12. Cho f (x) = 1; 1 x < 2;
: 4
x 15; x 2:
Xét sự liên tục của hàm f:
sin(1=x); x 6= 0;
13. Cho f (x) =
A; x = 0:
Có tồn tại số thực A sao cho f liên tục tại x = 0?:
14. Xét sự liên 8 tục của các hàm sau:
< sin2 x
(i) f (x) = 2
; x 6= 0;
: 1;x x = 0;
2
sin (1=x); x 6= 0;
(ii) f (x) =
1; x = 0;
Chương 5. HÀM SỐ THỰC LIÊN TỤC 64

8
< cos2 (1=x); x ;
(iii) f (x) = 1 + (x 2
) sin x; <x<2 ;
:
sin2 (1=x); x 2 ;
x2 + 1; x 2 Q;
(iv) f (x) =
x2 ; x 2 R r Q:
15. Cho f : [a; b] ! R liên tục trên [a; b] sao cho f (x) > x2 ; 8x 2 [a; b]: Chứng minh rằng tồn tại số
thực m > 0 sao cho f (x) > x2 + m; 8x 2 [a; b]:
16. Cho f : [a; b] ! R liên tục trên [a; b] sao cho f (x) < x3 ; 8x 2 [a; b]: Chứng minh rằng tồn tại số
thực M > 0 sao cho f (x) < x3 M; 8x 2 [a; b]:
17. Cho hai hàm f; g : [a; b] ! R liên tục trên [a; b] sao cho f (x) > g(x); 8x 2 [a; b]: Chứng minh
rằng tồn tại số thực m > 0 sao cho f (x) > g(x) + m; 8x 2 [a; b]:
18. Cho hai hàm f; g : [a; b] ! R liên tục trên [a; b] sao cho f (x) < g(x); 8x 2 [a; b]: Chứng minh
rằng tồn tại số thực M > 0 sao cho f (x) < g(x) M; 8x 2 [a; b]:
19. Cho hàm số f : R ! R thỏa điều kiện f (x + y) = f (x) + f (y); 8x; y 2 R: Chứng minh rằng:
(i) f (0) = 0;
(ii) f là hàm lẻ;
(iii) f (nx) = nf (x); 8x 2 R; 8n 2 N;
(iv) f (nx) = nf (x); 8x 2 R; 8n 2 Z;
(v) f (x) = f (1)x; 8x 2 Q;
(vi) Nếu f liên tục trên R thì f (x) = f (1)x; 8x 2 R:
20. Cho f : [a; b] ! [a; b] thỏa điều kiện jf (x) f (y)j < jx yj ; 8x; y 2 [a; b]; x 6= y:
Chứng minh rằng tồn tại duy nhất x 2 [a; b] sao cho x = f (x ):
21. Cho f : [a; b] ! [a; b] liên tục trên [a; b]: Chứng minh rằng tồn tại x 2 [a; b] sao cho x = f (x ):
22. Cho f : [a; b] ! [a; b] liên tục trên [a; b]: Chứng minh rằng tồn tại x0 2 [a; b] sao cho x0 =
(x0 a)(x0 b) + f (x0 ):
a b
23. Cho hai số thực dương a; b: Chứng minh rằng phương trình 3 + 3 2
= 0 có
x +x 2 x +x +x+1
nghiệm trong khoảng ( 1; 1): Nghiệm này có duy nhất không?
p
24. Cho f (x) = 3 x: Chứng minh rằng f liên tục đều trên R:
25. Cho f (x) = sin x: Chứng minh rằng f liên tục đều trên R:
26. Cho f (x) = x 3
8 : Chứng minh rằng f không liên tục đều trên R:
< 1; x < 1;
27. Cho f (x) = cos( x); 1 x < 2;
:
1; x 2:
Chứng minh rằng 8 f liên tục đều trên R:
< 0; x < 0;
28. Cho f (x) = x2 ; 0 x 1;
:
1; x > 1:
Chứng minh rằng f liên tục đều trên R:
29. Cho hàm f : (0; 1) ! R liên tục đều trên (0; 1) và fxn g (0; 1) là một dãy Cauchy. Chứng minh
rằng ff (xn )g là một dãy Cauchy.
30. Cho hàm f : R ! R liên tục trên R và fxn g là một dãy Cauchy. Chứng minh rằng ff (xn )g là
một dãy Cauchy.
31. Cho ví dụ về hàm f liên tục đều trên D R và fxn g D không là một dãy Cauchy mà ff (xn )g
là một dãy Cauchy.
32. Cho ví dụ về hai hàm f và g liên tục đều trên D R mà f g không liên tục đều trên D:
33. Hãy chỉ ra hai hàm f; g : R ! R sao cho f 6= 0; g 6= 0 mà f g 0:
34. Hãy chỉ ra hai hàm f; g : R ! R sao cho:
(i) f liên tục tại mọi điểm thuộc R;
(ii) g liên tục tại mọi điểm thuộc R r f0g và không liên tục tại x = 0;
(iii) f g liên tục tại mọi điểm thuộc R:
35. Hãy chỉ ra hai song ánh f; g : R ! R sao cho f g : R ! R không song ánh.
36. Hãy chỉ ra hai đơn ánh f; g : R ! R sao cho f g : R ! R không đơn ánh.
37. Hãy chỉ ra hai toàn ánh f; g : R ! R sao cho f g : R ! R không toàn ánh.
Chương 5. HÀM SỐ THỰC LIÊN TỤC 65

38. Cho hàm f : R ! R liên tục đều trên R: Chứng minh rằng 9a; b > 0 : jf (x)j a jxj + b; 8x 2 R:
38a. Cho f (x) = x sin x:
(i) Chứng minh rằng 9a; b > 0 : jf (x)j a jxj + b; 8x 2 R:
(ii) Chứng minh rằng f không liên tục đều trên R:
39. Cho A 6= và g : A ! Q là một đơn ánh. Chứng minh rằng A là tập quá lắm đếm được.
40. Cho hàm f : (0; 1) ! R là tăng. Chứng minh rằng tập A = fx 2 (0; 1) : f không liên tục tại x:
Chứng minh rằng A là tập quá lắm đếm được.
Hướng dẫn bài 38 :
(i) Ý nghĩa hình học: Ta có jf (x)j a jxj + b; 8x 2 R () (a jxj + b) f (x) a jxj + b; 8x 2 R:
Điều này nghĩa là đồ thị (C) của hàm liên tục đều trên R nằm dưới đồ thị của hàm g1 (x) = a jxj + b và
nằm trên đồ thị của hàm g2 (x) = a jxj b: Chi tiết hơn phần đồ thị (C) ở phía phải của trục Oy nằm
giữa (bị kẹp) hai nửa đường thẳng y = ax + b; y = ax b; và phần đồ thị (C) ở phía trái của trục Oy
nằm giữa (bị kẹp) hai nửa đường thẳng y = ax + b; y = ax b (Xem hình)

Tuy nhiên với bài tập 38a sẽ cho thấy hàm số có đồ thị bị kẹp giữa hai đường thẳng như trên chưa
chắc là hàm số liên tục đều.
(ii) Hướng dẫn chứng minh:
(j) Do f liên tục đều trên R; tồn tại > 0 : jf (x) f (y)j < 1; 8x; y 2 R và jx yj < :
1 1
Chọn n 2 N : < ; ta cũng có jf (x) f (y)j < 1; 8x; y 2 R và jx yj :
n n
(jj) 8x; y 2 R và jx yj 2: Hãy chứng minh rằng jf (x) f (y)j < 2n:
y x
HD: Giả sử x < y; xi = x + i ; i = 0; 1; ; 2n; ta có
2n

jf (x) f (y)j = jf (x0 ) f (x2n )j


jf (x0 ) f (x1 )j + jf (x1 ) f (x2 )j + + jf (x2n 1) f (x2n )j 2n:

(jjj) Chứng minh rằng jf (x)j 2nx + 2n + jf (0)j ; 8x > 0:


Chương 5. HÀM SỐ THỰC LIÊN TỤC 66

HD: Đặt N = [x] là số nguyên lớn nhất và x (là phần nguyên của x), fxg = x [x] = x N là
phần lẻ của x; 0 fxg < 1.

jf (x)j jf (x) f (N )j + jf (N ) f (N 1)j + + jf (1) f (0)j + jf (0)j


2n + 2n + + 2n + jf (0)j
= (N + 1)(2n) + jf (0)j
(x + 1)(2n) + jf (0)j = 2n jxj + 2n + jf (0)j :

(4j) Chứng minh rằng jf (x)j 2nx + 2n + jf (0)j ; 8x 2 R:


- Hiển nhiên bất đẳng thức đúng với x = 0:
- Ta chỉ cần chứng minh với x < 0; N1 = [x] 1; fxg = x [x] = x N1 2 [0; 1);

jf (x)j jf (x) f (N1 )j + jf (N1 ) f (N1 + 1)j + jf (N1 + 1) f (N1 + 2)j + + jf ( 1) f (0)j + jf (0)j
2n + 2n + + 2n + jf (0)j = ( N1 + 1)(2n) + jf (0)j
= (fxg x + 1)(2n) + jf (0)j
( x + 1)(2n) + jf (0)j = (jxj + 1)(2n) + jf (0)j
= 2n jxj + 2n + jf (0)j :

Hướng dẫn bài 38a:


(i) Chứng minh rằng 9a; b > 0 : jf (x)j a jxj + b; 8x 2 R:
Dễ dàng jf (x)j = jx sin xj jxj = a jxj + b; 8x 2 R; a = 1; b = 0:
(ii) Chứng minh rằng f không liên tục đều trên R: Ý tưởng chọn xn = yn + n; yn = 2n + ;
2
1
n = p 3
:
n

f (xn ) f (yn ) = (yn + n ) sin (yn + n) yn sin yn


= 2n + + n cos n 2n +
2 2
= 2n + (cos n 1) + n cos n
2
= 2n + (1 cos n) + n cos n ;
2
jf (xn ) f (yn )j 2n + (1 cos n) n cos n
2
2 1 cos n
2n + n 2 n
2 n

2n + 1 cos
= p 2 n
zn :
3 2 n
n2 n

1 cos1 n
Mặt khác, 2! ; nên zn ! +1; do đó tồn tại n0 2 N sao cho zn 1; 8n n0 :
n 2
1
Cho > 0; chọn n1 2 N : n = p 3
< ; 8n n1 :
n
Lấy n maxfn0 ; n1 g; ta có jxn yn j = n < và jf (xn ) f (yn )j zn 1: Vậy f không liên tục
đều trên R:
Hướng dẫn bài 39 :
Do g : A ! Q là một đơn ánh, ta có g : A ! g(A) là một song ánh, ta có ánh xạ ngược g 1 : g(A) ! A
cũng song ánh
Ta có g(A) Q; nên g(A) hữu hạn hoặc vô hạn đếm được (đếm được).
Nếu g(A) hữu hạn thì A = g 1 (g(A)), cũng hữu hạn.
Nếu g(A) đếm được thì A = g 1 (g(A)), cũng đếm được.
Vậy, A là tập quá lắm đếm được.
Hướng dẫn bài 40 :
Chương 5. HÀM SỐ THỰC LIÊN TỤC 67

(i) Cho x 2 (0; 1):


- Tập f ((0; x)) = ff (t) : 0 < t < xg 6= và bị chận trên (bởi f (x1 ); với x < x1 < 1). Do đó
9m(x) = sup f ((0; x)):
- Tập f ((x; 1)) = ff (t) : x < t < 1g 6= và bị chận dưới (bởi f (x2 ); với 0 < x2 < x). Do đó
9M (x) = inf f ((x; 1)):
Hãy chứng minh rằng
(ii) m(x) f (x) M (x);
(iii) x 2 A () m(x) < M (x):
(iv) Với x 2 A; chọn g(x) 2 Q \ (m(x); M (x)) : Chứng minh rằng g : A ! Q là một đơn ánh, và suy
ra A là tập quá lắm đếm được.
Hướng dẫn (ii): Chứng minh m(x) f (x):
Giả sử m(x) > f (x). Với " = m(x) f (x); và do m(x) = sup f ((0; x)); nên 9x 2 (0; x) : f (x ) >
m(x) " = f (x): Điều này mâu thuẫn vì f là hàm tăng. Vậy m(x) f (x):
Hướng dẫn (ii): Chứng minh f (x) M (x):
Giả sử f (x) > M (x). Với " = f (x) M (x); và do M (x) = inf f ((x; 1)); nên 9x 2 (x; 1) : f (x ) <
M (x) + " = f (x): Điều này mâu thuẫn vì f là hàm tăng. Vậy f (x) M (x):
Hướng dẫn (iii): Ta có m(x) M (x); 8x 2 (0; 1): Như vậy, hoặc là m(x) = M (x) hoặc là m(x) <
M (x): Ta sẽ chứng minh rằng
x2= A () m(x) = M (x):
Chứng minh (iii). Trước hết ta chứng minh rằng:
(a) f (x) lim f (t) = sup f ((0; x)) = m(x):
t!x
(b) f+ (x) lim f (t) = inf f ((x; 1)) = M (x):
t!x+
Chứng minh (a) Thật vậy, cho " > 0; 9x0 2 (0; x) : f (x0 ) > sup f ((0; x)) ":
8t 2 (x0 ; x); do f là hàm tăng, ta có sup f ((0; x)) " < f (x0 ) < f (t) sup f ((0; x)) < sup f ((0; x))+";
điều này dẫn tới jf (t) sup f ((0; x))j < ":
Vậy, f (x) lim f (t) = sup f ((0; x)) = m(x):
t!x
Chứng minh (b) cho " > 0; 9x0 2 (x; 1) : f (x0 ) < inf f ((x; 1)) + ":
8t 2 (x; x0 ); do f là hàm tăng, ta có inf f ((x; 1)) " < inf f ((x; 1)) f (t) < f (x0 ) < inf f ((x; 1)) + ";
điều này dẫn tới jf (t) inf f ((x; 1))j < ":
Vậy, f+ (x) lim f (t) = inf f ((x; 1)) = M (x):
t!x+
Chứng minh (iii)=): Giả sử f liên tục tại x; ta có lim f (t) = f (x) và lim f (t) = f (x); tức là
t!x+ t!x
M (x) = f (x) và m(x) = f (x). Điều này dẫn tới M (x) = m(x):
Chứng minh (iii)(=: Giả sử m(x) = M (x); ta có m(x) = M (x) = f (x); từ đó lim f (t) = lim
t!x+ t!x
f (t) = f (x): Vậy, f liên tục tại x:
Chứng minh (iv). Cho x 2 A; ta có m(x) < M (x); ta suy ra Q \ (m(x); M (x)) 6= : Tồn tại
g(x) 2 Q \ (m(x); M (x)) :
Ta có ánh xạ g : A ! Q: Ta cần chứng minh x; x0 2 (0; 1); x < x0 =) g(x) < g(x0 ):
Giả sử z = g(x) = g(x0 ) 2 Q \ (m(x); M (x)) \ (m(x0 ); M (x0 )) 6= ; khi đó m(x) < z < M (x);
m(x0 ) < z < M (x0 ):

lim f (t) = m(x0 ) < z < M (x) = lim f (t)


t!x0 t!x+

Ta có, do f là hàm tăng, ta có

x < t < s < x0 ;


f (x) < f (t) < f (s) < f (x0 ):

Cho t ! x+ ; ta có
f (x) lim f (t) f (s) < f (x0 ):
t!x+
Chương 5. HÀM SỐ THỰC LIÊN TỤC 68

Cho s ! x0 ; ta có
f (x) lim f (t) lim f (s) f (x0 );
t!x+ s!x0

Điều này dẫn tới lim f (t) lim f (s); mà điều này mâu thuẫn vì lim0 f (t) < z < lim f (t):
t!x+ s!x0 t!x t!x+

Vậy g(x) < g(x0 ); do đó g : A ! Q là một đơn ánh. Theo Bài tập 39, ta có A là tập quá lắm đếm
được.
Chương 6

ĐẠO HÀM

6.1 Đạo hàm và các tính chất cơ bản của đạo hàm
Định nghĩa 1. Cho f : (a; b) ! R và x 2 (a; b): Ta xét hàm số

: (a; b)nfxg ! R
f (y) f (x)
y 7 ! (y) = :
y x

Khi đó, x là điểm tụ của tập (a; b)nfxg: Ta nói f có đạo hàm tại x nếu tồn tại lim (y) = L 2 R và
y!x
số thực L này được gọi là đạo hàm của f tại x: Ta ký hiệu L là f 0 (x):
Như vậy
f (y) f (x) f (x + h) f (x)
f 0 (x) = lim = lim :
y!x y x h!0 h
Cũng một cách khác để định nghĩa đạo hàm của f tại x như sau: Đặt r = minfx a; b xg; ta có
r > 0 và (x r; x + r) (a; b): Khi đó, ta định nghĩa hàm

: ( r; r)nf0g ! R
f (x + h) f (x)
h 7 ! (h) = :
h
Chú ý rằng x + h 2 (a; b); do đó f (x + h) được xác định. Ta nói f có đạo hàm tại x nếu tồn tại
lim (h) = L 2 R và số thực L này cũng được gọi là đạo hàm của f tại x: Như vậy
h!0

f (x + h) f (x) f (y) f (x)


f 0 (x) = lim = lim :
h!0 h y!x y x

Định nghĩa 2. Cho f : (a; b) ! R và x 2 (a; b): Ta nói f khả vi tại x nếu tồn tại A 2 R và một hàm
' : ( r; r) ! R sao cho
f (x + h) = f (x) + Ah + jhj '(h); 8h 2 ( r; r);
trong đó, r > 0 đủ bé sao cho (x r; x + r) (a; b) và lim '(h) = 0:
h!0
Biểu thức Ah gọi là vi phân của f tại x ứng với h, ký hiệu nó là df (x; h). Vậy

df (x; h) = Ah:

Định lý 1. f khả vi tại x () f có đạo hàm tại x:


Khi đó, df (x; h) = f 0 (x)h:
Chứng minh Định lý 1.
Chứng minh =). Giả sử f khả vi tại x. Khi đó,

f (x + h) = f (x) + Ah + jhj '(h); 8h 2 ( r; r);

69
Chương 6. ĐẠO HÀM 70

với '(h) như trên. Ta có

f (x + h) f (x) jhj
=A+ '(h); 8h 2 ( r; r)nf0g;
h h
jhj f (x + h) f (x)
mà '(h) = '(h) ! 0 khi h ! 0: Vậy ! A khi h ! 0: Do đó f có đạo hàm tại x và
h h
0
f (x) = A:
Chứng minh (=. Giả sử f có đạo hàm tại x. Khi đó,

f (x + h) f (x)
'
~ (h) = f 0 (x) ! 0; khi h ! 0:
h
Khi đó,

f (x + h) = f (x) + f 0 (x)h + h~
'(h) = f (x) + f 0 (x)h + jhj '(h); 8h 2 ( r; r);

trong đó, r > 0 đủ bé sao cho (x r; x + r)(a; b) và


8
< '~ (h); h > 0;
'(h) = 0; h = 0;
:
'~ (h); h < 0;

và lim '(h) = lim '


~ (h) = 0: Vậy f khả vi tại x. Định lý 1 được chứng minh xong.
h!0 h!0

Định lý 2. Nếu f có đạo hàm tại x thì hàm f liên tục tại x.
Chứng minh Định lý 2. Do f có đạo hàm tại x ta có

f (x + h) f (x)
'
~ (h) = f 0 (x) ! 0; khi h ! 0:
h
Do đó
f (x + h) f (x) = f 0 (x)h + h~
'(h) ! 0; khi h ! 0:
Vậy f liên tục tại x. Định lý 2 được chứng minh xong.
Chú thích. Phần đảo không đúng, ví dụ hàm f (x) = jxj liên tục tại x = 0. Nhưng f không có đạo
hàm tại x = 0; bởi @f 0 (0):
Ví dụ 1: (Xem như Bài tập). Cho f (x) = C; 8x 2 R (hàm hằng). Ta có f 0 (x) = 0; 8x 2 R:
Định nghĩa 3. Cho f : (a; b) ! R và x 2 (a; b): Ta nói f có đạo hàm bên phải tại x nếu tồn tại
f (y) f (x)
lim = L+ 2 R và số thực L+ này được gọi là đạo hàm bên phải của f tại x: Ta ký hiệu L+
y!x+ y x
là f+0 (x): Như vậy
f (y) f (x) f (x + h) f (x)
f+0 (x) = lim = lim :
y!x+ y x h!0+ h
f (y) f (x)
Tương tự Định nghĩa f có đạo hàm bên trái tại x nếu tồn tại lim 2 R và ký hiệu đạo
y!x y x
hàm bên trái của f tại x như sau
f (y) f (x) f (x + h) f (x)
f 0 (x) = lim = lim :
y!x y x h!0 h

Định lý 3.
8
< (i) f có đạo hàm bên phải f+0 (x) tại x
f có đạo hàm tại x () (ii) f có đạo hàm bên trái f 0 (x) tại x
:
(iii) f+0 (x) = f 0 (x):

Hơn nữa f 0 (x) = f+0 (x) = f 0 (x):


Chương 6. ĐẠO HÀM 71

x2 + 1; x < 0;
Ví dụ 2. Tính đạo hàm của hàm số sau f (x) =
x3 ; x 0:
Giải.
(i) Với x < 0 : f (x) = x2 + 1 liên tục và có đạo hàm tại mọi x < 0; với f (x) = 2x; x < 0:
(ii) Với x > 0 : f (x) = x3 liên tục và có đạo hàm tại mọi x > 0; với f (x) = 3x2 ; x > 0:
(iii) Tại x = 0 : f (0) = 03 = 0:
f+ (0) = lim f (x) = lim x3 = 0 = f (0) : f liên tục bên phải tại x = 0;
x!0+ x!0+
f (0) = lim f (x) = lim x2 + 1 = 1 6= 0 = f (0) : f không liên tục bên trái tại x = 0:
x!0 x!0
Vậy, f không liên tục tại x = 0; do đó theo Định lý 2 thì f không có đạo hàm tại x = 0:
2x; x < 0;
Cuối cùng, ta có f 0 (x) = 2 và @ f 0 (0):
3x ; x > 0;
x2 + x; x 0;
Ví dụ 3. Tính đạo hàm của hàm số sau f (x) =
x3 + x; x > 0:
Giải.
(i) Với x < 0 : f (x) = x2 + x có đạo hàm tại mọi x < 0; với f (x) = 2x + 1; x < 0:
(ii) Với x > 0 : f (x) = x3 + x có đạo hàm tại mọi x > 0; với f (x) = 3x2 + 1; x > 0:
(iii) Tại x = 0 : f (0) = 02 + 0 = 0:
f (x) f (0) x3 + x 0
lim = lim = lim x2 + 1 = 1 : f có đạo hàm bên phải tại x = 0; và
x!0+ x 0 x!0+ x 0 x!0+
f+0 (0) = 1;
f (x) f (0) x2 + x 0
lim = lim = lim (x + 1) = 1 : f có đạo hàm bên trái tại x = 0; và
x!0 x 0 x!0 x 0 x!0
0
f (0) = 1;
Mà f+0 (0) = f10 (0) = 1; nên có đạo hàm tại x = 0; và f 08 (0) = 1:
< 2x + 1; x < 0;
0 2x + 1; x 0;
Cuối cùng, f có đạo hàm tại mọi x 2 R và ta có f (x) = 1; x = 0; = 2 + 1;
: 3x x > 0:
3x2 + 1; x > 0
Chú thích về ý nghĩa của đạo hàm
(i) Ý nghĩa cơ học của đạo hàm: Vận tốc chuyển động của chất điểm.
(ii) Ý nghĩa hình học của đạo hàm: Hệ số góc của tiếp tuyến với đường cong.
Xét đường cong (L) là đồ thị của hàm số y = f (x) và một điểm cố định M0 (x0 ; f (x0 )) 2 (L): Xét cát
tuyến M0 M; với M 2 (L): Nếu khi điểm M chạy trên đường cong (L) tới điểm M0 mà cát tuyến M0 M
tiến dần đến một vị trí giới hạn M0 T thì đường thẳng M0 T được gọi là tiếp tuyến của đường (L) tại M0 :
Vấn đề đặt ra là khi nào đường (L) có tiếp tuyến tại M0 và thì hệ số góc của tiếp tuyến ấy được tính
như thế nào? Giả sử M (x0 + h; f (x0 + h)) 2 (L): Hệ số góc của cát tuyến M0 M là

yM yM0 f (x0 + h) f (x0 )


tg = tg(Ox; M0 M ) = = :
xM xM0 h

f (x0 + h) f (x0 )
Bây giờ cho h ! 0; điểm M chạy trên đường (L) tới điểm M0 ; lúc đó nếu tồn tại lim =
h!0 h
f 0 (x0 ) thì tg ở vế trái cũng có cùng giới hạn f 0 (x0 ), do đó góc tiến tới một góc xác định ; nghĩa là
cát tuyến M0 M dần đến một vị trí giới hạn M0 T nghiêng với trục Ox một góc : Vậy hệ số góc tg của
tiếp tuyến M0 T nếu có chính là

f (x0 + h) f (x0 )
tg = lim = f 0 (x0 )
h!0 h

(như hình vẽ)

Suy ra ý nghĩa hình học của đạo hàm như sau:


Chương 6. ĐẠO HÀM 72

Nếu hàm f có đạo hàm tại x0 thì đồ thị của hàm y = f (x) có tiếp tuyến tại M0 (x0 ; f (x0 )) và hệ số
góc của tiếp tuyến là
tg = f 0 (x0 ):
Do đó phương trình của tiếp tuyến với đồ thị tại M0 là

y f (x0 ) = f 0 (x0 )(x x0 );

và nếu f 0 (x0 ) 6= 0; phương trình của pháp tuyến với đồ thị tại M0 (đường thẳng vuông góc với tiếp tuyến
với đồ thị tại M0 ) là
1
y f (x0 ) = 0 (x x0 ):
f (x0 )
Định lý 4. Cho f; g có đạo hàm tại x. Khi đó, các hàm f + g; f g có đạo hàm tại x và
(i) (f + g)0 (x) = f 0 (x) + g 0 (x);
(ii) (f g)0 (x) = f 0 (x)g(x) + f (x)g 0 (x);
(iii) (Cf )0 (x) = Cf 0 (x); với C là hằng số thực.
0
f f f 0 (x)g(x) f (x)g 0 (x)
(iv) Nếu g(x) 6= 0; thì cũng có đạo hàm tại x và (x) = .
g g g 2 (x)
Chứng minh Định lý 4.
Chứng minh (i).

(f + g)(x + h) (f + g)(x) f (x + h) f (x) g(x + h) g(x)


= + ! f 0 (x) + g 0 (x)
h h h
khi h ! 0: Vậy (i) đúng.
Chứng minh (ii). Do g có đạo hàm tại x nên g liên tục tại x; do đó g(x + h) ! g(x) khi h ! 0:
Do vậy

(f g)(x + h) (f g)(x) f (x + h) f (x) g(x + h) g(x)


= g(x + h) + f (x) ! f 0 (x)g(x) + f (x)g 0 (x)
h h h
khi h ! 0: Do đó (ii) đúng.
Chứng minh (iii). Lấy g(x) = C = hàm hằng, ta có g 0 (x) = 0: Áp dụng (ii), ta có (iii) đúng.
1 0 1 0 g 0 (x)
Chứng minh (iv). Trước hết, ta chứng minh rằng có đạo hàm tại x và (x) = 2 :
g g g (x)
Ta có
1 1 g(x + h) g(x)
(x + h) (x)
g g h g 0 (x)
= ! 2 ;
h g(x + h)g(x) g (x)
0
1 g 0 (x)
khi h ! 0: Vậy (x) = .
g g 2 (x)
1
Áp dụng (ii), với tích hai hàm f ; ta có
g
0
f 1 0 1 1 0
(x) = f (x) = f 0 (x) (x) + f (x) (x)
g g g g
f 0 (x) g 0 (x) f 0 (x)g(x) f (x)g 0 (x)
= + f (x) 2
= :
g(x) g (x) g 2 (x)

Vậy (iv) đúng.


Định lý 4 được chứng minh xong.
Định lý 5. Cho f : (a; b) ! (c; d) có đạo hàm tại x 2 (a; b) và g : (c; d) ! R có đạo hàm tại y = f (x).
Khi đó g f có đạo hàm tại x và (g f )0 (x) = g 0 (f (x))f 0 (x):
Chương 6. ĐẠO HÀM 73

Chứng minh Định lý 5. Chọn R > 0 đủ bé sao cho (f (x) R; f (x) + R) (c; d): Xét hàm ~" :
( R; R) ! R cho bởi
8
< g(f (x) + k) g(f (x))
g 0 (f (x)); k 2 ( R; R); k 6= 0;
~"(k) = k
: 0; k = 0:

Do g có đạo hàm tại f (x); nên ~" liên tục tại k = 0:


Mặt khác, do f có đạo hàm tại x; nên f liên tục tại x; và do k = f (x + h) f (x) ! 0; khi h ! 0:
Ta suy ra, tồn tại r > 0 sao cho (x r; x + r) (a; b) và k = f (x + h) f (x) 2 ( R; R); 8h 2 ( r; r):
f (x + h) f (x)
Hơn nữa, do lim = f 0 (x); cho h ! 0; ta suy ra
h!0 h
g(f (x + h)) g(f (x)) g(f (x) + k) g(f (x))
=
h h
f (x + h) f (x)
= g 0 (f (x)) + ~"(k) ! g 0 (f (x));
h
g(f (x + h)) g(f (x))
do đó tồn tại lim và = g 0 (f (x))f 0 (x); tức là g f có đạo hàm tại x và (g f )0 (x) =
h!0 h
g 0 (f (x))f 0 (x): Vậy Định lý 5 được chứng minh xong.
Định lý 6. (Định lý đạo hàm của ánh xạ ngược). Cho f : (a; b) ! (c; d) là một song ánh, liên tục
trên (a; b). Cho f có đạo hàm tại x 2 (a; b) và f 0 (x) 6= 0:
1 1
Khi đó, hàm ngược f 1 cũng có đạo hàm tại y = f (x) và (f 1 )0 (y) = 0 = 0 :
f (x) f (f 1 (y))
Chứng minh Định lý 6. Ta có x = (f 1 )(y); z = (f 1 )(y + k); do đó y = f (x); y + k = f (z): Do f 1

liên tục tại y; do đó z = (f 1 )(y + k) ! (f 1 )(y) = x; khi k ! 0; do đó


(f 1 )(y + k) (f 1 )(y) z x 1 1 1
= = ! 0 = 0 ;
k f (z) f (x) f (z) f (x) f (x) f (f 1 (y))
z x
khi k ! 0. Vậy Định lý 6 được chứng minh.
Định nghĩa 4. Cho f : (a; b) ! R và x0 2 (a; b): Ta nói
(i) f đạt cực tiểu tại x0 nếu tồn tại một khoảng J; x0 2 J (a; b) : f (x) f (x0 ); 8x 2 J;
Ta còn nói x0 là điểm cực tiểu của hàm f .
(ii) f đạt cực đại tại x0 nếu tồn tại một khoảng J; x0 2 J (a; b) : f (x) f (x0 ); 8x 2 J;
Ta còn nói x0 là điểm cực đại của hàm f .
Điểm cực tiểu và điểm cực đại gọi chung điểm cực trị.
Định lý 7. (Định lý Fermat) Cho f : (a; b) ! R đạt cực trị tại x0 2 (a; b): Nếu f có đạo hàm tại
x0 thì f 0 (x0 ) = 0:
Chứng minh Định lý 7. Giả sử f : (a; b) ! R đạt cực tiểu tại x0 2 (a; b): Khi đó, tồn tại một khoảng
J; x0 2 J = (c; d) (a; b) sao cho f (x) f (x0 ); 8x 2 J:
f (x) f (x0 ) f (x) f (x0 )
Với x0 < x < d : 0; do đó f 0 (x0 ) = f+0 (x0 ) = lim 0:
x x0 +
x!x0 x x0
f (x) f (x0 ) f (x) f (x0 )
Với c < x < x0 : 0; do đó f 0 (x0 ) = f 0 (x0 ) = lim 0:
x x0 x!x0 x x0
0
Vậy f (x0 ) = 0:
Tương tự cho f đạt cực đại.
Định lý 7 được chứng minh xong.
Chú ý: (Định lý Fermat ở dạng khác). Định lý Fermat cho một điều kiện cần để một hàm đạt
cực trị được phát biểu theo một dạng khác như sau:
Định lý Fermat. Cho f : (a; b) ! R đạt cực trị tại x0 2 (a; b): Khi đó f 0 (x0 ) = 0 hoặc không tồn
tại f 0 (x0 ):
Chương 6. ĐẠO HÀM 74

Ví dụ. Hàm f (x) = jxj đạt cực tiểu tại x0 = 0, mà không tồn tại f 0 (0):
Định nghĩa 5. Cho f : (a; b) ! R: Ta nói f có đạo hàm trong khoảng (a; b) nếu f có đạo hàm tại
mọi điểm trong khoảng (a; b):
Định nghĩa 6. Cho f : [a; b] ! R: Ta nói f có đạo hàm trên đoạn [a; b] nếu f có đạo hàm trong
khoảng (a; b); có đạo hàm bên phải tại a và bên trái tại b:
Định lý 8. (Định lý Rolle) Cho f : [a; b] ! R liên tục trên [a; b] và có đạo hàm trong khoảng (a; b)
sao cho f (a) = f (b). Khi đó, tồn tại c 2 (a; b) sao cho f 0 (c) = 0:
Chứng minh Định lý 8.
Trong trường hợp f là hàm hằng. Khi đó f 0 (x) = 0; 8x 2 (a; b): Định lý luôn luôn đúng.
Xét trường hợp f không là hàm hằng. Khi đó 9t 2 (a; b) : f (t) 6= f (a) = f (b).
Do f : [a; b] ! R liên tục trên [a; b]; nên nó đạt giá trị lớn nhất và giá trị nhỏ nhất trên đoạn [a; b];
tức là
9c; d 2 [a; b] : f (c) f (x) f (d) 8x 2 [a; b].
nghĩa là 9c; d 2 [a; b] :

f (c) = min f ([a; b]) = minff (x) : x 2 [a; b]g;


f (d) = max f ([a; b]) = maxff (x) : x 2 [a; b]g:

Do f không là hàm hằng, nên f (c) < f (d) và c 6= d:


Nếu c hoặc d trong (a; b) thì nhờ Định lý Fermat, ta có Định lý đúng.
Nếu c và d 2 = (a; b) thì c; d 2 fa; bg; mà c 6= d nên c = a; d = b hay c = b; d = a mà điều này không
xảy ra vì f (a) = f (b) và f (c) < f (d): Vậy ta có Định lý đúng.
Định lý 8 được chứng minh xong.
Về ý nghĩa hình học của Định lý Rolle. Từ ý nghĩa hình học của đạo hàm thì f 0 (c) = 0; có
nghĩa là tiếp tuyến của đồ thị tại C(c; f (c)) là song song với trục Ox (nằm ngang). Như vậy, trên đồ thị
của hàm f liên tục trên [a; b] và có đạo hàm trong khoảng (a; b) sao cho f (a) = f (b); đều có ít nhất một
điểm C(c; f (c)) mà tại đó tiếp tuyến với đồ thị là song song với trục Ox (như hình vẽ).

Định lý 9 (Định lý Lagrange, Định lý giá trị trung bình) Cho f : [a; b] ! R liên tục trên [a; b]
và có đạo hàm trong khoảng (a; b). Khi đó, tồn tại c 2 (a; b) sao cho

f (b) f (a) = (b a)f 0 (c).

f (b) f (a)
Chứng minh Định lý 9. Xét hàm h(x) = f (x) f (a) (x a): Ta có h : [a; b] ! R liên
b a
tục trên [a; b] và có đạo hàm trong khoảng (a; b) sao cho h(a) = h(b) = 0. Do đó, tồn tại c 2 (a; b) sao
cho
h0 (c) = 0;
tức là
f (b) f (a)
h0 (c) = f 0 (c) = 0;
b a
f (b) f (a)
bởi vì h0 (x) = f 0 (x) :
b a
Định lý 9 được chứng minh xong.
Về ý nghĩa hình học của Định lý Lagrange. Với A(a; f (a)); B(b; f (b)), là hai điểm trên đồ thị
cũng là hai đầu dây cung AB, công thức trong Định lý Lagrange được viết lại

f (b) f (a)
Hệ số góc của dây cung AB = = f 0 (c):
b a
Chương 6. ĐẠO HÀM 75

Như vậy, trên đồ thị của hàm f liên tục trên [a; b] và có đạo hàm trong khoảng (a; b); đều có ít nhất
một điểm C(c; f (c)) mà tại đó tiếp tuyến với đồ thị là song song với dây cung AB (như hình vẽ).

Định lý 10 (Định lý giá trị trung bình Cauchy) Cho f; g : [a; b] ! R liên tục trên [a; b] và có
đạo hàm trong khoảng (a; b) sao cho g 0 (x) 6= 0; 8x 2 (a; b): Khi đó, tồn tại c 2 (a; b) sao cho

f (b) f (a) f 0 (c)


= 0 .
g(b) g(a) g (c)

Chứng minh Định lý 10.


Nếu g(b) g(a) = 0; thì dùng Định lý Rolle, ta có c 2 (a; b) sao cho g 0 (c) = 0: Mâu thuẫn với điều
g 0 (x)
6= 0; 8x 2 (a; b): Vậy g(b) g(a) 6= 0:
f (b) f (a)
Xét hàm h(x) = f (x) f (a) (g(x) g(a)): Ta có h : [a; b] ! R liên tục trên [a; b] và có
g(b) g(a)
đạo hàm trong khoảng (a; b) sao cho h(a) = h(b) = 0. Do đó, tồn tại c 2 (a; b) sao cho

h0 (c) = 0;

tức là
f (b) f (a) 0
h0 (c) = f 0 (c) g (c) = 0;
g(b) g(a)
f (b) f (a) 0
bởi vì h0 (x) = f 0 (x) g (x):
g(b) g(a)
Định lý 10 được chứng minh xong.
Về ý nghĩa hình học của Định lý giá trị trung bình Cauchy. Với A1 (a; f (a)); B1 (b; f (b)), là
hai đầu dây cung A1 B1 của đồ thị hàm f và A2 (a; g(a)); B2 (b; g(b)), là hai đầu dây cung A2 B2 của đồ
thị hàm g và công thức trong Định lý giá trị trung bình Cauchy được viết lại

f (b) f (a)
Hệ số góc của dây cung A1 B1 b a f 0 (c)
= = 0 :
Hệ số góc của dây cung A2 B2 g(b) g(a) g (c)
b a

Như vậy, trên hai đồ thị của hàm f; g liên tục trên [a; b] và có đạo hàm trong khoảng (a; b); đều có ít
nhất hai điểm C1 (c; f (c)) và C2 (c; g(c)) lần lượt trên hai đồ thị (có cùng hoành độ c) mà hai tiếp tuyến
với hai đồ thị tương ứng tại hai điểm C1 (c; f (c)) và C2 (c; g(c)) có tỉ số hai hệ số góc bằng tỉ số hai hệ số
góc của hai dây cung tương ứng như công thức

Hệ số góc của tiếp tuyến C1 T1 Hệ số góc của dây cung A1 B1


= :
Hệ số góc của tiếp tuyến C2 T2 Hệ số góc của dây cung A2 B2

(như hình vẽ).

Định lý 11 (Qui tắc L’Hopital) Cho f; g : (a; b) ! R có đạo hàm trong khoảng (a; b) sao cho
g 0 (x) 6= 0; 8x 2 (a; b); trong đó, 1 a < b +1:
f 0 (x)
Giả sử tồn tại lim 0 và một trong hai trường hợp sau là đúng:
x!a+ g (x)

(i) lim f (x) = lim g(x) = 0;


x!a+ x!a+
(ii) lim g(x) = 1 hay lim g(x) = +1;
x!a+ x!a+

f (x) f (x) f 0 (x)


Khi đó, tồn tại lim và lim = lim 0 :
x!a+ g(x) x!a+ g(x) x!a+ g (x)
Chương 6. ĐẠO HÀM 76

Định lý vẫn đúng khi thay a+ bởi b ; 1; +1; x0 2 (a; b):


f 0 (x)
Chứng minh Định lý 11. Đặt L = lim :
x!a+ g 0 (x)

A1. Trường hợp: a 2 R; lim f (x) = lim g(x) = 0:


x!a+ x!a+
(i) Xét trường hợp: a 2 R; lim f (x) = lim g(x) = 0; 1 < L < +1:
x!a+ x!a+
f 0 (x) f 0 (x) "
Chọn " > 0: Do L = lim 0
; ta có c 2 (a; b); sao cho
0
L < ; ta có 8x 2 (a; c):
x!a+ g (x) g (x) 2
Với x; y 2 (a; c); x < y; theo Định lý 10 (Định lý giá trị trung bình Cauchy), ta có t 2 (x; y) sao cho

f (y) f (x) f 0 (t)


= 0 .
g(y) g(x) g (t)
Suy ra
f (y) f (x) f 0 (t) "
L = L < .
g(y) g(x) g 0 (t) 2
Do lim f (x) = lim g(x) = 0; cho x ! a+ ; ta có
x!a+ x!a+

f (y) "
L < "; 8y 2 (a; c).
g(y) 2
f (x)
Vậy lim = L:
x!a+ g(x)
(ii) Xét trường hợp: a 2 R; lim f (x) = lim g(x) = 0; L = +1:
x!a+ x!a+
f 0 (x)
Cho A 2 R; chọn r; sao cho A < r < L = +1: Do L = lim > r; ta có c 2 (a; b); sao cho
x!a+ g 0 (x)
f 0 (x)
> r; ta có 8x 2 (a; c):
g 0 (x)
Với x; y 2 (a; c); x < y; theo Định lý 10 (Định lý giá trị trung bình Cauchy), ta có t 2 (x; y) sao cho

f (y) f (x) f 0 (t)


= 0 > r > A; 8x; y 2 (a; c); x < y.
g(y) g(x) g (t)

Do lim f (x) = lim g(x) = 0; cho x ! a+ ; ta có


x!a+ x!a+

f (y)
r > A; 8y 2 (a; c).
g(y)
f (x)
Vậy lim = +1:
x!a+ g(x)
(iii) Xét trường hợp: a 2 R; lim f (x) = lim g(x) = 0; L = 1:
x!a+ x!a+
f 0 (x)
Cho A 2 R; chọn r; sao cho 1 = L < r < A: Do L = lim < r < A; ta có c 2 (a; b); sao cho
x!a+ g 0 (x)
f 0 (x)
< r; ta có 8x 2 (a; c):
g 0 (x)
Với x; y 2 (a; c); x < y; theo Định lý 10 (Định lý giá trị trung bình Cauchy), ta có t 2 (x; y) sao cho

f (y) f (x) f 0 (t)


= 0 < r < A; 8x; y 2 (a; c); x < y.
g(y) g(x) g (t)

Do lim f (x) = lim g(x) = 0; cho x ! a+ ; ta có


x!a+ x!a+

f (y)
r < A; 8y 2 (a; c).
g(y)
Chương 6. ĐẠO HÀM 77

f (x)
Vậy lim = 1:
x!a+ g(x)
A2. Trường hợp: a 2 R; lim g(x) = +1:
x!a+
(i) Xét trường hợp: a 2 R; lim g(x) = +1; 1 < L < +1:
x!a+
f 0 (x) f 0 (x) "
Chọn " > 0: Do L = lim ; ta có c 2 (a; b); sao cho
L < ; ta có 8x 2 (a; c):
x!a+ g 0 (x) 0
g (x) 2
Cho y0 2 (a; c); do lim g(x) = +1; ta có c1 2 (a; y0 ) : g(x) > jg(y0 )j ; 8x 2 (a; c1 ):
x!a+
Với 8x 2 (a; c1 ); theo Định lý 10 (Định lý giá trị trung bình Cauchy), ta có t 2 (x; y0 ) sao cho

f (y0 ) f (x) f 0 (t)


= 0 ;
g(y0 ) g(x) g (t)

hay
g(x) g(y0 ) f (y0 ) f (x) f 0 (t) g(x) g(y0 )
= 0 .
g(x) g(y0 ) g(x) g (t) g(x)
hay
f (y0 ) f (x) f 0 (t) g(y0 )
= 0 1 .
g(x) g (t) g(x)
hay
f (x) f 0 (t) g(y0 ) f (y0 ) f 0 (t) f 0 (t) f (y0 )
= 0 1 + = 0 + 1 .
g(x) g (t) g(x) g(x) g (t) g 0 (t) g(x)
hay

f (x) f 0 (t) f 0 (t) f (y0 )


L L + 1
g(x) g 0 (t) g 0 (t) g(x)
" " f (y0 )
+ 1+L+ .
2 2 g(x)

f (y0 ) " f (y0 ) "


Với y0 2 (a; c); như trên, do lim = 0; ta có c2 2 (a; c1 ) : 1 + L + < ; 8x 2 (a; c2 ):
x!a+ g(x) 2 g(x) 2
Do đó, ta có
f (x) " "
L < + = "; 8x 2 (a; c2 ):
g(x) 2 2
f (x)
Vậy lim = L:
x!a+ g(x)
(ii) Xét trường hợp: a 2 R; lim g(x) = +1; L = +1:
x!a+
f 0 (x)
Cho A > 0; chọn r; sao cho A < r < L = +1: Do L = lim > r; ta có c 2 (a; b); sao cho
x!a+ g 0 (x)
f 0 (x)
> 2r; ta có 8x 2 (a; c):
g 0 (x)
Cho y0 2 (a; c); do lim g(x) = +1; ta có c1 2 (a; y0 ) : g(x) > jg(y0 )j ; 8x 2 (a; c1 ):
x!a+
Với 8x 2 (a; c1 ); theo Định lý 10 (Định lý giá trị trung bình Cauchy), ta có t 2 (x; y0 ) sao cho

f (y0 ) f (x) f 0 (t)


= 0 ;
g(y0 ) g(x) g (t)

hay

f (x) f 0 (t) g(y0 ) f (y0 )


= 1 +
g(x) g 0 (t) g(x) g(x)
g(y0 ) f (y0 ) f (y0 ) 2rg(y0 )
> 2r 1 + = 2r + .
g(x) g(x) g(x)
Chương 6. ĐẠO HÀM 78

f (y0 ) 2rg(y0 ) f (y0 ) 2rg(y0 )


Với y0 2 (a; c); như trên, do lim = 0; ta có c2 2 (a; c1 ) : < r;
x!a+ g(x) g(x)
8x 2 (a; c2 ):
Do đó, ta có
f (x) f (y0 ) 2rg(y0 )
> 2r > r > A; 8x 2 (a; c2 ):
g(x) g(x)
f (x)
Vậy lim = +1:
x!a+ g(x)
(iii) Xét trường hợp: a 2 R; lim g(x) = +1; L = 1:
x!a+
f 0 (x)
Cho A > 0; chọn r; sao cho 1=L<r< A: Do L = lim = 1; ta có c 2 (a; b); sao cho
x!a+ g 0 (x)
f 0 (x)
< 2r; ta có 8x 2 (a; c):
g 0 (x)
Cho y0 2 (a; c); do lim g(x) = +1; ta có c1 2 (a; y0 ) : g(x) > jg(y0 )j ; 8x 2 (a; c1 ):
x!a+
Với 8x 2 (a; c1 ); theo Định lý 10 (Định lý giá trị trung bình Cauchy), ta có t 2 (x; y0 ) sao cho
f (y0 ) f (x) f 0 (t)
= 0 ;
g(y0 ) g(x) g (t)
hay
f (x) f 0 (t) g(y0 ) f (y0 )
= 1 +
g(x) g 0 (t) g(x) g(x)
g(y0 ) f (y0 ) f (y0 ) 2rg(y0 )
< 2r 1 + = 2r + .
g(x) g(x) g(x)

f (y0 ) 2rg(y0 ) f (y0 ) 2rg(y0 )


Với y0 2 (a; c); như trên, do lim = 0; ta có c2 2 (a; c1 ) : < jrj ;
x!a+ g(x) g(x)
8x 2 (a; c2 ):
Do đó, ta có
f (x) f (y0 ) 2rg(y0 )
< 2r + < 2r + jrj r< A; 8x 2 (a; c2 ):
g(x) g(x)
f (x)
Vậy lim = 1:
x!a+ g(x)
Chứng minh tương tự chúng ta cũng chứng minh được cho các trường hợp sau
A3. Trường hợp: a 2 R; lim g(x) = 1;
x!a+
và các trường hợp khi thay a+ bởi b ; 1; +1; x0 2 (a; b):
Định lý 11 được chứng minh xong.

6.2 Đạo hàm cấp cao


Định nghĩa 7. Cho f : (a; b) ! R có đạo hàm trong khoảng (a; b). Khi đó, hàm số

f 0 : (a; b) ! R
x 7 ! f 0 (x)

gọi là đạo hàm cấp một của f:


f 0 (y) f 0 (x)
Nếu tồn tại lim = f 00 (x) 2 R thì ta nói f có đạo hàm cấp hai tại x:
y!x y x
Giả sử f : (a; b) ! R có đạo hàm cấp hai tại mọi x 2 (a; b). Khi đó, hàm số

f 00 : (a; b) ! R
x 7 ! f 00 (x)
Chương 6. ĐẠO HÀM 79

gọi là đạo hàm cấp hai của f; tức là f 00 = (f 0 )0 :


Định nghĩa tương tự:
f 000 = (f 00 )0 = đạo hàm cấp ba của f;
f (4) = (f (3) )0 = đạo hàm cấp ba của f;
..
.
f (n) = (f (n 1) )0 = đạo hàm cấp n của f:
Nếu hàm số f (n) (đạo hàm cấp n của f ) liên tục trong khoảng (a; b), ta nói f thuộc lớp C n trong
khoảng (a; b). Ta ký hiệu f 2 C n (a; b):
Ta qui ước f (0) = f:
x2 + x; x 0;
Ví dụ 4. Tính đạo hàm cấp hai của hàm số sau f (x) =
x3 + x; x > 0:
Theo ví dụ trên ta có f có đạo hàm tại mọi x 2 R và ta có
8
< 2x + 1; x < 0;
2x + 1; x 0;
f 0 (x) = 1; x = 0; = 2 + 1;
: 3x x > 0:
3x2 + 1; x > 0

2; x < 0;
Dễ dàng tính được f 00 (x) =
6x; x > 0:
Ta kiểm tra lại không tồn tại f 00 (0):
Thật vậy, tại x = 0, f 0 (0) = 1:
f 0 (x) f 0 (0) 6x 1 1
lim = lim = lim 6 = 1 : f 0 không tồn tại có đạo hàm bên phải tại
x!0+ x 0 x!0+ x x!0+ x
f 0 (x) f 0 (0)
x = 0; dẫn đến không tồn tại f 00 (0) = lim :
x!0 x 0

6.3 Công thức khai triển Taylor hữu hạn


Định lý 11 (Công thức khai triển Taylor hữu hạn với phần dư Lagrange). Cho f : (a; b) ! R
có đạo hàm đến cấp n + 1 trong khoảng (a; b): Khi đó, với mọi x; x0 2 (a; b); ta có 2 (0; 1) sao cho
n
X f (k) (x0 ) f (n+1) (x0 + (x x0 ))
f (x) = (x x0 )k + (x x0 )n+1 :
k! (n + 1)!
k=0

Chú thích.
(i) minfx0 ; xg < c = x0 + (x x0 ) = x + (1 )x0 < maxfx0 ; xg:
f (n+1) (x0 + (x x0 ))
(ii) Số hạng Rn (x; x0 ; f ) = (x x0 )n+1 gọi là phần dư Lagrange trong công
(n + 1)!
thức khai triển Taylor hữu hạn.
X n
f (k) (x0 )
(iii) Đa thức Pn (x) = (x x0 )k gọi là đa thức Taylor.
k!
k=0
X n
f (k) (x0 )
Chứng minh Định lý 11. Đặt G(x) = (x x0 )n+1 và F (x) = f (x) (x x0 )k ; giả sử x0 < x,
k!
k=0
áp dụng Định lý giá trị trung bình Cauchy cho hai hàm F; G; trong khoảng (x0 ; x); ta có x1 2 (x0 ; x)
sao cho
F (x) F (x) F (x0 ) F 0 (x1 )
= = 0 .
G(x) G(x) G(x0 ) G (x1 )

Áp dụng Định lý giá trị trung bình Cauchy cho hai hàm F 0 ; G0 ; trong khoảng (x1 ; x); ta có x2 2 (x0 ; x1 )
sao cho
F (x) F 0 (x1 ) F 0 (x1 ) F 0 (x0 ) F 00 (x2 )
= 0 = 0 0
= 00 .
G(x) G (x1 ) G (x1 ) G (x0 ) G (x2 )
Chương 6. ĐẠO HÀM 80

Tiếp tục quá trình trên, vẫn áp dụng Định lý giá trị trung bình Cauchy cho hai hàm F (n) ; G(n) ; trong
khoảng (x0 ; xn ); ta có xn+1 2 (x0 ; xn ) sao cho

F (x) F (n) (xn ) F (n) (xn ) F (n) (x0 ) F (n+1) (xn+1 ) f (n+1) (xn+1 )
= = = = = .
G(x) G(n) (xn ) G(n) (xn ) G(n) (x0 ) G(n+1) (xn+1 ) (n + 1)!

Chú ý xn+1 2 (x0 ; xn ) (x0 ; xn 1 ) (x0 ; x); ta viết xn+1 = x0 + (x x0 ); 0 < < 1:
Định lý 11 được chứng minh xong.
Định lý 12 (Công thức khai triển Taylor hữu hạn với phần dư Peano). Cho f : (a; b) ! R
có đạo hàm đến cấp n 1 trong khoảng (a; b): Cho x0 2 (a; b) sao cho tồn tại f (n) (x0 ): Khi đó ta có
n
X f (k) (x0 )
f (x) = (x x0 )k + Rn (x); khi (x x0 ) đủ bé,
k!
k=0

Rn (x)
với lim = 0:
x!x0 (x x0 )n
Chú thích. Số hạng Rn (x) gọi là phần dư Peano trong công thức khai triển Taylor hữu hạn. Ta còn
viết Rn (x) = o ((x x0 )n ) là vô cùng bé bậc n khi x ! x0 :
Vậy, ta viết lại Công thức khai triển Taylor hữu hạn với phần dư Peano như sau
n
X f (k) (x0 )
f (x) = (x x0 )k + o ((x x0 )n ) ; khi (x x0 ) đủ bé.
k!
k=0
n
X f (k) (x0 )
Chứng minh Định lý 12. Đặt R(x) = f (x) (x x0 )k ; ta có
k!
k=0

R(x0 ) = R0 (x0 ) = R00 (x0 ) = = R(n) (x0 ) = 0;


R(n 1)
(x) = f (n 1)
(x) f (n 1)
(x0 ) f (n) (x0 )(x x0 );

Do f (n) (x0 ) tồn tại, nên f (n 1) khả vi tại x0 ; do đó R(n 1) (x) cũng khả vi tại x0 ; do đó

R(n 1)
(x) = R(n 1)
(x) R(n 1)
(x0 ) = (x x0 )R(n) (x0 ) + o ((x x0 )) = o ((x x0 )) :

Áp dụng Định lý giá trị trung bình Largrange cho hàm R(n 2) (x); ta có x2 2 (minfx; x0 g; maxfx; x0 g) ;
sao cho

R(n 2)
(x) = R(n 2)
(x) R(n 2)
(x0 ) = (x x0 )R(n 1)
(x2 )
2
= (x x0 )o ((x2 x0 )) = o (x x0 ) ;

bởi vì khi x ! x0 ; từ x2 2 (minfx; x0 g; maxfx; x0 g) ; dẫn đến x2 ! x0 và do đó


j(x x0 )o ((x2 x0 ))j o ((x2 x0 )) x2 x0 o ((x2 x0 ))
= ! 0;
(x x0 )2 x2 x0 x x0 x2 x0
khi x ! x0 ;

vậy
R(n 2)
(x) = o (x x0 )2 ; khi x ! x0 :
Bằng qui nạp giả sử ta có

R(n k)
(x) = o (x x0 )k ; khi x ! x0 :

Áp dụng Định lý giá trị trung bình Largrange cho hàm R(n k 1) (x); ta có xk+1 2 (minfxk ; x0 g; maxfxk ; x0 g) ;
sao cho

R(n k 1)
(x) = R(n k 1)
(x) R(n k 1)
(x0 ) = (x x0 )R(n k)
(xk+1 )
k k+1
= (x x0 )o (xk+1 x0 ) = o (x x0 ) ; khi x ! x0 :
Chương 6. ĐẠO HÀM 81

Với k = n 1, ta có
R(x) = o ((x x0 )n ) ; khi x ! x0 :
Định lý 12 được chứng minh xong.
Chú thích về vô cùng bé.
Định nghĩa 8.
(i) Nếu lim f (x) = 0; ta nói f (x) là vô cùng bé (VCB) khi x ! x0 :
x!x0
(ii) Cho f (x) và g(x) là hai vô cùng bé (VCB) khi x ! x0 :
f (x)
- Nếu lim = 0; ta nói f (x) là VCB cấp cao hơn VCB g(x) khi x ! x0 : Khi đó ta viết
x!x0 g(x)
f (x) = o (g(x)) khi x ! x0 :
- Nếu f (x) = o ((g(x))n ) ; khi x ! x0 ; ta nói f (x) là VCB cấp n so với VCB g(x) khi x ! x0 :
- Nếu f (x) = o ((x x0 )n ) ; khi x ! x0 ; ta nói f (x) là VCB cấp n so với VCB (x x0 ) khi
x ! x0 :

6.4 Công thức khai triển Maclaurin hữu hạn


Nếu thay x0 = 0 trong công thức khai triển Taylor hữu hạn ta thu được công thức khai triển Maclaurin
hữu hạn sau
Định lý 13 (Công thức khai triển Maclaurin hữu hạn với phần dư Lagrange). Giả sử
0 2 (a; b); cho f : (a; b) ! R có đạo hàm đến cấp n + 1 trong khoảng (a; b): Khi đó, với mọi x 2 (a; b);
ta có 2 (0; 1) sao cho
n
X f (k) (0) k f (n+1) ( x) n+1
f (x) = x + x :
k! (n + 1)!
k=0

f (n+1) ( x) n+1
Số hạng Rn (x; f ) = x gọi là phần dư Lagrange trong công thức khai triển Maclaurin.
(n + 1)!
Định lý 14 (Công thức khai triển Maclaurin hữu hạn với phần dư Peano). Giả sử 0 2 (a; b);
cho f : (a; b) ! R có đạo hàm đến cấp n 1 trong khoảng (a; b) sao cho tồn tại f (n) (x0 ): Khi đó ta có
n
X f (k) (0)
f (x) = xk + Rn (x); khi x ! 0,
k!
k=0

Rn (x)
trong đó lim = 0:
x!0 xn
Số hạng Rn (x) gọi là phần dư Peano trong công thức khai triển Maclaurin. Ta còn viết Rn (x) = o(xn )
và gọi nó là vô cùng bé bậc n khi x ! 0:
Vậy, ta viết lại Công thức khai triển Maclaurin hữu hạn với phần dư Peano như sau
n
X f (k) (0)
f (x) = xk + o(xn ); khi x ! 0.
k!
k=0

Chú thích. Công thức khai triển Taylor hữu hạn ngoài các phần dư dạng Lagrange và dạng Peano,
còn có phần dư dạng tích phân (Phần này sẽ đọc sau khi đọc chương tích phân):
Định lý 15 (Công thức khai triển Taylor hữu hạn với phần dư dạng tích phân). Cho
f : (a; b) ! R có đạo hàm đến cấp n + 1 trong khoảng (a; b): Khi đó, với mọi x; x0 2 (a; b); ta có
n
X f (k) (x0 )
f (x) = (x x0 )k + Rn (x; x0 ; f );
k!
k=0
Chương 6. ĐẠO HÀM 82

trong đó số hạng
Z x
1
Rn (x; x0 ; f ) = (x t)n f (n+1) (t) dt
n! x0
Z 1
(x x0 )n+1
= (1 )n f (n+1) (x0 + (x x0 )) d
n! 0

gọi là phần dư dạng tích phân trong công thức khai triển Taylor hữu hạn.
Định lý 16 (Công thức khai triển Maclaurin hữu hạn với phần dư dạng tích phân). Giả
sử 0 2 (a; b); cho f : (a; b) ! R có đạo hàm đến cấp n + 1 trong khoảng (a; b): Khi đó, với mọi x 2 (a; b);
ta có
n
X f (k) (0) k
f (x) = x + Rn (x; f );
k!
k=0

trong đó số hạng
Z x Z 1
1 n (n+1) xn+1
Rn (x; f ) = (x t) f (t) dt = (1 )n f (n+1) ( x) d ;
n! 0 n! 0

gọi là phần dư dạng tích phân trong công thức khai triển Maclaurin hữu hạn.

6.5 Ứng dụng của công thức Taylor để tính gần đúng

n
X f (k) (x0 )
f (x) (x x0 )k , khi (x x0 ) bé.
k!
k=0

Đặc biệt với n = 1:


f (x) f (x0 ) + f 0 (x0 )(x x0 ), khi (x x0 ) bé.

6.6 Các ứng dụng của đạo hàm vào khảo sát hàm số
6.6.1 Tính đơn điệu của hàm số
Định lý 17. (Tính đơn điệu của hàm số). Cho hàm số f : (a; b) ! R có đạo hàm trong khoảng (a; b).
Khi đó
(i) f 0 (x) = 0; 8x 2 (a; b) () f là hàm hằng trên (a; b);
(ii) f 0 (x) > 0; 8x 2 (a; b) =) f là hàm tăng trên (a; b);
(iii) f 0 (x) < 0; 8x 2 (a; b) =) f là hàm giảm trên (a; b);
(iv) f là hàm không giảm trên (a; b) () f 0 (x) 0; 8x 2 (a; b);
(v) f là hàm không tăng trên (a; b) () f 0 (x) 0; 8x 2 (a; b):
Chứng minh Định lý 17. Sử dụng Định lý giá trị trung bình Lagrange.
Chú thích. Phần đảo của (ii) và (iii) không đúng, ví dụ hàm f (x) = x3 ; x 2 R tăng trên R; mà ta
có f 0 (0) = 0:

6.6.2 Điều kiện đủ để hàm số cực trị


Định lý 18. (Điều kiện đủ để hàm số cực trị). Cho hàm số f : (a; b) ! R liên tục trên (a; b): Cho
x0 2 (a; b) sao cho f có đạo hàm trong hai khoảng (a; x0 ); (x0 ; b): Khi đó
(i) Nếu f 0 (x) < 0; 8x 2 (a; x0 ) và f 0 (x) > 0; 8x 2 (x0 ; b); thì f đạt cực tiểu tại x0 ;
(ii) Nếu f 0 (x) > 0; 8x 2 (a; x0 ) và f 0 (x) < 0; 8x 2 (x0 ; b); thì f đạt cực đại tại x0 :
Chứng minh Định lý 18. Sử dụng Định lý 17.
Chương 6. ĐẠO HÀM 83

Chú thích. Định lý 18 cho thấy rằng đạo hàm f 0 (x) đổi dấu từ âm sang dương khi x vượt qua x0
thì f đạt cực tiểu tại x0 :
Và, đạo hàm f 0 (x) đổi dấu từ dương sang âm khi x vượt qua x0 thì f đạt cực đại tại x0 :
Có thể kiểm tra lại rằng thì f không đạt cực trị tại x0 nếu đạo hàm f 0 (x) không đổi dấu khi x vượt
qua x0 ; tức là f 0 (x)f 0 (y) > 0; 8(x; y) 2 (x; x0 ) (x0 ; b):
Định lý 19. (Điều kiện đủ để hàm số cực trị). Cho hàm số f : (a; b) ! R có đạo hàm trong khoảng
(a; b): Giả sử x0 2 (a; b) sao cho f 0 (x0 ) = 0:
Giả sử tồn tại f 00 (x0 ) 6= 0: Khi đó f đạt cực trị tại x0 :
(i) Nếu f 00 (x0 ) > 0; thì f đạt cực tiểu tại x0 ;
(ii) Nếu f 00 (x0 ) < 0; thì f đạt cực đại tại x0 :
Chứng minh Định lý 19.
(i) f 00 (x0 ) > 0: Sử dụng công thức khai triển Taylor hữu hạn với phần dư Peano với n = 2, ta có
1
f (x) f (x0 ) = f 0 (x0 )(x x0 ) + f 00 (x0 )(x x0 )2 + R2 (x)
2
1 00 R2 (x)
= (x x0 )2 f (x0 ) + ;
2 (x x0 )2

R2 (x)
với lim = 0:
x!x0 (x x0 )2
1 00 R2 (x) 1 1 00
Do lim f (x0 ) + 2
= f 00 (x0 ) > 0; tồn tại > 0 : (x0 ; x0 + ) (a; b) và f (x0 ) +
x!x0 2 (x x0 ) 2 2
R2 (x)
> 0; 8x 2 (x0 ; x0 + ) nfx0 g:
(x x0 )2
Do đó, f đạt cực tiểu tại x0 .
(ii) f 00 (x0 ) < 0: Lý luận tương tự, ta cũng có thì f đạt cực đại tại x0 :
Định lý 19 được chứng minh xong.
Định lý 20. (Điều kiện đủ để hàm số cực trị). Cho hàm số f : (a; b) ! R có đạo hàm đến cấp n 1
trong khoảng (a; b):
Cho x0 2 (a; b) sao cho tồn tại f (n) (x0 ) và thỏa f 0 (x0 ) = f 00 (x0 ) = = f (n 1) (x0 ) = 0 6= f (n) (x0 ):
f đạt cực tiểu tại x0 ; nếu f (n) (x0 ) > 0;
(i) Nếu n chẳn, thì f đạt cực trị tại x0 :
f đạt cực đại tại x0 ; nếu f (n) (x0 ) < 0;
(ii) Nếu n lẻ, thì f không đạt cực trị tại x0 :
Chứng minh Định lý 20.
Sử dụng công thức khai triển Taylor hữu hạn với phần dư Peano, ta có
1 (n)
f (x) f (x0 ) = f (x0 )(x x0 )n + Rn (x)
n!
1 (n) Rn (x)
= (x x0 )n f (x0 ) + ;
n! (x x0 )n

Rn (x)
với lim = 0:
x!x0 (x x0 )n
(i) Với n chẳn, lý luận tương tự như trên với n = 2.
1 Rn (x)
(ii) Với n lẻ, số hạng f (n) (x0 ) + giữ một dấu nhất định (cùng dấu với f (n) (x0 ) ) trong
n! (x x0 )n
một khoảng (x0 ; x0 + ) (a; b) (ngoại trừ điểm x0 ). Mà (x x0 )n và f (x) f (x0 ) cũng đổi dấu khi
x vượt qua x0 ; do đó f không đạt cực trị tại x0 :
Định lý 20 được chứng minh xong.
Chương 6. ĐẠO HÀM 84

6.6.3 Hàm lồi, điểm uốn


Định nghĩa 9. Cho f : (a; b) ! R: Ta nói
(i) f là hàm lồi trên khoảng (a; b) nếu
[8x1 ; x2 2 (a; b); 8 2 [0; 1] =) f ( x1 + (1 )x2 ) f (x1 ) + (1 )f (x2 )];
(ii) f là hàm lồi ngặt trên khoảng (a; b) nếu
[8x1 ; x2 2 (a; b); x1 6= x2 ; 8 2 (0; 1) =) f ( x1 + (1 )x2 ) < f (x1 ) + (1 )f (x2 )];
(iii) f là hàm lõm trên khoảng (a; b) nếu f là hàm lồi trên khoảng (a; b);
(iv) f là hàm lõm ngặt trên khoảng (a; b) nếu f là hàm lồi ngặt trên khoảng (a; b);
(v) Đồ thị của hàm số lồi gọi là đường cong lõm;
(vi) Đồ thị của hàm số lõm gọi là đường cong lồi.

Ý nghĩa hình học của hàm lồi. Cho f là hàm lồi trên khoảng (a; b) và (L) là đồ thị của nó. Xét
hai điểm M1 (x1 ; f (x1 )); M2 (x2 ; f (x2 )) 2 (L); với x1 < x2 :
Xét cát tuyến M1 M2 là đường thẳng có phương trình

f (x1 ) f (x2 )
y = Y (x) = f (x2 ) + (x x2 ) :
x1 x2
x2 x0
Lấy x0 2 [x1 ; x2 ]; ta có x0 = x1 + (1 )x2 ; với = 2 [0; 1]: Khi đó, do f là hàm lồi ta có
x2 x1
f (x0 ) f (x1 ) + (1 )f (x2 ).

Mặt khác,

f (x1 ) + (1 )f (x2 ) = f (x2 ) + (f (x1 ) f (x2 ))


x2 x0
= f (x2 ) + (f (x1 ) f (x2 ))
x2 x1
f (x1 ) f (x2 )
= f (x2 ) + (x0 x2 ) = Y (x0 ):
x1 x2

Suy ra f (x0 ) ^ 0 (x0 ; Y (x0 )) 2 M1 M2 ,


Y (x0 ); do đó điểm M0 (x0 ; f (x0 )) 2 (L) nằm phía dưới điểm M
_
tức là phần đồ thị M1 M2 của (L) với x1 x x2 luôn nằm phía dưới cát tuyến M1 M2 : Điều này có
nghĩa là mọi dây cung đều nằm trên cung đồ thị mà nó chắn.
Mặt khác, giả sử (L) là đồ thị của một hàm f có đạo hàm trong khoảng (a; b): Ta cố định M1 (x1 ; f (x1 )) 2
(L); cho M2 (x2 ; f (x2 )) 2 (L); chạy trên đường cong (L) tới điểm M1 ; khi đó cát tuyến M1 M2 tiến dần
đến một vị trí giới hạn M1 T là tiếp tuyến của đường (L) tại M1 : Tiếp tuyến M1 T này nằm phía dưới
của đường (L): Điều này có nghĩa là mọi tiếp tuyến của đồ thị (L) đều nằm phía dưới (L):
Chương 6. ĐẠO HÀM 85

Định lý 21. Cho hàm số f : (a; b) ! R có đạo hàm đến cấp hai trong khoảng (a; b): Khi đó,

f là hàm lồi trên khoảng (a; b) () f 00 (x) 0; 8x 2 (a; b).

Chứng minh Định lý 21.


Chứng minh phần thuận =): Cho f là hàm lồi trên khoảng (a; b) ta có

f ( x1 + (1 )x2 ) f (x1 ) + (1 )f (x2 ); 8x1 ; x2 2 (a; b); 8 2 [0; 1]. (*)


x2 x
Xét x1 < x2 ; x 2 [x1 ; x2 ]; x = x1 + (1 )x2 ; = 2 [0; 1]:
x2 x1
Từ (*), ta suy ra
x2 x
f (x) f (x1 ) (1 ) (f (x2 ) f (x1 )) = (1 ) (f (x2 ) f (x1 ))
x2 x1
x x1
= (f (x2 ) f (x1 )) :
x2 x1
Do đó
f (x) f (x1 ) f (x2 ) f (x1 )
: (a1)
x x1 x2 x1
Từ (*), ta cũng có
x2 x
f (x) f (x2 ) (f (x1 ) f (x2 )) = (f (x1 ) f (x2 ))
x2 x1
x x2
= (f (x2 ) f (x1 ))
x2 x1
Vậy
f (x) f (x2 ) f (x2 ) f (x1 )
: (a2)
x x2 x2 x1
Từ (a1) và (a2), ta có
f (x) f (x1 ) f (x) f (x2 )
: (a3)
x x1 x x2
Trong (a3), ở vế trái cho x ! x+
1 ; sau đó ở vế phải cho x ! x2 ; ta thu được

f (x2 ) f (x1 )
f 0 (x1 ) f 0 (x2 ):
x2 x1
Do f 0 không giảm trên (a; b), theo Định lý 17 (iv), ta có f 00 (x) 0; 8x 2 (a; b):
Chứng minh phần đảo (=: Giả sử f 00 (x) 0; 8x 2 (a; b):
Cho x1 < x2 ; x 2 (x1 ; x2 ); theo Định lý giá trị trung bình Largrange, tồn tại c1 2 (x1 ; x); c2 2 (x; x2 )
sao cho
f (x) f (x1 )
= f 0 (c1 );
x x1
f (x) f (x2 )
= f 0 (c2 ):
x x2
Do f 00 (x) 0; 8x 2 (a; b) nên f 0 không giảm trên (a; b); ta có f 0 (c1 ) f 0 (c2 ):
Do đó
f (x) f (x1 ) f (x) f (x2 )
:
x x1 x x2
x2 x
Điều này đúng với mọi x 2 x 2 (x1 ; x2 ): Với mọi 2 (0; 1); lấy x = x1 +(1 )x2 ; ta có = ;
x2 x1
x x1
1 = ; thay vào bất đẳng thức này, sau khi biến đổi ta thu được
x2 x1
f (x) f (x1 ) + (1 )f (x2 ):
Chương 6. ĐẠO HÀM 86

Định lý 21 được chứng minh xong.


Tương tự, ta cũng có Định lý sau.
Định lý 22. Cho hàm số f : (a; b) ! R có đạo hàm đến cấp hai trong khoảng (a; b): Khi đó,
f là hàm lõm trên khoảng (a; b) () f 00 (x) 0; 8x 2 (a; b).
Định nghĩa 10. Cho (L) là đồ thị của hàm số f : (a; b) ! R và điểm M0 (x0 ; f (x0 )) 2 (L): Giả sử
_ _ _ _
có một cung CD nằm trên (L) với xC < x0 < xD mà CD được chia thành hai cung CM0 ; M0 D sao cho
một cung là đường cong lồi, một cung còn lại là đường cong lõm, khi đó ta nói M0 (x0 ; f (x0 )) là điểm
uốn của (L).

Định lý 23. Cho x0 2 (a; b): Cho hàm số f : (a; b) ! R liên tục trên (a; b) và có đạo hàm đến cấp
hai trong hai khoảng (a; x0 ); (x0 ; b):
Giả sử rằng
f 00 (x) < 0 < f 00 (y) hay f 00 (x) > 0 > f 00 (y); 8(x; y) 2 (a; x0 ) (x0 ; b).
(tức là f 00 (x) đổi dấu khi x vượt qua x0 ).
Khi đó, M0 (x0 ; f (x0 )) là điểm uốn của đồ thị của hàm số f:
Định lý 24. Cho hàm số f : (a; b) ! R có đạo hàm đến cấp hai trong hai khoảng (a; b): Nếu
x0 2 (a; b) là hoành độ điểm uốn thì f 00 (x0 ) = 0:
_ _
Chứng minh Định lý 24. Từ định nghĩa, giả sử CM0 là đường cong lồi, M0 D là đường cong lõm. Khi
đó
f 00 (x) < 0 < f 00 (y); 8(x; y) 2 (xC ; x0 ) (x0 ; xD ).
Cho x ! x0 và y ! x+
0; khi đó f 00 (x0 ) 0 f 00 (x0 ): Vậy f 00 (x0 ) = 0: Định lý 23 được chứng minh
xong.
Chú thích. Cho f là hàm lồi trên khoảng (a; b): Khi đó, tập C[f ] = f(x; y) : y f (x); x 2 (a; b)g:
Khi đó, ta có C[f ] là tập lồi, nghĩa là, mọi đoạn thẳng nối hai điểm trong C[f ] cũng nằm trong C[f ]:
Lấy A( ; yA ); B( ; yB ) 2 C[f ]; ta chứng minh đoạn thẳng AB C[f ]:
Thật vậy, cho M (x; y) 2 AB; ta chứng minh rằng y f (x):
x
Do x ; ta có x = + (1 ) ; = 2 [0; 1]: Do f là hàm lồi trên khoảng (a; b); và
yA f ( ); yB f ( ); ta có
f (x) f ( ) + (1 )f ( ) yA + (1 )yB :
yA yB yA yB
Do M (x; y) 2 AB, ta có y = yB + (x xB ) = yB + (x ) = yB + (yA yB ) =
xA xB
yA + (1 )yB :
Vậy y f (x); tức là M (x; y) 2 AB: Điều này dẫn tới AB C[f ]:
Chương 6. ĐẠO HÀM 87

6.7 Bài tập bổ sung


1. Các bài tập bổ sung về phép tính giới hạn (Có thể chứng minh trực tiếp hoặc dùng qui tắc L’Hopital
tùy bài)
1.1. Hàm lũy thừa
(i) lim x = +1; 8 > 0;
x!+1
(ii) lim xn = +1; 8n 2 N;
x!+1
+1; = 2n; n 2 N;
(iii) lim x =
x! 1 1; = 2n + 1; n 2 N;
(iv) lim x = 0; 8 > 0;
x!0+
(v) lim x = +1; 8 > 0;
x!0+
+1; = 2n; n 2 N;
(vi) lim x =
x!0 1; = 2n + 1; n 2 N:
1.2. Hàm mũ.
(i*) lim ex = +1;
x!+1
ex
(ii*) lim = +1;
x!+1 x
ex
(iii) lim = +1; 8 > 0;
x!+1 x
e x
(iiia) lim = +1; 8 > 0; 8 > 0;
x!+1 x
x
(iv*) lim e = 0;
x! 1
(v*) lim xe x = 0;
x! 1
(vi) lim x e x = 0; 8 > 0;
x!+1
(via) lim x e x = 0; 8 > 0; 8 > 0;
x!+1
ex 1
(vii*) lim = 1:
x!0 x !
ex 1 ex= 1 ey
Hướng dẫn (iii): Dùng (ii*), lim = lim = lim = +1:
x!+1 x x!+1 x= y=x= !+1 y
!
e x e x= ey
Hướng dẫn (iiia): Dùng (ii*), lim = lim = lim =
x!+1 x x!+1 x= y= x= !+1 y
+1:
x 1
Hướng dẫn (vi): Dùng (iii), lim x e = = 0:
x!+1 ex
lim
x!+1 x
x = 1
Hướng dẫn (via): Dùng (iiia), lim x e = 0:
x!+1 e x
lim
x!+1 x
1.3. Hàm logarit.
(i*) lim ln x = +1;
x!+1
ln x
(ii*) lim = 0;
x!+1 x
ln x
(iii) lim = 0; 8 > 0;
x!+1 x
(ln x)
(iiia) lim = 0; 8 > 0; 8 > 0;
x!+1 x
(iv*) lim ln x = 1;
x!0+
(v*) lim x ln x = 0;
x!0+
(vi) lim x ln x = 0; 8 > 0;
x!0+
Chương 6. ĐẠO HÀM 88

(via) lim x ( ln x) = 0; 8 > 0; 8 > 0;


x!0+
ln(1 + x)
(vii*) lim = 1:
x!0 x
ln x 1 ln x 1 ln y
Hướng dẫn (iii): Dùng (ii*), lim = lim = lim = 0:
x!+1 x x!+1 x y=x !+1 y
!
(ln x) ln x = ln y
Hướng dẫn (iiia): Dùng (ii*), lim = lim = lim =
x!+1 x x!+1 x = y=x = !+1 y
0:
1 1
Hướng dẫn (vi): Dùng (v*), lim x ln x = lim x ln x = lim y ln y = 0:
x!0+ x!0+ y=x !0+

Hướng dẫn (via): Dùng (v*), lim x ( ln x) = lim x = ln x = = lim ( y ln y)


x!0+ x!0+ y=x = !0+
0:
2. Tính các đạo hàm của các hàm số sau
(x 1)2 + x; x < 1;
(i) f (x) = 2
x 3(x 1) ; x 1;
8
< 0; x < 0;
(ii) f (x) = 2
x ; 0 x < 1;
:
x; x 1;
8 3
< x 2x; x < 1;
(iii) f (x) = x2 x 1; 1 x < 2;
:
(x 2)3 + 3x 5; x 2;
(iv) f (x) = jxj ; là hằng số thực;
(v) f (x) = jxj 1 x; là hằng số thực;
(vi) f (x) = jxj sin x; là hằng số thực;
(vii) f (x) = jxj (x) ; là hàm số có đạo hàm tại mọi x 2 R; (0) > 1; ( 1) > 0;
(viii) f (x) = (u(x))v(x) ; u; v là hai hàm số có đạo hàm tại mọi x 2 R; sao cho 0 < u(x) 6= 1;
8x 2 R.
(ix) f (x) = jxj3 ; tính f 0 (x); f 00 (x) tại mọi x 2 R: Chứng minh rằng f 000 (0) không tồn tại.
Hướng dẫn Bài 2 (iv): f (x) = jxj ; là hằng số thực.
(j) > 1 :
x > 0 : f (x) = jxj = x có đạo hàm là f 0 (x) = x 1 = jxj 2 x;
x < 0 : f (x) = jxj = ( x) có đạo hàm được tính theo công thức đạo hàm của hàm hợp là
f (x) = ( x) 1 ( x)0 =
0 ( x) 1 = jxj 2 x;
f (x) f (0) jxj f (x) f (0)
x = 0 : f (0) = 0; = ; do đó = jxj 1 ! 0; khi x ! 0: Vậy tồn tại
x 0 x x 0
0 f (x) f (0)
f (0) = lim = 0:
x!0 x 0
Cuối cùng với > 1 ta có
jxj 2 x; x 6= 0;
f 0 (x) =
0; x = 0:
1; x < 0;
(jj) = 1 : f 0 (x) = và không tồn tại f 0 (0):
1; x > 0;
(jjj) 0 < < 1 : f 0 (x) = jxj 2 x; x 6= 0; và không tồn tại f 0 (0):
(jv) = 0 : f (x) = 1; 8x 2 R : f 0 (x) = 0; 8x 2 R:
2
(v) < 0 : Hàm f xác định trên R r f0g và có đạo hàm tại mọi x 6= 0; và f 0 (x) = jxj x; 8x 6= 0:
Hướng dẫn Bài 2 (v): f (x) = jxj 1 x; là hằng số thực.
(j) > 1 :
jxj 1 ; x 6= 0;
f 0 (x) =
0; x = 0:
(jj) = 1 : f (x) = x; f 0 (x) = 1; 8x 2 R:
(jjj) 0 < < 1 : f 0 (x) = jxj 1 ; x 6= 0; và không tồn tại f 0 (0):
Chương 6. ĐẠO HÀM 89

x 1; x > 0;
(jv) = 0 : f (x) = =
jxj 1; x < 0:
Hàm f xác định trên R r f0g và có đạo hàm f 0 (x) = 0 tại mọi x 6= 0:
(v) < 0 : Hàm f xác định trên R r f0g và có đạo hàm tại mọi x 6= 0; và f 0 (x) = jxj 1 ; 8x 6= 0:
Hướng dẫn Bài 2 (vi): f (x) = jxj sin x; là hằng số thực.
(j) > 0 :
x > 0 : f (x) = jxj sin x = x sin x có đạo hàm là f 0 (x) = x 1 sin x + x cos x = jxj 2 x sin x +
jxj cos x;
x < 0 : f (x) = jxj sin x = ( x) sin x có đạo hàm được tính theo công thức đạo hàm của hàm hợp

f 0 (x) = ( x) 1
( x)0 sin x + ( x) cos x = ( x) 1
sin x + ( x) cos x
2
= jxj x sin x + jxj cos x;

f (x) f (0) jxj sin x sin x


x = 0 : f (0) = 0; = = jxj ! 0; khi x ! 0: Vậy tồn tại f 0 (0) = lim
x 0 x x x!0
f (x) f (0)
= 0:
x 0
Cuối cùng với > 0 ta có
2
jxj x sin x + jxj cos x; x 6= 0;
f 0 (x) =
0; x = 0:

(jj) = 0 : f 0 (x) = cos x 8x 2 R:


(jjj) < 0 : f 0 (x) = jxj 2 x sin x + jxj cos x; x 6= 0: Cũng chú ý là f không xác định tại x = 0 và
do đó không tồn tại f 0 (0):
Hướng dẫn Bài 2 (vii): f (x) = jxj (x) ; là hàm số có đạo hàm tại mọi x 2 R; (0) > 1; ( 1) > 0:
Tại 0 6= x 6= 1 :

ln f (x) = (x) ln jxj ;


f 0 (x) 0 (x)
= (x) ln jxj + ; 0 6= x 6= 1;
f (x) x
(x) (x)
f 0 (x) = jxj 0
(x) ln jxj + ; 0=6 x 6= 1;
x

f (x) f (1)
Tại x = 1 : Giả sử (1) > 0; f (1) = 1; xét lim :
x!1 x 1

f (x) f (1) jxj (x) 1 e (x) lnjxj 1 e (x) ln x 1


lim = lim = lim = lim
x!1 x 1 x!1 x 1 x!1 x 1 x!1 x 1
e (1+y) ln(1+y) 1
= lim
y=x 1!0 y
e (1+y) ln(1+y) 1 (1 + y) ln(1 + y)
= lim
y!0 (1 + y) ln(1 + y) y
e (1+y) ln(1+y) 1 ln(1 + y)
= lim lim lim (1 + y) = (1):
y!0 (1 + y) ln(1 + y) y!0 y y!0

Vậy f 0 (1) = (1):


Chương 6. ĐẠO HÀM 90

f (x) f ( 1)
Tại x = 1 : Giả sử ( 1) > 0; f ( 1) = 1; xét lim :
x! 1 x+1

f (x) f ( 1) jxj (x) 1 e (x) lnjxj 1 e (x) ln( x) 1


lim = lim = lim = lim
x! 1 x+1 x! 1 x+1 x! 1 x+1 x! 1 x+1
e (y 1) ln(1 y) 1
= lim
y=x+1!0 y
e (y 1) ln(1 y) 1 (y 1) ln(1 y)
= lim
y!0 (y 1) ln(1 y) y
e (y 1) ln(1 y) 1 ln(1 y)
= lim lim lim (y 1) = ( 1):
y!0 (y 1) ln(1 y) y!0 y y!0

Vậy f 0 (1) = ( 1):


f (x) f (0) jxj (x)
Tại x = 0 : Giả sử (0) > 1; f (0) = 0; xét lim = lim :
x!0 x 0 x!0 x

f (x) f (0) jxj (x)


lim = lim = lim e( (x) 1) ln x
= 0;
x!0+ x 0 x!0+ x x!0+

f (x) f (0) ( x) (x)


lim = lim = lim e( (x) 1) ln( x)
= 0;
x!0 x 0 x!0 x x!0
f 0 (0) = 0:

Vậy
8
>
> jxj (x) 0 (x) ln jxj + (x) ; 0 6= x 6= 1;
>
>
< x
0
f (x) = 0; x = 0;
>
>
>
> (1); x = 1;
:
( 1); x= 1
8
<
jxj (x) 0 (x) ln jxj + (x) ; x 6= 0;
= x
:
0; x = 0:

Hướng dẫn Bài 2 (viii):

f (x) = (u(x))v(x) = ev(x) ln u(x) > 0; 8x 2 R:


u ln u
x 7 ! u(x) 7 ! ln u(x) : có đạo hàm tại mọi x 2 R;
x 7 ! w(x) = v(x) ln u(x) : có đạo hàm tại mọi x 2 R;
0
f (x) = w0 (x)ew(x)
u0 (x)
= v 0 (x) ln u(x) + v(x) (u(x))v(x) :
u(x)

Hướng dẫn Bài 2 (ix): f (x) = jxj3 : f 0 (x) = 3 jxj x; f 00 (x) = 6 jxj ; 8x 2 R: (f 00 )0+ (0) = 6 6= 6=
(f 00 )0 (0); do đó f 000 (0) không tồn tại.
3. Xét tính liên tục và tính khả vi của các hàm số sau
x sin(1=x); x 6= 0;
(i) f (x) =
0; x = 0;
x2 cos(1=x); x 6= 0;
(ii) f (x) =
0; x = 0;
1=x 2
e ; x 6= 0;
(iii) f (x) =
0; x = 0;
2
e 1=x ; x 6= 0;
4. Cho hàm số f (x) =
0; x = 0;
Chương 6. ĐẠO HÀM 91

(i) Chứng minh rằng f (n) (0) = 0; 8n 2 N;


(ii) Suy ra rằng f có đạo hàm ở mọi cấp trong R (tức là có đạo hàm ở mọi cấp tại mọi điểm
thuộc R).
5. Cho hàm f : (a; b) ! R và x 2 (a; b): Đặt r = minfx a; b xg: Chứng minh rằng:
(i) (x r; x + r) (a; b);
(ii) f có đạo hàm tại x
f (x + hn ) f (x)
() 8fhn g ( r; r) và hn ! 0 =) hội tụ ;
hn
(iii) f có đạo hàm bên phải tại x
f (x + hn ) f (x)
() 8fhn g (0; b x) và hn ! 0 =) hội tụ ;
hn
(iv) f có đạo hàm bên trái tại x
f (x + hn ) f (x)
() 8fhn g (a x; 0) và hn ! 0 =) hội tụ ;
hn
x2 ; x 2 Q;
6. Cho hàm số f (x) =
0; x 2 R r Q:
(i) Chứng minh rằng f (0) tồn tại và tính f 0 (0);
0

(ii) Chứng minh rằng f không liên tục tại mọi điểm x 6= 0:
x2 + x; x 2 Q;
7. Cùng câu hỏi với Bài 6 với hàm số f (x) =
x; x 2 R r Q:
0 0
(i) Chứng minh rằng f (0) tồn tại và tính f (0);
(ii) Chứng minh rằng f không liên tục tại mọi điểm x 6= 0:
8. Cho hàm số f : (0; 1) ! R có đạo hàm trong khoảng (0; 1): Cho x 2 (0; 1) và một dãy số
fxn g (0; 1) r fxg sao cho f (xn ) = f (x); 8n 2 N:
Chứng minh rằng, nếu xn ! x; thì f 0 (x) = 0:
9. Cho hàm số f : (a; b) ! R và x 2 (a; b) sao cho tồn tại f 0 (x): Cho hai dãy số fxn g; fyn g (a; b)
sao cho:
(i) xn ! x; yn ! x;
(ii) a < xn < x < yn < b; 8n 2 N:
f (yn ) f (xn )
Chứng minh rằng lim = f 0 (x):
n!+1 yn xn
yn x x xn
Hướng dẫn Bài 9 : Đặt n = ;1 n = ; ta có
yn xn yn xn
f (yn ) f (xn ) f (yn ) f (x) + f (x) f (xn )
f 0 (x) = f 0 (x)
yn xn yn xn
f (yn ) f (x) f (x) f (xn )
= + f 0 (x)
yn xn yn xn
yn x f (yn ) f (x) x xn f (x) f (xn )
= + f 0 (x)
yn xn yn x yn xn x xn
f (yn ) f (x) f (x) f (xn )
= n + (1 n) f 0 (x)
yn x x xn
f (yn ) f (x) f (x) f (xn )
= n f 0 (x) + (1 n) f 0 (x) :
yn x x xn
f (yn ) f (x) f (x) f (xn )
Do n; 1 n 2 (0; 1) và f 0 (x) ! 0; f 0 (x) ! 0; nên
yn x x xn
f (yn ) f (xn )
f 0 (x)
yn xn
f (yn ) f (x) f (x) f (xn )
f 0 (x) + f 0 (x) ! 0; khi n ! 1:
yn x x xn
10. Cho hàm số f : (a; b) ! R và x 2 (a; b) sao cho tồn tại f 0 (x): Cho hai dãy số fxn g; fyn g (a; b)
sao cho:
Chương 6. ĐẠO HÀM 92

(i) xn ! x; yn ! x;
(ii) a < x < xn < yn < b; 8n 2 N;
yn x
(iii) Dãy bị chặn.
yn xn
f (yn ) f (xn )
Chứng minh rằng lim = f 0 (x):
n!+1 yn xn
yn x x xn
Hướng dẫn Bài 10 : Đặt n = > 1; 1 n = < 0; ta có
yn xn yn xn

f (yn ) f (xn ) f (yn ) f (x) + f (x) f (xn )


f 0 (x) = f 0 (x)
yn xn yn xn
f (yn ) f (x) f (x) f (xn )
= + f 0 (x)
yn xn yn xn
yn x f (yn ) f (x) x xn f (x) f (xn )
= + f 0 (x)
yn xn yn x yn xn x xn
f (yn ) f (x) f (x) f (xn )
= n + (1 n) f 0 (x)
yn x x xn
f (yn ) f (x) f (x) f (xn )
= n f 0 (x) + (1 n) f 0 (x) :
yn x x xn

Do f ng ; bị chặn, do đó cũng vậy f1 n g, ta có hằng số C > 0 sao cho:

j nj + j1 nj C; 8n 2 N:

f (yn ) f (x) f (x) f (xn )


Mà f 0 (x) ! 0; f 0 (x) ! 0; nên
yn x x xn

f (yn ) f (xn )
f 0 (x)
yn xn
f (yn ) f (x) f (x) f (xn )
C f 0 (x) + C f 0 (x) ! 0; khi n ! 1:
yn x x xn

x2 sin (1=x) ; x 6= 0;
11. Cho hàm số f (x) =
0; x = 0:
(i) Tính f 0 (0).
1 1
(ii) Cho hai dãy số fxn g; fyn g như sau xn = ; yn = :
2n 2n
2
f (yn ) f (xn ) 0
Chứng minh rằng lim 6= f (0): Hãy cho một giải thích về điều này.
n!+1 yn xn
Hướng dẫn Bài 11 :
sao cho:
f (x) f (0)
(i) = jx sin (1=x)j jxj ! 0; khi x ! 0: vậy tồn tại f 0 (0) và f 0 (0) = 0:
x 0
(ii) Ta có

f (yn ) f (xn ) yn2 sin (1=yn ) x2n sin (1=xn )


=
yn xn yn xn
2
yn 2 1 2
= = ! 6= 0 = f 0 (0):
yn xn 1
1
4n
yn 0
Kết quả này cho một phản ví dụ mà điều kiện dãy bị chận bị bỏ qua.
yn xn
Chương 6. ĐẠO HÀM 93

Thật vậy
1 1
1
yn 0 2n 2n
= 2 = 2 = 4n ! +1:
yn xn 1 1 1 1
2n 2n 1 2n
2 2n
2
12. Cho hàm số f : (a; b) ! R có đạo hàm trong khoảng (a; b) sao cho f 0 (x) = 0; 8x 2 (a; b):
Chứng minh rằng
(i) f (x) = f (y); 8x; y 2 (a; b);
a+b
(ii) f (x) = f ( ); 8x 2 (a; b):
2
Hướng dẫn Bài 12 :
(i) Cho a0 ; b0 2 (a; b); a < a0 < b0 < b; ta có f liên tục trên [a0 ; b0 ], có có đạo hàm trong khoảng (a0 ; b0 )
và f 0 (x) = 0; 8x 2 (a0 ; b0 ):
8x; y 2 [a0 ; b0 ]; dùng định lý Lagrange, tồn tại 2 (0; 1) : f (x) f (y) = (x y)f 0 ( y + (1 )y) = 0;
vì c = y + (1 0 0
)y 2 (a ; b ):
Như vậy, mệnh đề f (x) f (y) = 0; 8x; y 2 [a0 ; b0 ] đúng với mọi a0 ; b0 2 (a; b); sao cho a < a0 < b0 < b;
ta suy ra, mệnh đề f (x) f (y) = 0; 8x; y 2 (a; b) đúng.
a+b
(ii) Suy ra từ (i) với y = :
2
13. Cho hàm số f : [a; b] ! R liên tục trên đoạn [a; b]; có đạo hàm trong khoảng (a; b) sao cho
f 0 (x) = 0; 8x 2 (a; b):
Chứng minh rằng
(i) f (x) = f (y); 8x; y 2 [a; b];
(ii) f (x) = f (a); 8x 2 [a; b]:
Hướng dẫn Bài 13 : (i) Dùng định lý Lagrange cho hàm f trên [x; y]; với x; y 2 [a; b]:
14. Cho hàm số f (x) = arctgx; x 2 R:
(i) Tính f 0 (x); x 2 R;
(ii) Chứng minh rằng arctgx + arctg(1=x) = ; 8x > 0;
2
(iii) Chứng minh rằng arctgx + arctg(1=x) = ; 8x < 0:
2
Hướng dẫn Bài 14 :
1
(i) Dùng f 0 (x) = (arctg)0 (x) = ; 8x 2 R:
1 + x2
1 1 1
(ii) g(x) = arctg(1=x) = f (1=x); g 0 (x) = f 0 (1=x)(1=x)0 = 2 x2 = 1 + x2 ; 8x 6= 0:
1 + (1=x)
(iii) h(x) = arctgx + arctg(1=x); x 6= 0:
Cho b > 1, hàm h có đạo hàm trong (0; b) và h0 (x) = 0; 8x 2 (0; b); do đó h là hàm hằng trong (0; b);
do đó h(x) = h(1) = arctg1 + arctg1 = 8x 2 (0; b): Mà điều này đúng với mọi b > 1, do đó h(x) =
2 2
8x > 0:
Do hàm là hàm lẻ trên R; nên ta có hàm h cũng vậy, do đó h(x) = 8x < 0:
2
15. Cho hai hàm số f; g : R ! R có đạo hàm trên R sao cho
(i) f 0 (x) = g(x); 8x 2 R;
(ii) g 0 (x) = f (x); 8x 2 R;
(iii) f (0) = 1; g(0) = 0:
Chứng minh rằng f (x) = cos x; g(x) = sin x; 8x 2 R:
Hướng dẫn 15: Dùng hàm số h(x) = (f (x) cos x)2 + (g(x) sin x)2 ; và chứng minh h0 (x) = 0;
8x 2 R:
16. Cho hai hàm số f; g : R ! R có đạo hàm trên R sao cho
(i) f 0 (x) = g(x); 8x 2 R;
(ii) g 0 (x) = f (x); 8x 2 R;
(iii) f (0) = 1; g(0) = 0:
Chương 6. ĐẠO HÀM 94

ex + e x ex e x
Chứng minh rằng f (x) = ; g(x) = ; 8x 2 R:
2 2
x x
Hướng dẫn: Đặt F (x) = e (f (x) + g(x)) ; G(x) = e (f (x) g(x)) ; và chứng minh F 0 (x) = G0 (x) =
0; 8x 2 R:
16a. Cho hàm số f : [a; b] ! R có đạo hàm trên [a; b] sao cho f+0 (a) < f 0 (b): Chứng minh rằng, nếu
f+0 (a) < < f 0 (b); thì tồn tại c 2 (a; b) sao cho f 0 (c) = :
a+b
Hướng dẫn 16a: Đặt c = ; xét hai hàm số
2
a; a t c;
(t) =
2t b; c t b;
2t a; a t c;
(t) =
b; c t b:

Khi đó, ta có a (t) < (t) b; 8t 2 (a; b): Xét hàm số g : [a; b] ! R như sau
8
>
> f ( (t)) f ( (t))
< ; a < t < b;
(t) (t)
g(t) =
>
> f 0 (a); x = a;
: +0
f (b); x = b:

Khi đó g liên tục trên [a; b]: Thật vậy, do g liên tục trong khoảng (a; b); ta chỉ cần kiểm tra điều này
bằng cách chứng minh lim g(t) = g(a) = f+0 (a) và lim g(t) = g(b) = f 0 (b):
t!a+ t!b
Chú ý rằng, lim (t) = a; ta có
t!a+

f ( (t)) f ( (t)) f ( (t)) f (a)


lim g(t) = lim = lim = f+0 (a) = g(a):
t!a+ t!a+ (t) (t) t!a+ (t) a

Tương tự, do lim (t) = b; ta có


t!b

f ( (t)) f ( (t)) f (b) f ( (t))


lim g(t) = lim = lim = f 0 (b) = g(b):
t!b t!b (t) (t) t!b b (t)

Do Định lý giá trị trung gian, với g : [a; b] ! R liên tục trên [a; b] và g(a) < < g(b); thì tồn tại
t0 2 (a; b) sao cho g(t0 ) = :
Do Định lý giá trị trung bình Lagrange, với f : [ (t0 ); (t0 )] ! R liên tục trên [ (t0 ); (t0 )] và có đạo
hàm trong ( (t0 ); (t0 )) ; khi đó tồn tại c 2 ( (t0 ); (t0 )) sao cho

f ( (t0 )) f ( (t0 ))
= g(t0 ) = = f 0 (c):
(t0 ) (t0 )

Do a (t0 ) < c < (t0 ) b; nên a < c < b:


Chương 6. ĐẠO HÀM 95

17. Cho đa thức bậc n với các hệ số thực a0 ; a1 ; ; an như sau Pn (x) = a0 + a1 x + + an xn =
n
X
ai xi ; với an 6= 0: Giả sử x1 ; ; xk là k nghiệm của đa thức Pn (x); tức là Pn (xj ) = 0; j = 1; 2; ; k:
i=0
Chứng minh rằng k n:
18. Cho hàm số f : (a; b) ! R: Chứng minh rằng f là hàm lồi trên khoảng (a; b) khi và chỉ khi
n
! n n
X X X
f x
i i i f (xi ); 8x1 ; ; xn 2 (a; b); 8 1 ; ; n 0; i = 1.
i=1 i=1 i=1
n
X n
X
Chú ý rằng i xi 2 (a; b); 8x1 ; ; xn 2 (a; b); 8 1; ; n 0; i = 1.
i=1 i=1
Hướng dẫn Bài 18 :
(i) Phần đảo ((=): Chỉ cần lấy n = 2; 1 = ; 2 =1 :
n
X
(ii) Phần thuận (=)): Cho x1 ; ; xn 2 (a; b); 1; ; n 0; i = 1: Ta chứng minh rằng
i=1
n
! n
X X
f i xi i f (xi ). (*)
i=1 i=1

Ta chứng minh (*) đúng bằng qui nạp theo n.


Hiển nhiên (*) đúng với n = 1.
n
X1
Giả sử (*) đúng với n 1; ta có i =1 n:
i=1
Nếu n = 1; thì =
1 = n 1 = 0; hiển nhiên (*) đúng.
n
X 1 n
X1 i
Nếu 0 n < 1; thì i =1 n > 0; do đó = 1; theo giả thiết qui nạp, ta có
1 n
i=1 i=1
n 1
! n 1
X i
X i
f xi f (xi ) : (**)
1 n 1 n
i=1 i=1
Chương 6. ĐẠO HÀM 96

Mặt khác, ta viết


n
X n
X1 i
i xi = (1 n) xi + n xn :
1 n
i=1 i=1

Do f là hàm lồi trong khoảng (a; b), sau đó dùng (**), ta có


n
! n
!
X X1 i
f i xi = f (1 n) xi + n xn
1 n
i=1 i=1
n
!
X1 i
(1 n) f xi + n f (xn )
1 n i=1
n
X1 n
X
i
(1 n) f (xi ) + n f (xn ) = i f (xi ).
1 n
i=1 i=1

Vậy phần thuận đúng.


19. Cho hàm số f : (a; b) ! R liên tục trong (a; b). Chứng minh rằng f là hàm lồi trên khoảng (a; b)
khi và chỉ khi
x+y f (x) + f (y)
f ; 8x; y 2 (a; b).
2 2
Hướng dẫn Bài 19 :
(i) Phần thuận (=)): hiển nhiên đúng do định nghĩa và không cần giả thiết về tính liên tục của hàm
f:
(ii) Phần đảo ((=): Ta cần chứng minh rằng
1 1 1 1
(j) f n
x + (1 n
)y n
f (x) + (1 )f (y); 8x; y 2 (a; b); 8n 2 N:
2 2 2 2n
m m m m
(jj) f n x + (1 n
)y n
f (x) + (1 )f (y); 8x; y 2 (a; b); 8n; m 2 N; 0 m 2n :
2 2 2 2n
m
(jjj) Chứng minh tập A = f n : n 2 Z+ ; m 2 Zg trù mật trong R:
2
(jv) Chứng minh f ( x + (1 )y) f (x) + (1 )f (y); 8x; y 2 (a; b); 8 2 [0; 1].
Hướng dẫn (j): Chứng minh bằng qui nạp theo n. Giả sử đúng với n 1: Chú ý rằng

1 1
1 1 y + n 1 x + (1 )y
x + (1 )y = 2 2n 1 ;
2n 2n 2
1 1
là trung bình cộng của hai số y và x + (1 )y; do đó
2n 1 2n 1
0 1
1 1
1 1 y + n 1 x + (1 )y
B 2 2n 1 C
f x + (1 )y = f@ A
2n 2n 2

1 1
f (y) + f x + (1 )y
2n 1 2n 1
2
1 1
f (y) + f (x) + (1 )f (y)
2n 1 2n 1
2
1 1
= f (x) + (1 )f (y):
2n 2n
Hướng dẫn (jj): Chứng minh bằng qui nạp theo m. Giả sử đúng với m: Chú ý rằng

1 1 m m
m+1 m+1 n 1
x + (1 )y + n 1 x + (1 )y
x + (1 )y = 2 2n 1 2 2n 1 ;
2n 2n 2
Chương 6. ĐẠO HÀM 97

1 1 m m
là trung bình cộng của hai số x + (1 )y và n 1 x + (1 )y; do đó
2n 1 2n 1 2 2n 1
0 1
1 1 m m
m+1 m+1 x + (1 )y + x + (1 )y
B n 1 2n 1 2n 1 2n 1 C
f x + (1 )y = f@2 A
2n 2n 2

1 1 m m
f x + (1 )y + f x + (1 )y
2n 1 2n 1 2n 1 2n 1
2
1 1 m m
f (x) + (1 )f (y) + f (x) + (1 )f (y)
2n 1 2n 1 2n 1 2n 1
2
m+1 m+1
= f (x) + (1 )f (y):
2n 2n
Hướng dẫn (jjj): Chứng minh tương tự như trong chứng minh tính trù mật của Q trong R:
[
Ở đây có thể làm theo cách sau. Ta có x 2 R+ = [0; +1) = [n; n + 1); ta có n0 2 Z+ : n0
n2Z+
x < n0 + 1:
1 k k + n0 2m0
Cho " > 0; tồn tại m0 2 N : m0 < ": Đặt k = n0 + m0 = ; k = 1; 2; ; 2m0 : Khi đó,
[2m0 2 2 2m0
x 2 [n0 ; n0 + 1) = [ k 1 ; k ); ta có k 2 f1; 2; ; 2m0 g sao cho k 1 x < k :
k=1
Như vậy
1
j k xj < j k k 1j = < ":
2m0
Tương tự cho x < 0:
m mk
Hướng dẫn (jv): Cho 2 (0; 1); do A = f : 8n; m 2 Zg trù mật trong R; ta có dãy f k = g A
2n 2nk
so cho k ! : Do 2 (0; 1); ta có k0 2 N :
mk
k = 2 (0; 1); 8k k0 :
2nk
Do liên tục trong (a; b); từ bất đẳng thức

f( kx + (1 k )y) k f (x) + (1 k )f (y); 8x; y 2 (a; b); 8k k0 ;

ta suy ra (jv) đúng.


20. Chứng minh rằng, 8p > 1; ta có
n
!p n
X X
xi np 1
xpi ; 8x1 ; ; xn 0:
i=1 i=1

Hướng dẫn Bài 20 : Xét hàm f (x) = xp ; x 0; p > 1:


21. Chứng minh rằng, 8p > 1; ta có
n
!p n n
X X p
X
i xi i xi ; 8x1 ; ; xn 0; 8 1; ; n 0; i = 1.
i=1 i=1 i=1

Hướng dẫn Bài 21 : Xét hàm f (x) = xp ; x 0; p > 1:


22. Chứng minh rằng các bất đẳng thức sau
(i) x y 1 x + (1 )y; 8x; y > 0; 8 2 [0; 1].
(ii) x y z x + y + z; 8x; y; z > 0; 8 ; ; 0 0; + + = 1.
1 2
(iii) x1 x2 xnn
1 x1 + 2 x2 + + n xn ;
Xn
8x1 ; ; xn > 0; 8 1 ; ; n 0; i = 1.
i=1
Chương 6. ĐẠO HÀM 98

x+y sin x + sin y


(iv) sin ; 8x; y 2 [0; ];
2 2
x+y+z sin x + sin y + sin z
(v) sin ; 8x; y; z 2 [0; ];
3 3
(vi) sin ( x + (1 )y) sin x + (1 ) sin y; 8x; y 2 [0; ]; 8 2 [0; 1];
n
! n
X X
(vii) sin i xi i sin xi ;
i=1 i=1
n
X
8x1 ; ; xn 2 [0; ]; 8 1; ; n 0; i = 1.
s i=1
p p
x+y sin x + sin y
(viii) sin ; 8x; y 2 (0; );
2 2
s p p p
3
3
x+y+z sin x + 3 sin y + 3 sin z
(ix) sin ; 8x; y; z 2 (0; );
3 3
p p
n p
(x) n sin ( x + (1 )y) sin x + (1 ) n sin y;
8x; y 2 (0; ); 8 2 [0; 1]; 8n > 1;
n
! n
X X
p p
(xi) sin i xi i sin xi ;
i=1 i=1
n
X
8x1 ; ; xn 2 (0; ); 8 1 ; ; n 0; i = 1; 8p 2 (0; 1):
v i=1
u n
! n
u X X p
(xii) tsin
m
x
i i i
m
sin xi ;
i=1 i=1
n
X
8x1 ; ; xn 2 (0; ); 8 1; ; n 0; i = 1; 8m > 1:
i=1
(xiii) (sin x) (sin y)1 sin ( x + (1 )y) ; 8x; y 2 (0; ); 8 2 [0; 1]:
n
!
X
1 2 n
(xiiii) (sin x1 ) (sin x2 ) (sin xn ) sin i xi ;
i=1
n
X
8x1 ; ; xn 2 (0; ); 8 1; ; n 0; i = 1:
i=1

1
Hướng dẫn Bài 22 : (i)- (iii). Xét hàm f (x) = > 0; x 2 [0; ]: ln x; x > 0; f 00 (x) =
x2
Hướng dẫn Bài 22 : (iv)- (vii). Xét hàm f (x) = sin x; x 2 [0; ]; f 00 (x) = cos x 0; x 2 [0; ]:
Hướng dẫn Bài 22 : (viii)- (xii). Xét hàm f (x) = sinp x; x 2 (0; ); 0 < p < 1:

f (x) = sinp x; x 2 [0; ]; p 2 (0; 1):


f 0 (x) = p cos x sinp 1
x;
00 p 1
f (x) = p sin x sin x + (p 1) cos2 x sinp 2
x
p 2 p 2
= p sin x + (p 1) 1 sin x sin x
2 p 2
= p p 1 p sin x sin x < 0; 8x 2 (0; ); 8p 2 (0; 1):

1
Hướng dẫn Bài 22 : (xiii)- (xiiii). Xét hàm f (x) = ln(sin x); x 2 (0; ); f 00 (x) = 2 < 0; x 2 (0; ):
sin x

ln sin ( x + (1 )y) ln sin x + (1 ) ln sin y = ln sin x sin1 y;


1
(sin x) (sin y) sin ( x + (1 )y) :
n
X xk
23 (i) Chứng minh rằng ex = + o(xn ); khi x ! 0:
k!
k=0
Chương 6. ĐẠO HÀM 99

x2 x3
ex 1 x
(ii) Tính lim 2 6 :
x!0 x4
Hướng dẫn Bài 23 (i) Xét hàm f (x) = ex ; và f (k) (0) = 1; 8k = 0; 1; :
4
X k 2 3
x x x x4
Hướng dẫn Bài 23 (ii) n = 4 : ex = + o(x4 ) = 1 + x + + + + o(x4 );
k! 2 6 24
k=0

x2 x3
ex 1 x 4
2 6 = 1 + o(x ) ! 1 ; khi x ! 0:
x4 24 x4 24
Vậy
x2 x3
ex 1 x 4
lim 2 6 = 1 + lim o(x ) = 1 :
x!0 x4 24 x!0 x4 24
Chú ý rằng, nếu dùng qui tắc L’Hopital thì

x2 x3 x2
ex 1 x ex 1 x
lim 2 6 = lim 2
x!0 x4 x!0 4x3
ex 1 x
= lim
x!0 12x2
ex 1
= lim
x!0 24x
ex 1
= lim = :
x!0 24 24
x2 x3 x4
ex 1 x +
24. Tính lim 2 6 12 :
x!0 x4
4
X xk x2 x3 x4
Hướng dẫn Bài 24 : n = 4 : ex = + o(x4 ) = 1 + x + + + + o(x4 ); do đó
k! 2 6 24
k=0

x2 x3 x4 x4 x4
ex 1 x + = + o(x4 ) +
2 6 12 24 12
x4
= + o(x4 )
8
x2 x3 x4
ex 1 x + 1 o(x4 ) 1
2 6 12 = + ! ; khi x ! 0:
x4 8 x4 8
x2 x3 x4
ex 1 x + 1 o(x4 ) 1
lim 2 6 12 = + lim 4 = :
x!0 x4 8 x!0 x 8
25. Chứng minh rằng
Xn
( 1)k 1 2k 1 x3 x5 ( 1)n 1 2n
(i) sin x = x + o(x2n ) = x + + x 1 + o(x2n ); khi x ! 0:
(2k 1)! 3! 5! (2n 1)!
k=1
n
X ( 1) k 2 4
x x ( 1)n 2n
(ii) cos x = x2k + o(x2n+1 ) = 1 + + x + o(x2n+1 ); khi x ! 0:
(2k)! 2 4! (2n)!
k=0
Xn
(iii) (1 + x) = C k xk + o(xn )
k=0
( 1) 2
=1+ x+ x + + C n xn + o(xn ); khi x ! 0;
2
( 1) ( k + 1)
trong đó C k = ; 2 R:
k!
Chương 6. ĐẠO HÀM 100

Xn
1
(iv) = (1 + x) 1 = ( 1)k xk + o(xn ); khi x ! 0:
1+x
k=0
Xn
( 1)k 1 k
(v) ln(1 + x) = x + o(xn ); khi x ! 0:
k
k=0
Hướng dẫn Bài 25 :
(i) Xét hàm f (x) = sin x; ta có
k
f (k) (x) = sin(x + ); 8k = 0; 1; ;
2
k
f (k) (0) = sin( ); 8k = 0; 1; ;
2
f (2n) (0) = sin(n ) = 0; 8n = 0; 1; ;
f (2n+1) (0) = sin (2n + 1) = cos n = ( 1)n ; 8n = 0; 1; ;
2
2n
X f (k) (0)
sin x = f (x) = xk + o(x2n )
k!
k=0
n
X n
X
f (2k 1) (0) ( 1)k 1 2k
= x2k 1 + o(x2n ) = x 1 + o(x2n ):
(2k 1)! (2k 1)!
k=1 k=1
(ii) Xét hàm f (x) = cos x; ta có
f (k) (x) = sin x + (k 1) ; 8k = 0; 1; ;
2
f (k) (0) = sin (k 1) ; 8k = 0; 1; ;
2
f (2n+1) (0) = sin (n ) = 0; 8n = 0; 1; ;
f (2n) (0) = sin (2n = sin n
1) = cos n = ( 1)n ; 8n = 0; 1; ;
2 2
2n+1
X f (k) (0)
cos x = f (x) = xk + o(x2n+1 )
k!
k=0
Xn Xn
f (2k) (0) 2k ( 1)k 2k
= x + o(x2n+1 ) = x + o(x2n+1 ):
(2k)! (2k)!
k=0 k=0
(iii) Xét hàm f (x) = (1 + x) ; ta có
f 0 (x) = (1 + x) 1 ;
f 00 (x) = ( 1)(1 + x) 2 ; ;
(k)
f (0) = ( 1)( k + 1)(1 + x) k ; ; 8k = 1; 2; ;
(k)
f (0) = ( 1)( k + 1); 8k = 1; 2; ;
f (k) (0) ( 1) ( k + 1)
Ck = = ;
k! n
k! n
X f (k) (0) X
(1 + x) = f (x) = k n
x + o(x ) = C k xk + o(xn ):
k!
k=0 k=0
f (n+1) ( x) n+1
Chú ý với = n; f (n+1) = 0; do đó, Rn (x; f ) = x = 0;
(n + 1)!
n(n 1) (n k + 1)
Cnk =
k!
n(n 1) (n k + 1)(n k)(n k 1) 2:1 n!
= = :
k!(n k)(n k 1) 2:1 k!(n k)!

Khi đó ta thu được công thức khai triển nhị thức Newton
n
X
n
(1 + x) = Cnk xk :
k=0

(iv) Với = 1; ta có
1( 1 1) ( 1 k + 1) 1( 2) ( k)
Ck 1 = = = ( 1)k :
k! k!
Chương 6. ĐẠO HÀM 101

n
X
1 1
Vậy = (1 + x) = ( 1)k xk + o(xn ); khi x ! 0:
1+x
k=0
Chú ý rằng, ta có công thức tổng của n + 1 số hạng đầu tiên của một cấp số nhân công bội q
1 qn
1 + q + q2 + + qn = ; nếu q 6= 1:
1 q
n
X 1 ( x)n 1 ( x)n
Lấy q = x 6= 1; ta có ( 1)k xk = = ; hay
1+x 1+x 1+x
k=0

X n n
1 ( x)n X
= ( 1)k xk + = ( 1)k xk + o(xn ); khi x ! 0:
1+x 1+x
k=0 k=0

(v) Xét hàm f (x) = ln(1 + x); x > 1; ta có


1 1 2 2:3
f 0 (x) = ; f 00 (x) = ; f 000 (x) = ; f (4) (x) = ; ;
1+x (1 + x)2 (1 + x)3 (1 + x)4
( 1)k 1 (k 1)!
f (k) (x) = ; 8k = 1; 2; ;
(1 + x)k
f (k) (0) = ( 1)k 1 (k 1)!; 8k = 1; 2; ;
Xn (k) Xn
f (0) k n ( 1)k 1 k
ln(1 + x) = f (x) = x + o(x ) = x + o(xn ):
k! k
k=0 k=0
26. Tính các giới hạn
x2
e 2 cos x
(i) lim :
x!0 x4
p x2
1 x2 e 2
(ii) lim :
x!0 x4
1 1
(iii) lim :
x!0 x2 sin2 x
x2
Hướng dẫn Bài 26 (i): Khai triển Maclaurin các hàm e 2 và cos x :
2
X xk x2
ex = + o(x2 ) = 1 + x + + o(x2 );
k! 2
k=0
2
x2
x2 x2 2 x2 x4
e 2 = 1 + + o(x4 ) = 1 + + o(x4 );
2 2 2 8
x2 x4
cos x = 1 + + o(x5 );
2 4!
x2 x2 x4 x2 x4
e 2 cos x = 1 + + o(x4 ) 1 + + o(x5 )
2 8 2 4!
x4 x4
= + o(x4 ) o(x5 ) = + o(x4 ):
12 12
Do đó
x2
e 2 cos x 1 o(x4 ) 1
= + ! ; khi x ! 0:
x4 12 x4 12
Chương 6. ĐẠO HÀM 102

p x2
Hướng dẫn Bài 26 (ii): Khai triển Maclaurin các hàm 1 x2 và e 2 :

(1 + x)1=2 = 1 + C1=2
1 2
x + C1=2 x2 + o(x2 );
1 1 1
1 ( 1) 1
1
C1=2 = 2 = ; C1=2 2
= 2 2 = :
1! 2 2! 8
p 1 1 2
1+x = 1+ x x + o(x2 );
2 8
p 1 1 2
1 x = 1 x x + o(x2 );
2 8
p 1 2 1 4
1 x2 = 1 x x + o(x4 );
2 8
X2
xk x2
ex = + o(x2 ) = 1 + x + + o(x2 );
k! 2
k=0
2
x2
x2 x2 2 x2 x4
e 2 = 1 + + o(x4 ) = 1 + + o(x4 );
2 2 2 8
p x2 1 2 1 4 x2 x4
1 x2 e 2 = 1 x x + o(x4 ) 1 + + o(x4 )
2 8 2 8
1 4
= x + o(x4 ):
4
Do đó
p x2
1 x2 e 2 1 o(x4 ) 1
= + ! ; khi x ! 0:
x4 4 x 4 4
1 1
(iii) lim 2 :
x!0 sin x x2
Hướng dẫn Bài 26 (iii): Ta phân tích

1 1 x2 sin2 x (x + sin x)(x sin x)


= =
sin2 x x2 2 2
x sin x x2 sin2 x
sin x x 2 x sin x
= 1+ :
x sin x x3
sin x x sin x
Do ! 1 khi x ! 0; nên ta chỉ cần tính lim : Khai triển Maclaurin hàm sin x :
x x!0 x3
x3
sin x = x + o(x4 );
3!
x3
x sin x = + o(x4 ):
3!
Do đó
x3
x sin x + o(x4 ) 1 o(x4 ) 1
= 3! = + ! ; khi x ! 0:
x3 x3 6 x3 6
Vậy

1 1 sin x x 2 x sin x
lim = lim 1+ lim lim
x!0 sin2 x x2 x!0 x x!0 sin x x!0 x3
1 1
= 2 1 = :
6 3
27. Tìm cực trị các hàm số sau
(i) f (x) = x3 ; 8x 2 [ 1; 1];
Chương 6. ĐẠO HÀM 103

(ii) f (x) = x3 + jxj ; 8x 2 [ 1; 1];


(iii) f (x) = x4=5 ; 8x 2 [ 1; 1];
(iv) f (x) = sin x + cos x; 8x 2 [ ; ]:
Hướng dẫn Bài 27 : Nếu hàm số đạt cực trị thì tại nơi đạt cực trị sẽ có đạo hàm triệt tiêu hoặc đạo
hàm không tồn tại. Vậy ta sẽ tìm tập A = fx 2 (a; b) : f 0 (x) = 0 hay @f 0 (x)g và kiểm tra những điểm
trong A có phải cực trị hay không.
Hướng dẫn Bài 27 (i): f 0 (x) = 3x2 6= 0; 8x 2 ( 1; 1) r f0g: Vậy mọi điểm trong ( 1; 1) r f0g không
là điểm cực trị.
Tại x = 0; f 0 (0) = f 00 (0) = 0 6= 6 = f 000 (0): Do n = 3 lẻ, nên f không đạt cực trị tại x = 0:
Kết luận: hàm số không có cực trị.
Chú ý: Ta có thể kết luận tại x = 0 hàm f không đạt cực trị nhờ vào f 0 (x) = 3x2 không thay đổi
dấu khi x vượt qua 0 từ âm sang dương.
Hướng dẫn Bài 27 (ii):
(j) 1 < x < 0 : f (x) = x3 x; do đó f 0 (x) = 3x2 1; 8x 2 ( 1; 0);
f 0 (x) = 3x2 1 = 0; 1
() x = p = x0 ; mà f 0 (x0 ) = 0 6= 6x0 = f 00 (x0 ) < 0: Vậy f đạt cực đại
1<x<0 3
1
tại x0 = p :
3
(jj) 0 < x < 1 : f (x) = x3 + x; do đó f 0 (x) = 3x2 + 1 6= 0; 8x 2 (0; 1): Vậy f không đạt cực trị trong
(0; 1):
f (x) f (0) x3 + x 0
(jjj) x = 0 : f (0) = 0; lim = lim = lim x2 + 1 = 1: Vậy f+0 (0) = 1:
x!0+ x 0 x!0+ x 0 x!0+
f (x) f (0) x3 x 0
lim = lim = lim x2 1 = 1: Vậy f 0 (0) = 1 6= 1 = f+0 (0):
x!0 x 0 x!0+ x 0 x!0+
Do đó @f (0): 0
1
Mà f 0 (x) = 3x2 1 < 0; 8x 2 ( p ; 0); và f 0 (x) = 3x2 + 1 > 0; 8x 2 (0; 1):
3
Đạo hàm f 0 (x) thay đổi dấu từ âm sang dương khi x vượt qua 0. Vậy f đạt cực tiểu tại x = 0:
1
Kết luận: Hàm số f đạt cực trị tại hai chỗ: cực đại tại x0 = p và cực tiểu tại x = 0:
3
0 4
Hướng dẫn Bài 27 (iii): f (x) = p 6= 0; 8x 2 ( 1; 1) r f0g: Vậy mọi điểm trong ( 1; 1) r f0g
55x
không là điểm cực trị.
f (x) f (0) x4=5 1
Tại x = 0 : f (0) = 0; = = p5
! +1; khi x ! 0+ ; do đó @f+0 (0) và cũng vậy
x 0 x x
@f 0 (0):
1 1
Mặt khác, f 0 (x) = p 5
< 0; 8x 2 ( 1; 0); và f 0 (x) = p 5
> 0; 8x 2 (0; 1): Vậy f tại đạt cực tiểu tại
x x
x = 0:
Kết luận: Hàm số f đạt cực tiểu tại x = 0:
p
Hướng dẫn Bài 27 (iv): f 0 (x) = cos x sin x = 2 cos x + ; 8x 2 ( ; ):
( p ( 4 (
f 0 (x) = 2 cos x + = 0; x+ = +k ; x = + k ; k 2 Z;
4 () 4 2 () 4
x2( ; ) x2( ; ) x2( ; )
3
() x 2 x1 = ; x2 = :
4 4
p
Mặt khác, f 00 (x) = 2 sin x + ;
4
3 p 3
x1 = : f 0 (x1 ) = 0 6= f 00 (x1 ) = 2 > 0 : Vậy f tại đạt cực tiểu tại x1 = :
4 p 4
x2 = : f 0 (x2 ) = 0 6= f 00 (x2 ) = 2 < 0 : Vậy f tại đạt cực đại tại x2 = :
4 4
3
Kết luận: Hàm số f đạt cực trị tại hai chỗ: cực tiểu tại x1 = và cực đại tại x2 = :
4 4
28. Tìm điểm uốn của đồ thị của các hàm số sau
(i) f (x) = x3 ln x; x > 0;
Chương 6. ĐẠO HÀM 104

(ii) f (x) = x3 e x ; x 2 R;
(iii) f (x) = x3=7 ; x 2 R;
(iv) f (x) = sin2 x; x 2 [ ; ]:
29. Tìm cực trị và điểm uốn của đồ thị của các hàm số sau
(i) f (x) = cos x;
(ii) f (x) = x3 + 3 jxj ; 4 < x < 1:
Hướng dẫn Bài 29 (i): Cực trị.
f 0 (x) = sin x = 0 () x = xk = k ; k 2 Z;
00
f (xk ) = cos xk = cos (k ) = ( 1)k 1
:

k = 2n chẵn : f 0 (x2n ) = 0 6= f 00 (x2n ) = 1 < 0 : f đạt cực đại tại x2n = 2n ;


0 00
k = 2n + 1 lẻ : f (x2n+1 ) = 0 6= f (x2n+1 ) = 1 > 0 : f đạt cực tiểu tại x2n+1 = (2n + 1) :
Điểm uốn.
f 00 (x) = cos x = 0 () x = xk =
+ k ; k 2 Z;
2
f 00 (x) đổi dấu khi vượt qua xk ; do đó đồ thị của hàm cos x có các điểm uốn có hoành độ
+ k ; k 2 Z:
xk =
2
Hướng dẫn Bài 29 (ii): Tính toán các đạo hàm
8 3
< x 3x; 4 < x < 0;
f (x) = x3 + 3 jxj = 0; x = 0;
: 3
x + 3x; 0 < x < 1:

4 < x < 0; f (x) = x3 3x; f 0 (x) = 3x2 3 = 0 () x = 1;


f 00 (x) = 6x; f 0 ( 1) = 0 6= f 00 ( 1) = 6 < 0: Vậy f đạt cực đại tại x = 1;
0 < x < 1; f (x) = x3 + 3x; f 0 (x) = 3x2 + 3 6= 0; f không đạt cực trị trong (0; 1);
x = 0 : f (0) = 0;
f (x) f (0) x3 + 3x 0
lim = lim = lim x2 + 3 = 3: Vậy 9f+0 (0) = 3;
x!0+ x 0 x!0+ x 0 x!0+

f (x) f (0) x3 3x 0
lim = lim = lim x2 3 = 3: Vậy 9f 0 (0) = 3;
x!0 x 0 x!0+ x 0 x!0

f+ (0) = 3 6= 3 = f (0): Vậy @f 0 (0):


0 0

Mà f 0 (x) thay đổi dấu từ âm sang dương khi x vượt qua x = 0; bởi vì
f 0 (x) = 3(x2 1) < 0; 8x 2 ( 1; 0);
0 2
f (x) = 3(x + 1) > 0; 8x 2 (0; 1):
Vậy f đạt cực tiểu tại x = 0:
Tính đạo hàm cấp hai. Ta có
3(x2 1); 4 < x < 0;
f 0 (x) =
3(x2 + 1); 0 < x < 1;
Tại x = 0; @f 0 (0):
Do đó
6x; 4 < x < 0;
f 00 (x) =
6x; 0 < x < 1;
Tại x = 0; @f 0 (0); @f 00 (0):
Mà f 00 (x) thay đổi dấu khi x vượt qua x = 0; bởi vì f 00 (x) = 6x; 8x 6= 0: Vậy đồ thị của hàm f đi
qua điểm uốn O(0; 0):
Chương 7

TÍCH PHÂN RIEMANN

7.1 Khái niệm và tính chất căn bản tích phân Riemann
Định nghĩa 1. Cho n + 1 số thực x0 ; x1 ; ; xn 2 [a; b] sao cho a = x0 < x1 < < xn = b và n số
thực i 2 [xi 1 ; xi ]; 8i = 1; ; n:
Khi đó ta gọi P = fx0 ; x1 ; ; xn ; 1 ; ; ng là một phân hoạch của đoạn [a; b] và đặt

jP j = max (xi xi 1) = độ mịn (chuẩn) của phân hoạch P .


1 i n

Đặt P([a; b]) là tập hợp tất cả các phân hoạch của đoạn [a; b].
Định nghĩa 2. Cho f : [a; b] ! R bị chận trên đoạn [a; b] và P = fx0 ; x1 ; ; xn ; 1; ; ng là một
phân hoạch của đoạn [a; b]: Ta đặt
n
X
S(f; P ) = f ( i )(xi xi 1 );
i=1
Xn
U (f; P ) = sup f (x)(xi xi 1 );
i=1 xi 1 x xi

Xn
L(f; P ) = inf f (x)(xi xi 1 );
xi 1 x xi
i=1

và lần lượt gọi các tổng số này là tổng Riemann, tổng Riemann trên và tổng Riemann dưới của f tương
ứng với phân hoạch P .
(Cũng chú ý rằng tổng Riemann của f tương ứng với phân hoạch P phụ thuộc vào các điểm 1 ; ; n;
trong khi đó các tổng Riemann trên và tổng Riemann dưới của f thì không phụ thuộc).
Ta nói hàm f khả tích Riemann trên [a; b] nếu tồn tại I(f ) 2 R sao cho 8" > 0; 9 > 0 :

8P 2 P([a; b]); jP j < =) jS(f; P ) I(f )j < ".


Z b
Khi đó, ta ký hiệu I(f ) = f (x)dx và gọi I(f ) là tích phân Riemann (vắn tắt gọi là tích phân) của
a
f trên [a; b]:
Ta cũng viết
Z b
I(f ) = f (x)dx = lim S(f; P ):
a jP j!0
Z a Z b
Ta cũng ký hiệu f (x)dx = f (x)dx:
b Z 0 a

Nếu a = b; ta ký hiệu f (x)dx = 0:


0

===========Chú ý===============

105
Chương 7. TÍCH PHÂN RIEMANN 106

Chú ý: Một cách diễn đạt khác để đi đến định nghĩa tích phân Riemann như sau
Định nghĩa 1a. Một tập con hữu hạn Q [a; b] sao cho a; b 2 Q được gọi là một phân hoạch của
đoạn [a; b]:
Cho Q là một phân hoạch của đoạn [a; b]; ta có thể viết Q như sau

Q = fx0 ; x1 ; ; xn g;
với a x0 < x1 < < xn b:

Như vậy phân hoạch Q hình thành một phép chia đoạn [a; b] thành một số hữu hạn các đoạn con

[x0 ; x1 ]; [x1 ; x2 ]; ; [xn 1 ; xn ]:

Khi đó phân hoạch Q cũng hiểu là một phép chia đoạn [a; b] thành một số hữu hạn các đoạn con như
trên.
Với một phân hoạch Q = fx0 ; x1 ; ; xn g của đoạn [a; b] như trên, ta định nghĩa

jQj = max (xi xi 1) = độ mịn (hay chuẩn) của phân hoạch Q.


1 i n

Định nghĩa 2a. Cho f : [a; b] ! R bị chận trên đoạn [a; b] và Q = fx0 ; x1 ; ; xn g là một phân
hoạch của đoạn [a; b]:
Với mỗi đoạn con [xi 1 ; xi ]; ta chọn i 2 [xi 1 ; xi ] và thiết lập tổng
n
X
S(f; Q; f 1 ; ; n g) = f ( i )(xi xi 1 );
i=1

và được gọi là tổng Riemann của f tương ứng với phân hoạch Q và cách chọn các điểm i như trên.
Nếu tồn tại
Xn
lim S(f; Q; f 1 ; ; n g) = lim f ( i )(xi xi 1 ) = I
jQj!0 jQj!0
i=1

sao cho số thực I không phụ thuộc vào phân hoạch Q và không phụ thuộc vào cách chọn các điểm i như
Z b
trên, khi đó, ta nói hàm f khả tích Riemann trên [a; b]. Ta ký hiệu số thực I = f (x)dx và gọi I là
a
tích phân Riemann (vắn tắt gọi là tích phân) của f trên [a; b]:
==========================
Định lý 1. Cho f : [a; b] ! R liên tục, khi đó f khả tích Riemann trên [a; b].
Chứng minh Định lý 1.
b a
Xét một phân hoạch đều Pn = fx0 ; x1 ; ; x2n ; 1; ; 2n g; với xi = a + i ; i = 0; ; 2n ;
2n
và i= xi ; i = 1; ; 2n :
Khi đó, tổng Riemann của f tương ứng với phân hoạch Pn là
2 n 2n
X b aX
S(f; Pn ) = f (xi )(xi xi 1) = f (xi ).
2n
i=1 i=1

Ta sẽ chứng minh rằng fS(f; Pn )g là dãy Cauchy, và do đó nó hội tụ về số thực I(f ). Sau đó ta chứng
minh rằng
8" > 0; 9 > 0 : 8P 2 P([a; b]); jP j < =) jS(f; P ) I(f )j < ".
(i) Chứng minh fS(f; Pn )g là dãy Cauchy trong R:
Cho m > n; xét
X2m 2m
b aX
S(f; Pm ) = f (yj )(yj yj 1) = f (yj ),
2m
j=1 j=1
Chương 7. TÍCH PHÂN RIEMANN 107

b a
trong đó Pm = fy0 ; y1 ; ; y 2m ; y 1 ; ; y2m g; với yj = a + j ; j = 0; ; 2m :
2m
b a
Chú ý rằng fy0 ; y1 ; ; y 2m 1 ; y 2m g fx0 ; x1 ; ; x2n 1 ; x2n g; bởi vì xi = a + i = a+
2n
b a
i2m n = yi2m n :
2m
J1 = fj 2 N : x0 < yj x1 g = f1; 2; ; 2m ng = có 2m n số,
J2 = fj 2 N : x1 < yj x2 g = f2m n + 1; 2m n + 2; ; 2:2m ng = có 2m n số,
J3 = fj 2 N : x2 < yj x3 g = f2:2m n + 1; 2:2m n + 2; ; 3:2m n g = có 2m n số,
..
.
Ji = fj 2 N : xi 1 < yj xi g = f(i 1)2m n + 1; (i 1)2m n + 2; ; i2m ng = có 2m n số,
..
.
J2n = fj 2 N : x2n 1 < yj x2n g = f(2n 1)2m n + 1; (2n 1)2m n + 2; ; 2m g = có 2m n số.
Ta cũng chú ý rằng
2n
[
Ji = fj 2 N : 1 j 2m g;
i=1
X
(yj yj 1) = xi xi 1;
j2Ji
n
2 X
X
(yj yj 1) = b a;
i=1 j2Ji
2m 2 X n
X X
= :
j=1 i=1 j2Ji

Khi đó, ta viết lại hai tổng Riemann ở trên của f như sau
2n n
2 X
X X
S(f; Pn ) = f (xi )(xi xi 1) = f (xi )(yj yj 1 );
i=1 i=1 j2Ji
2m
X 2 X
X
n

S(f; Pm ) = f (yj )(yj yj 1) = f (yj )(yj yj 1 ):


j=1 i=1 j2Ji

Trừ hai tổng trên


n
2 X
X
S(f; Pn ) S(f; Pm ) = [f (xi ) f (yj )] (yj yj 1 );
i=1 j2Ji

và do đó
n
2 X
X
jS(f; Pn ) S(f; Pm )j = [f (xi ) f (yj )] (yj yj 1)
i=1 j2Ji
2 X n
X
jf (xi ) f (yj )j (yj yj 1)
i=1 j2Ji
n
2 X
X
sup jf (xi ) f (yj )j (yj yj 1)
j2Ji ; 1 i 2n i=1 j2Ji
= (b a) sup jf (xi ) f (yj )j :
j2Ji ; 1 i 2n
Chương 7. TÍCH PHÂN RIEMANN 108

Do f liên tục trên [a; b] nên f liên tục đều trên [a; b]: Cho " > 0; tồn tại >0:
"
8x; y 2 [a; b]; jx yj < =) jf (x) f (y)j < .
2(b a)
b a
Nếu m > n; jPn j = < , ta có jxi yj j < ; 8j 2 Ji ; 8i = 1; ; 2n ; do đó jf (xi ) f (yj )j <
2n
"
:
2(b a)
Từ đó
"
jS(f; Pn ) S(f; Pm )j < ;
2
b a
với mọi m > n > n0 ; với < :
2n0
Như vậy fS(f; Pm )g là dãy Cauchy, và do đó nó hội tụ về số thực I(f ). Sau đó chỉ cần chứng minh
rằng lim S(f; P ) = I(f ); i.e.,
jP j!0

8" > 0; 9 > 0 : 8P 2 P([a; b]); jP j < =) jS(f; P ) I(f )j < ".

(ii) Chứng minh lim S(f; P ) = I(f ):


jP j!0
Xét một phân hoạch P = fx0 ; x1 ; ; xn ; 1; ; ng là một phân hoạch của đoạn [a; b] và tổng
Riemann của f tương ứng với phân hoạch P là
n
X
S(f; P ) = f ( i )(xi xi 1 ).
i=1

Xét phân hoạch Pm như sau


b a
Pm = fy0 ; y1 ; ; y 2m ; y 1 ; ; y2m g; với yj = a + j ; j = 0; ; 2m :
2m

Ta tổ hợp phân hoạch P với phân hoạch Pm thành một phân hoạch mới như sau

R = fz0 ; z1 ; ; z k ; z1 ; ; zk g;

với
fz1 ; ; zk g = fy1 ; ; y2m g [ fx1 ; ; xn g:
Tổng Riemann của f tương ứng với phân hoạch R là
k
X
S(f; R) = f (zi )(zi zi 1 ).
i=1

Đặt Ii = fj : xi 1 < zj xi g = fj : [zj 1 ; zj ] [xi 1 ; xi ]g, ta có f1; 2; ; kg = I1 [ I2 [ [ In =


S
n
fj 2 N : [zj 1 ; zj ] [xi 1 ; xi ]g:
i=1

k
X n
X X
S(f; R) = f (zj )(zj zj 1) = f (zj )(zj zj 1)
j=1 i=1 [zj 1 ;zj ] [xi 1 ;xi ]
n
X X n
X X
= [f (zj ) f ( i )] (zj zj 1) + f ( i )(zj zj 1)
i=1 [zj 1 ;zj ] [xi 1 ;xi ] i=1 [zj 1 ;zj ] [xi 1 ;xi ]
n
X X n
X
= [f (zj ) f ( i )] (zj zj 1) + f ( i )(xi xi 1)
i=1 [zj 1 ;zj ] [xi 1 ;xi ] i=1
n
X X
= [f (zj ) f ( i )] (zj zj 1) + S(f; P ):
i=1 [zj 1 ;zj ] [xi 1 ;xi ]
Chương 7. TÍCH PHÂN RIEMANN 109

Từ đó
n
X X
S(f; R) S(f; P ) = [f (zj ) f ( i )] (zj zj 1 );
i=1 [zj 1 ;zj ] [xi 1 ;xi ]


n
X X
jS(f; R) S(f; P )j (zj zj 1)
i=1 [zj 1 ;zj ] [xi 1 ;xi ]

= (b a) sup jf (zj ) f ( i )j :
[zj 1 ;zj ] [xi 1 ;xi ]; i=1; ;n

Tương tự
2 m
X X
jS(f; R) S(f; Pm )j (zj zj 1)
i=1 [zj 1 ;zj ] [yi 1 ;yi ]

= (b a) sup jf (zj ) f (yi )j .


[zj 1 ;zj ] [yi 1 ;yi ]; i=1; ;2m

Do f liên tục trên [a; b] nên f liên tục đều trên [a; b]: Cho " > 0; tồn tại > 0 :
"
8x; y 2 [a; b]; jx yj < =) jf (x) f (y)j < .
3(b a)
"
Nếu jP j < , ta có jxi xi 1j jP j < ; 8i = 1; ; n; do đó jf (x) f (y)j < ; 8x; y 2
3(b a)
[xi 1 ; xi ]; 8i = 1; ; n:
Từ đó
"
sup jf (zj ) f ( i )j < ; nếu jP j < :
[zj 1 ;zj ] [xi 1 ;xi ]; i=1; ;n 3(b a)
b a b a
Nếu jPm j = m < , ta có jyi yi 1 j jPm j = < ; 8i = 1; ; 2m ; do đó jf (x) f (y)j <
2 2m
"
; 8x; y 2 [yi 1 ; yi ]; 8i = 1; ; 2m :
3(b a)
" b a
sup jf (zj ) f (yi )j < ; nếu < :
[zj 1 ;zj ] [yi 1 ;yi ]; i=1; ;2m 3(b a) 2m

b a
Vậy nếu < và jP j < , ta có
2m
jS(f; R) S(f; P )j (b a) sup jf (zj ) f ( i )j
[zj 1 ;zj ] [xi 1 ;xi ]; i=1; ;n
" "
(b a) = ;
3(b a) 3

jS(f; R) S(f; Pm )j (b a) sup jf (zj ) f (yi )j
[zj 1 ;zj ] [yi 1 ;yi ]; i=1; ;2m
" "
(b a) = :
3(b a) 3
b a "
Mặt khác do S(f; Pm ) ! I(f ); nên tồn tại m0 2 N sao cho m0 < và jS(f; Pm ) I(f )j < ;
2 3
8m > m0 :
Như vậy nếu jP j < , ta có
jS(f; P ) I(f )j jS(f; P ) S(f; R)j + jS(f; R) S(f; Pm )j + jS(f; Pm ) I(f )j
" " "
< + + = ":
3 3 3
Định lý 1 được chứng minh xong.
Chương 7. TÍCH PHÂN RIEMANN 110

7.2 Các tính chất cơ bản của phép tính tích phân
Định lý 2. Cho f; g : [a; b] ! R liên tục, c 2 (a; b); ; 2 R: Khi đó
(i) Các hàm f + g; jf j khả tích trên [a; b];
Z b Z b Z b
(ii) ( f (x) + g(x)) dx = f (x)dx + g(x)dx;
Za b Z c Z ba a

(iii) f (x)dx = f (x)dx + g(x)dx;


Za b aZ
b
c

(iv) f (x)dx jf (x)j dx;


a a Z b Z b
(v) Nếu f (x) g(x); 8x 2 [a; b] thì f (x)dx g(x)dx;
Z x a a

(vi) Hàm F (x) = f (t)dt; 8x 2 [a; b]; liên tục trên đoạn [a; b] và có đạo hàm trong khoảng (a; b) và
a

F 0 (x) = f (x); 8x 2 (a; b);


F+0 (a) = f (a); F 0 (b) = f (b);

(vii) Cho f : [a; b] ! R liên tục trên [a; b]; có đạo hàm f 0 liên tục trong khoảng (a; b) ; thì
Z x
f (x) f (a) = f 0 (t)dt; 8x 2 [a; b];
a

(viii) (Công thức tích phân từng phần) Cho f; g : [a; b] ! R liên tục trên [a; b]; có đạo hàm liên tục
trong khoảng (a; b); thì
Z b Z b
0 b
f (x)g (x)dx = f (x)g(x)ja f 0 (x)g(x)dx;
a a

trong đó f (x)g(x)jba = f (b)g(b) f (a)g(a);


(ix) (Công thức đổi biến) Cho f : [a; b] ! R liên tục trên đoạn [a; b] và hàm ' : [ ; ] ! [a; b] liên
tục trên đoạn [ ; ] và có đạo hàm '0 liên tục trong khoảng ( ; ): Khi đó
Z Z '( )
0
f ('(t))' (t)dt = f (x)dx;
'( )

(x) Cho f : [a; b] ! f ([a; b]) là một song ánh, liên tục trên [a; b]; có đạo hàm liên tục trong khoảng
(a; b): Khi đó
Z b Z f (b)
f (x)dx + f 1 (y)dy = xf (x)jba = bf (b) af (a):
a f (a)

Chứng minh Định lý 2.


Chứng minh (i). Do các hàm f + g; jf j liên tục nên chúng khả tích trên [a; b]:
Chứng minh (ii), (iv), (v). 8P 2 P([a; b]); 8 ; 2 R; ta có

S( f + g; P ) = S(f; P ) + S(g; P ); jS(f; P )j S(jf j ; P );


S(f; P ) S(g; P ) nếu f (x) g(x); 8x 2 [a; b]:
Cho jP j ! 0; ta có (ii), (iv), (v) đúng.
Chứng minh (iii). Xét P = fx0 ; x1 ; ; xn ; 1 ; ; n g là một phân hoạch của đoạn [a; b] sao cho
c = xj0 2 fx0 ; x1 ; ; xn g; i = xi :
Hai phân hoạch của hai đoạn [a; c] và [c; b] lần lượt là

P1 = fx0 ; x1 ; ; xj0 ; 1; ; n g; P2 = fxj0 ; xj0 +1 ; ; xn ; j0 +1 ; ; n g:


Chương 7. TÍCH PHÂN RIEMANN 111

Các tổng Riemann của hàm f tương ứng với hai phân hoạch P1 ; P2 là
j0
X n
X
S(f; P1 ) = f ( i )(xi xi 1 ); S(f; P2 ) = f ( i )(xi xi 1 ):
i=1 i=j0 +1

Hơn nữa, ta còn có


S(f; P ) = S(f; P1 ) + S(f; P2 ):
Qua giới hạn trong đẳng thức trên khi cho M axfjP1 j ; jP2 jg = jP j ! 0; và với chú ý rằng do f liên
tục trên các đoạn [a; b]; [a; c]; [c; b]; ta có
Z b Z c Z b
S(f; P ) ! f (x)dx; S(f; P1 ) ! f (x)dx; S(f; P2 ) ! f (x)dx;
a a c

và cuối cùng ta thu được (iii).


Chứng minh (vi). 8x; y 2 [a; b]; x < y; ta có
Z y Z y
jF (y) F (x)j = f (t)dt jf (t)j dt
x x

Do f : [a; b] ! R liên tục trên đoạn [a; b]; do đó nó bị chận trên đoạn [a; b], tức là tồn tại M > 0, sao
cho jF (x)j M; 8x 2 [a; b]: Do đó ta có

jF (y) F (x)j M jy xj ; 8x; y 2 [a; b]:

Vậy hàm F liên tục trên đoạn [a; b]:


8x 2 (a; b); và 8h > 0 đủ bé sao cho x + h 2 (a; b); ta có
Z
F (x + h) F (x) 1 x+h
f (x) = [f (t) f (x)] dt
h h x
Z
1 x+h
jf (t) f (x)j dt
h x
sup jf (t) f (x)j :
x t x+h

Cho " > 0; do f liên tục tại x; tồn tại > 0; < minfb x; x ag sao cho

jf (t) f (x)j < "; 8t 2 (x ; x + ):

Vậy nếu 0 < h < ; dẫn đến [x; x + h] (x ; x + ); do đó

F (x + h) F (x)
f (x) sup jf (t) f (x)j ":
h x t x+h

F (x + h) F (x)
Vậy tồn tại lim và = f (x): Điều này dẫn tới F+0 (x) = f (x):
h!0+ h
Tương tự, thực hiện lại lý luận trên với h < 0, ta cũng có tồn tại F 0 (x) = f (x):
Vậy F có đạo hàm trong khoảng (a; b) và F 0 (x) = f (x); 8x 2 (a; b):
Tương tự, như trên ta cũng có F+0 (a) = f (a); F 0 (b) = f (b):
Vậy (vi) đúng.
Z x
Chứng minh (vii). Xét hàm số g(x) = f (x) f (a) f 0 (t)dt; 8x 2 [a; b]:
a
Theo (vi), g có đạo hàm trên [a; b]; và

g 0 (x) = f 0 (x) f 0 (x) = 0; 8x 2 (a; b);


Chương 7. TÍCH PHÂN RIEMANN 112

do đó g là hàm hằng trên [a; b]; vậy ta có

g(x) = g(a) = 0; 8x 2 [a; b];

tức là (vii) đúng.

Chứng minh (viii). Áp dụng (vii) với F = f g, ta có F : [a; b] ! R liên tục trên [a; b]; có đạo hàm F 0
liên tục trong khoảng (a; b); khi đó
Z b Z b
0
F (b) F (a) = F (x)dx = f (x)g 0 (x) + f 0 (x)g(x) dx;
a a

hay
Z b Z b
0
f (x)g (x)dx = f (b)g(b) f (a)g(a) f 0 (x)g(x)dx:
a a

tức là (viii) đúng.


Z x Z '(t)
Chứng minh (ix). Xét hàm số F (x) = f (y)dy; 8x 2 [a; b] và hàm G(t) = F ('(t)) = f (y)dy;
a a
t 2 [ ; ]:
Ta có G0 (t) = F 0 ('(t))'0 (t) = f ('(t))'0 (t); 8t 2 ( ; ): Ta có G : [ ; ] ! R liên tục trên [ ; ]; có
đạo hàm G0 liên tục trong khoảng ( ; ); khi đó áp dụng (vii) với hàm G(t); ta có
Z
G( ) G( ) = G0 (t)dt;


Z Z
G0 (t)dt = f ('(t))'0 (t)dt;
Z '( ) Z '( ) Z '( )
G( ) G( ) = f (y)dy f (y)dy = f (y)dy;
a a '( )

do đó (ix) đúng.
Z x Z f (x)
Chứng minh (x). Xét hàm số F (x) = f (t)dt + f 1 (y)dy; 8x 2 [a; b]: Ta có F liên tục trên
a f (a)
[a; b]; có đạo hàm F 0 liên tục trong khoảng (a; b); và

F 0 (x) = f (x) + f 1
(f (x))f 0 (x) = f (x) + xf 0 (x) = (xf (x))0 :

Khi đó áp dụng (vii) với hàm F (x); ta có


Z b Z b
F (b) F (a) = F 0 (x)dx = (xf (x))0 dx = xf (x)jba = bf (b) af (a);
a a

mà F (a) = 0; do đó (x) đúng.


Định lý 2 được chứng minh xong.

7.3 Tích phân suy rộng


Trường hợp a; b 2 R; a < b; f : (a; b) ! R:
Định nghĩa 3 (Tích phân suy rộng loại 2, các cận hữu hạn) Cho f : (a; b) ! R thỏa điều kiện:
(i) f khả tích trên mọi đoạn [c; d] (a; b);
Chương 7. TÍCH PHÂN RIEMANN 113

(ii) Tồn tại số thực I(f ) sao cho 8" > 0; 9 > 0 :
Z d
8(c; d) 2 [(a; b) \ (a; a + )] [(a; b) \ (b ; b)] =) f (x)dx I(f ) < ".
c
Z b
(Điều kiện (ii) có nghĩa là tồn tại lim f (x)dx = I(f ) 2 R).
c!a+ ; d!b a Z b
Khi đó, ta gọi I(f ) là tích phân suy rộng của f trên (a; b) và ký hiệu là f (x)dx.
a

Trường hợp b = +1; a 2 R; f : [a; +1) ! R:


Định nghĩa 4 (Tích phân suy rộng loại 1, có cận vô hạn) Cho f : [a; +1) ! R thỏa điều kiện:
(i) f khả tích trên mọi đoạn [a; b] [a; +1);
(ii) Tồn tại số thực I(f ) sao cho 8" > 0; 9B > a :
Z b
8b > B =) f (x)dx I(f ) < ".
a
Z b
(Điều kiện (ii) có nghĩa là tồn tại lim f (x)dx = I(f ) 2 R)
b!+1 a Z +1
Khi đó, ta gọi I(f ) là tích phân suy rộng của f trên tia [a; +1) và ký hiệu là f (x)dx.
a
Trường hợp a = 1; b 2 R; f : ( 1; b] ! R:
Định nghĩa 5 (Tích phân suy rộng loại 1, có cận vô hạn) Cho f : ( 1; b] ! R thỏa điều kiện:
(i) f khả tích trên mọi đoạn [a; b] ( 1; b];
(ii) Tồn tại số thực I(f ) sao cho 8" > 0; 9A < b :
Z b
8a < A =) f (x)dx I(f ) < ".
a
Z b
(Điều kiện (ii) có nghĩa là tồn tại lim f (x)dx = I(f ) 2 R)
a! 1 a Z b
Khi đó, ta gọi I(f ) là tích phân suy rộng của f trên tia ( 1; b] và ký hiệu là f (x)dx.
1

Định nghĩa tương tự cho các tích phân suy rộng loại 2 cho các hàm xác định trên [a; b); (a; b];
[a; c) [ (c; b] và các tích phân suy rộng loại 1 cho các hàm xác định trên (a; +1); ( 1; b).
Định lý 3. Cho a; b 2 R và giả sử f : (a; b) ! R liên tục sao cho f ((a; b)) bị chận. Khi đó tồn tại
Z b
tích phân suy rộng f (x)dx của f trên (a; b).
a
Chứng minh Định lý 3. Do f ((a; b)) bị chận, ta có M > 0 sao cho jf (x)j M; 8x 2 (a; b). Cho hai
dãy fan g; fbn g (a; b); sao cho an < bn ; 8n 2 N; an ! a; bn ! b:
8m; n 2 N; ta có
Z bn Z bm Z bn Z bn Z bn Z bm
f (x)dx f (x)dx = f (x)dx f (x)dx + f (x)dx f (x)dx
an am an am am am
Z am Z bn
= f (x)dx + f (x)dx
an bm
Z am Z bn
f (x)dx + f (x)dx
an bm
M (jam an j + jbm bn j) ! 0; khi m; n ! 1:
Chương 7. TÍCH PHÂN RIEMANN 114

Z bn
Do đó, f (x)dx là dãy Cauchy trong R nên nó hội tụ về số thực L:
an
Cho hai dãy bất kỳ fAn g; fBn g (a; b); sao cho An < Bn ; 8n 2 N; An ! a; Bn ! b: Ta sẽ chứng
Z Bn
minh rằng f (x)dx ! L; khi n ! 1:
An
8n 2 N; ta có
Z Bn Z bn Z Bn Z Bn Z Bn Z bn
f (x)dx f (x)dx = f (x)dx f (x)dx + f (x)dx f (x)dx
An an An an an an
Z an Z Bn
= f (x)dx + f (x)dx
An bn
Z an Z Bn
f (x)dx + f (x)dx
An bn
M (jAn an j + jBn bn j) ! 0; khi n ! 1:

Do đó
Z Bn Z Bn Z bn Z bn
f (x)dx L f (x)dx f (x)dx + f (x)dx L
An An an an
Z bn
M (jAn an j + jBn bn j) + f (x)dx L ! 0; khi n ! 1:
an
Z Bn
Vậy f (x)dx ! L; khi n ! 1:
An
Định lý 3 được chứng minh xong.
S
n
Định nghĩa 6. Cho a = a0 < a1 < a2 < < an = b: Đặt A = (ai 1 ; ai ) : Cho f : A ! R liên
i=1
n Z
X ai
tục trên A sao cho f (A) bị chận. Khi đó theo Định lý 3, tích phân suy rộng f (x)dx tồn tại và
i=1 ai 1
Z b
được gọi là tích phân Riemann của f trên (a; b) và được ký hiệu là f (x)dx:
a
Định nghĩa 7 (Định nghĩa khác, dùng các tổng Riemann trên và dưới). Cho f : [a; b] ! R bị chận
trên đoạn [a; b], ta đặt

L(f ) = supfL(f; P ) : P 2 P([a; b])g;


U (f ) = inffU (f; Q) : Q 2 P([a; b])g:
Z b
Nếu L(f ) = U (f ) ta nói hàm f khả tích Riemann trên [a; b] và ký hiệu L(f ) = U (f ) = f (x)dx và
Z b a

gọi f (x)dx là tích phân của f trên [a; b]:


a

Định lý 4. Cho f : [a; b] ! R liên tục, ta đặt

L(f ) = supfL(f; P ) : P 2 P([a; b])g;


U (f ) = inffU (f; Q) : Q 2 P([a; b])g:

Khi đó U (f ) = L(f ):
Chứng minh Định lý 4. Dùng các bài tập bên dưới.
Chú thích. Cho f : [a; b] ! R liên tục, ta có L(f ) = U (f ) = I(f ):
Do liên tục, ta có L(f ) = U (f ): Ta chỉ cần chứng minh I(f ) = U (f ):
Chương 7. TÍCH PHÂN RIEMANN 115

b a
Với phân hoạch R = P [ Pm như trên (trong chứng minh Định lý 1), nếu < và jP j < , ta có
2m
"
jS(f; R) S(f; P )j < ;
3
"
jS(f; Pm ) I(f )j < ;
3
"
jS(f; R) S(f; Pm )j < :
3
Mặt khác
L(f; P ) L(f; R) S(f; R) U (f; R) U (f; P );

" 2" 2"
I(f ) < S(f; Pm ) + < S(f; R) + U (f; P ) + ;
3 3 3
" 2" 2"
I(f ) > S(f; Pm ) > S(f; R) L(f; P ) :
3 3 3
Ta suy ra
2" 2"
L(f; P ) < I(f ) < U (f; P ) + ; 8P 2 P([a; b]); jP j < :
3 3
Từ bất đẳng thức vế phải
2"
I(f ) inffU (f; P ) : P 2 P([a; b]); jP j < g +
3
2" 2"
inffU (f; P ) : P 2 P([a; b]) < g + = U (f ) + ;
3 3
Từ bất đẳng thức vế trái
2"
I(f ) supfL(f; P ) : P 2 P([a; b]); jP j < g
3
2" 2" 2"
supfL(f; P ) : P 2 P([a; b])g = L(f ) = U (f ) ;
3 3 3
Do đó
2" 2" 2"
U (f ) I(f ) U (f ) + ; hay jU (f ) I(f )j < "; 8" > 0:
3 3 3
Vậy I(f ) = U (f ) = L(f ):
Định lý 4 được chứng minh xong.
Bài tập 1: Cho f : [a; b] ! R liên tục, và P = fx0 ; x1 ; ; xn ; 1 ; ; n g; Q = fy0 ; y1 ; ; ym ;
1 ; ; m g hai phân hoạch của đoạn [a; b]: Giả sử fx0 ; x 1 ; ; xn g fy 0 ; y 1 ; ; ym ]: Chứng minh rằng
U (f; Q) U (f; P ):
Giải bài tập 1. Đặt Ii = fj : xi 1 < yj xi g = fj : [yj 1 ; yj ] [xi 1 ; xi ]g, ta có f1; 2; ; mg =
S
n
I1 [ I2 [ [ In = fj 2 N : [yj 1 ; yj ] [xi 1 ; xi ]g:
i=1 P
Ta cũng chú ý rằng [yj 1 ;yj ] [xi 1 ;xi ] (yj yj 1 ) = (xi xi 1 ): Từ đó
m
X n
X X
U (f; Q) = sup f (y)(yj yj 1) = sup f (y)(yj yj 1)
j=1 yj 1 y yj i=1 [yj 1 ;yj ] [xi 1 ;xi ]
yj 1 y yj

n
X X
sup f (y)(yj yj 1)
i=1 [yj xi 1 y xi
1 ;yj ] [xi 1 ;xi ]
n
X X
= sup f (y) (yj yj 1)
i=1 xi 1 y xi
[yj 1 ;yj ] [xi 1 ;xi ]
n
X
= sup f (y)(xi xi 1) = U (f; P ):
i=1 xi 1 y xi
Chương 7. TÍCH PHÂN RIEMANN 116

Bài tập 2: Cho f : [a; b] ! R liên tục, và P = fx0 ; x1 ; ; xn ; 1 ; ; n g; Q = fy0 ; y1 ; ; ym ;


1; ; m g hai phân hoạch của đoạn [a; b]:
Giả sử fx0 ; x1 ; ; xn g fy0 ; y1 ; ; ym g: Chứng minh rằng L(f; P ) L(f; Q):
Giải bài tập 2. Đặt
m
X n
X X
L(f; Q) = inf f (y)(yj yj 1) = inf f (y)(yj yj 1)
yj 1 y yj yj 1 y yj
j=1 i=1 [yj 1 ;yj ] [xi 1 ;xi ]
n
X X
inf f (y)(yj yj 1)
xi 1 y xi
i=1 [yj 1 ;yj ] [xi 1 ;xi ]
n
X X
= inf f (y) (yj yj 1)
xi 1 y xi
i=1 [yj 1 ;yj ] [xi 1 ;xi ]
n
X
= inf f (y)(xi xi 1) = L(f; P ):
xi 1 y xi
i=1

Bài tập 3: Cho f : [a; b] ! R liên tục, và P = fx0 ; x1 ; ; xn ; 1; ; n g; Q = fy0 ; y1 ; ; ym ;


1; ; m g hai phân hoạch của đoạn [a; b]: Chứng minh rằng L(f; P ) U (f; Q):
Giải bài tập 3. Đặt fx0 ; x1 ; ; xn g [ fy0 ; y1 ; ; ym g = fz0 ; z1 ; ; zk g:
Xét phân hoạch mới R = fz0 ; z1 ; ; zk g ta có theo hai bài tập trên

L(f; P ) L(f; R) U (f; R) U (f; Q).

Bài tập 4: Cho f : [a; b] ! R liên tục, ta đặt

L(f ) = supfL(f; P ) : P 2 P([a; b])g;


U (f ) = inffU (f; Q) : Q 2 P([a; b])g:.

Chứng minh rằng L(f ) U (f ):


Giải bài tập 4. Ta có
L(f; P ) U (f; Q); 8P; Q 2 P([a; b]):
Cho 8P 2 P([a; b]); ta có
L(f; P ) U (f; Q); 8Q 2 P([a; b]):
Ta suy ra
L(f; P ) inffU (f; Q) : Q 2 P([a; b])g = U (f ):
Điều này đúng với mọi P 2 P([a; b]); ta suy ra

L(f ) = supfL(f; P ) : L 2 P([a; b])g U (f ):

Bài tập 5: Cho f : [a; b] ! R liên tục, ta đặt

L(f ) = supfL(f; P ) : P 2 P([a; b])g;


U (f ) = inffU (f; Q) : Q 2 P([a; b])g:

Chứng minh rằng U (f ) = L(f ):


Giải bài tập 5. Ta xét một phân hoạch đều Pn của đoạn [a; b] như sau

b a
Pn = fx0 ; x1 ; ; xn g với xi = a + i ; i = 0; 1; ; n:
n
Ta có
L(f; Pn ) L(f ) U (f ) U (f; Pn ):
Chương 7. TÍCH PHÂN RIEMANN 117

Do đó
n
" #
X
0 U (f ) L(f ) U (f; Pn ) L(f; Pn ) = sup f (x) inf f (x) (xi xi 1 ):
xi 1 x xi xi 1 x xi
i=1

Do f liên tục trên [a; b] nên f liên tục đều trên [a; b]: Cho " > 0; tồn tại >0:
"
8x; y 2 [a; b]; jx yj < =) jf (x) f (y)j < .
b a
b a b a
Nếu jPn j = < , ta có jxi xi 1j jPn j = < ; 8i = 1; ; n; do đó jf (x) f (y)j <
n n
"
; 8x; y 2 [xi 1 ; xi ]; 8i = 1; ; n:
b a
Từ đó
" "
f (y) < f (x) < f (y) + ; 8x; y 2 [xi 1 ; xi ]; 8i = 1; ; n;
b a b a
" "
f (y) inf f (x) f (y) + ; 8y 2 [xi 1 ; xi ]; 8i = 1; ; n;
b a xi 1 x xi b a
" "
sup f (y) inf f (x) sup f (y) + ; 8i = 1; ; n;
xi 1 y xi b a xi 1 x xi xi 1 y xi b a
"
0 sup f (y) inf f (x) :
xi 1 y xi xi 1 x xi b a

0 U (f ) L(f ) U (f; Pn ) L(f; Pn )


n
" #
X
= sup f (x) inf f (x) (xi xi 1)
xi 1 x xi xi 1 x xi
i=1
Xn
" "
(xi xi 1) = (b a) = ":
b a b a
i=1

Ta có 0 U (f ) L(f ) "; 8" > 0: Vậy U (f ) = L(f ):


Chương 7. TÍCH PHÂN RIEMANN 118

7.4 Bài tập bổ sung


1. Cho f (x) = x; 0 x 1:
(i) Chứng minh rằng f khả tích Riemann trên [0; 1],
Z 1
1
(ii) Sử dụng phân hoạch đều hãy chứng tỏ rằng xdx = :
0 2
2. Cho f (x) = x ; 0 x 1; trong đó = 2 hay = 3:
(i) Chứng minh rằng f khả tích Riemann trên [0; 1],
Z 1
1
(ii) Sử dụng phân hoạch đều hãy chứng tỏ rằng x dx = :
0 + 1
3. Cho f; g : [a; b] ! R liên tục trên [a; b]. Chứng minh rằng
(i) f g khả tích trên [a; b].
Z b Z b 1=2 Z b 1=2
(ii) jf (x)g(x)j dx 2
f (x)dx g(x)dx :
a a a
4. Cho f; g : [a; b] ! R liên tục trên [a; b]. Chứng minh rằng
Z b Z b 1=4 Z b 3=4
4
(i) jf (x)g(x)j dx jf (x)j dx jg(x)j4=3 dx ;
a a a
Z b Z b 1=p Z b (p 1)=p
p p=(p 1)
(ii) jf (x)g(x)j dx jf (x)j dx jg(x)j dx ; 1 < p < 1;
a Z b a Z b a Z b
(iii) min jg(x)j jf (x)j dx jf (x)g(x)j dx max jg(x)j jf (x)j dx :
a x b a a a x b
( Z a 1=n
)
b
5. Cho f : [a; b] ! R liên tục trên [a; b]. Chứng minh rằng dãy jf (x)jn dx hội tụ và
a
Z b 1=n
n
lim jf (x)j dx = max jf (x)j :
n!+1 a a x b
Hướng dẫn Bài 5 : M = max jf (x)j ; ta có
a x b

jf (x)j M; 8x 2 [a; b];


Z b Z b
jf (x)jn dx M n dx = (b a)M n ;
a a
Z b 1=n
jf (x)jn dx (b a)1=n M;
a
Z b 1=n
1
hay jf (x)jn dx M:
b a a
( Z b 1=n
)
1
Để chứng minh rằng dãy jf (x)jn dx hội tụ, ta chỉ cần chứng minh rằng
b a a

( Z 1=n
)
b
1 n
jf (x)j dx là dãy không giảm.
b a a

Z b Z b 1=p Z b (p 1)=p
p p=(p 1)
Áp dụng bất đẳng thức jF (x)G(x)j dx jF (x)j dx jG(x)j dx ; với
a a a
Chương 7. TÍCH PHÂN RIEMANN 119

n+1
G(x) = 1; F (x) = jf (x)jn ; p = ; ta có p=(p 1) = n + 1:
n
Z Z n Z 1
b b n+1 n+1 b n+1
n n n+1
jf (x)j dx (jf (x)j ) n dx 1 dx
a a a
Z n
b n+1 1
n+1
= jf (x)j dx (b a) n+1 ;
a
Z Z 1
b 1=n b n+1 1
n n+1
hay jf (x)j dx jf (x)j dx (b a) n(n+1)
a a
Z 1
b n+1 1 1
n+1
= jf (x)j dx (b a) n (b a) n+1
a
Z b 1=n Z b 1=(n+1)
1 n 1 n+1
hay jf (x)j dx jf (x)j dx :
b a a b a a
( Z 1=n
) ( Z 1=n
)
b b
1 n 1 n
Vậy dãy jf (x)j dx là dãy không giảm và bị chận trên, do đó jf (x)j dx
b a a b a a
hội tụ.
Mặt khác, do (b a)1=n ! 1; khi n ! +1; từ đẳng thức
Z b 1=n Z b 1=n
1
jf (x)jn dx = (b a)1=n jf (x)jn dx ;
a b a a
( Z 1=n
)
b
chứng tỏ rằng dãy jf (x)jn dx hội tụ. Do đó
a

Z b 1=n
lim jf (x)jn dx M:
n!+1 a

Cho " > 0; do M = max jf (x)j = sup jf (x)j ; ta có x0 2 [a; b] : jf (x0 )j > M ":
a x b a x b
Nếu x0 2 (a; b); do tính liên tục của f tại x0 ; nên 9 > 0; < minfx0 a; b x0 g : jf (x)j > M ";
8x 2 (x0 ; x0 + ) ( ; ):
Nếu x0 = a; do tính liên tục của f bên phải tại a; nên 9 > 0; < b a : jf (x)j > M ";
8x 2 (a; a + ) ( ; ):
Nếu x0 = b; do tính liên tục của f bên trái tại b; nên 9 > 0; < b a : jf (x)j > M ";
8x 2 (b ; b) ( ; ):
Trong các trường hợp trên, ta đều có một khoảng ( ; ) (a; b) sao cho jf (x)j > M "; 8x 2 ( ; );
do đó
Z b Z
n
jf (x)j dx (M ")n dx = ( ) (M ")n ;
a
Z b 1=n
n
jf (x)j dx ( )1=n (M ") :
a
( Z 1=n
)
b
n
Cho n ! +1; do ( )1=n ! 1; do dãy jf (x)j dx hội tụ, ta suy ra
a

Z b 1=n
n
lim jf (x)j dx (M ") :
n!+1 a
Chương 7. TÍCH PHÂN RIEMANN 120

Từ hai bất đẳng thức trên ta có


Z b 1=n
M " lim jf (x)jn dx M:
n!+1 a

Z b 1=n
n
Bất đẳng thức này đúng với mọi " > 0; cho " ! 0+ ; ta suy ra lim jf (x)j dx = M:
Z x n!+1 a
6. Cho f : [a; b] ! R liên tục trên đoạn [a; b]. Đặt F (x) = f (x)dx; x 2 [a; b]: Chứng minh rằng
a
(i) F có đạo hàm trên đoạn [a; b];
(ii) Sử dụng hàm F để tính đạo hàm của các hàm sau
Z (x)
(j) F1 (x) = f (x)dx; trong đó : [a; b] ! [a; b] là hàm có đạo hàm trên đoạn [a; b];
a
Z (x)
(jj) F2 (x) = f (x)dx; trong đó ; : [a; b] ! [a; b] là hai hàm có đạo hàm trên đoạn
(x)
[a; b]:
Hướng dẫn Bài 6 (i):

F 0 (x) = f (x); a < x < b;


F+0 (a) = f (a);
F 0 (b) = f (b):
Z (x)
Hướng dẫn Bài 6 (ii): F1 (x) = f (x)dx = F ( (x)); F10 (x) = f ( (x)) 0 (x); a < x < b:
a Z (x)
Hướng dẫn Bài 6 (iii): F2 (x) = f (x)dx = F ( (x)) F ( (x)); F20 (x) = f ( (x)) 0 (x)
(x)
f ( (x)) 0 (x); a < x < b:
Z x2
7. Cho F (x) = cos(t2 )dt; x 1: Chứng minh rằng
1
(i) F có đạo hàm cấp 1 và cấp 2 tại mọi x > 1;
(ii) Tìm cực trị của hàm F ;
Z x2
(iii) Tồn tại lim cos(t2 )dt.
x!+1 1
Hướng dẫn Bài 7 (i), (ii):
r
0 4 4 4
F (x) = 2x cos(x ) = 0 () x = + k ; k 2 Z+ () x = xk = + k ; k 2 Z+ ;
2 2
F 00 (x) = 2 cos(x4 ) 8x4 sin(x4 );
F 00 (xk ) = 8x4k sin(x4k ) = 8x4k sin( +k )= 8x4k cos(k ) = 8x4k ( 1)k 1
:
2
F 0 (xk ) = 0 6= F 00 (xk ) = 8x4k ( 1)k 1

k = 2n chẵn : F 0 (x2n ) = 0 6= F 00 (x2n ) = 8x42n < 0 :


r
F đạt cực đại tại x2n = 4 + 2n ; n 2 Z+ ;
2
k = 2n + 1 chẵn : F 0 (x2n+1 ) = 0 6= F 00 (x2n+1 ) = 8x42n+1 > 0 :
r
F đạt cực tiểu tại x2n+1 = 4 + (2n + 1) ; n 2 Z+ :
2
Chương 7. TÍCH PHÂN RIEMANN 121

Hướng dẫn Bài 7 (iii):


Z x2
F (x) = cos(t2 )dt; x 1:
1
ds
s = t2 ; dt = p ;
2 s
Z x2 Z x4 Z 4
2 ds 1 x ds
F (x) = cos(t )dt = cos s p = cos s p
1 1 2 s 2 1 s
" Z x4 #
x4
1 1 1
= p sin s + 3=2
sin sds
2 s 1 1 2s
Z 4
sin x4 1 1 x sin s
= sin 1 + ds:
2x2 2 4 1 s3=2

sin x4 1 sin x4
Do ! 0 khi x ! +1; nên lim ! 0: Do đó lim F (x) tồn tại khi và chỉ khi
2x2 2x2 x!+1 2x2 x!+1
Z x4
sin s
lim ds tồn tại.
x!+1 1 s3=2
Ta sẽ sử dụng định lý sau

8" > 0; 9N 2 R :
lim H(x) tồn tại () =) jH(x) H(y)j < " :
x!+1 8x; y 2 R; x; y > N
Z x4
sin s
Đặt H(x) = ds; với mọi x; y > 1; giả sử x > y; ta có
1 s3=2
Z x4 Z x4 Z x4
sin s jsin sj 1
jH(x) H(y)j = ds ds ds
y4 s3=2 y4 s3=2 y4 s3=2
x4
2 2 2 2
= p = 2
+ 2 :
s y4 x y y2

2
Cho " > 0; Chọn N : < ": Khi đó, 8x; y > N =) jH(x) H(y)j < ":
N2
sin 1 L
Vậy tồn tại lim H(x) = L 2 R: Kết luận tồn tại lim F (x) = + :
x!+1 x!+1 2 4
BÀI TẬP CHƯƠNG I

122
Tài liệu tham khảo

[1] Bài giảng dựa trên các slides soạn theo quyển sách “Toán Giải Tích” của GS Dương Minh Đức, Nhà
xuất bản Thống Kê 2005. Các slides này được để trên webpage https://sites.google.com/view/dmduc-
giangday/ và được photocopy để sinh viên đọc trước khi nghe bài giảng.

[2] Dương Minh Đức, Phương pháp mới học toán Đại học, Tập 1, NXB Giáo dục, 2001.

[3] W. Rudin, Principles of mathematical analysis, McGraw-Hill, New York, 1964.

[4] Jean Marie Monier, Giải tích 2, 4, bản dịch tiếng Việt bởi Đoàn Quỳnh, Lý Hoàng Tú, NXB Giáo
dục, 2000.

[5] James Stewart, Calculus, Early Transcendentals, 7 th edition, Brooks Cole, 2018.

123

You might also like